[go: up one dir, main page]

0% found this document useful (0 votes)
2K views274 pages

Soal Unas Maret 2016 - Agustus 2020

1. The document contains 35 multiple choice questions related to obstetrics and gynecology. The questions cover topics like contraception counseling, cervical cancer screening, postpartum hemorrhage management, and ectopic pregnancy treatment. 2. The questions are part of a practice exam or assessment for medical students or residents to test their knowledge in OB/GYN. 3. Correct answers are not provided, as the purpose is to evaluate the test-taker's knowledge rather than simply check answers.

Uploaded by

Hambrian Wijaya
Copyright
© © All Rights Reserved
We take content rights seriously. If you suspect this is your content, claim it here.
Available Formats
Download as DOCX, PDF, TXT or read online on Scribd
0% found this document useful (0 votes)
2K views274 pages

Soal Unas Maret 2016 - Agustus 2020

1. The document contains 35 multiple choice questions related to obstetrics and gynecology. The questions cover topics like contraception counseling, cervical cancer screening, postpartum hemorrhage management, and ectopic pregnancy treatment. 2. The questions are part of a practice exam or assessment for medical students or residents to test their knowledge in OB/GYN. 3. Correct answers are not provided, as the purpose is to evaluate the test-taker's knowledge rather than simply check answers.

Uploaded by

Hambrian Wijaya
Copyright
© © All Rights Reserved
We take content rights seriously. If you suspect this is your content, claim it here.
Available Formats
Download as DOCX, PDF, TXT or read online on Scribd
You are on page 1/ 274

SOAL UNAS JAKARTA 19 MARET 2016

24. You suspect Asherman’s syndrome, and perform a hysterosalpingogram,which reveals


multiple synechiae within the uterus, confirming your suspicions. Your next step in therapy
is which of the following?
a. Diagnostic and operative hysteroscopy
b. Provera 10 mg daily for 5 days in an attempt to achieve a withdrawal bleed
c. In vitro fertilization
d. Place an intrauterine device
e. Inform your patient that unfortunately, she is “barren” and will not be able to carry a
pregnancy again

25. An 18-year-old G0 F presents to your office for contraceptive counseling. She has never
used any method of contraception before and is engaged in a monogamous sexual
relationship. Gynecologic history is significant for regular, heavy menstrual cycles using up
to eight pads per day, lasting up to 7 days at a time, with severe pain (dysmenorrhea). She
smokes one-half pack of cigarettes per day and tells you that her mother and aunt both
have Factor V Leiden disease, but that she has never been tested herself or had a
thromboembolic event. She will attend college soon and has no plans for a pregnancy in
the near future. She indicates her desire for the “most reliable” method of contraception
that you can offer.
Which of the following methods of contraception has the least efficacy?
a. Ortho Evra patch
b. Combined oral contraceptive pills
c. Mirena IUD
e. Condoms with spermicide
f. Coitus interruptus

26. Of course, during your discussion at this visit, you could encourage smoking cessation and
recommend weight loss to help improve her overall health. You and the patient have
decided to proceed with IUD placement. Prior to placement, it is important to perform
which of the following tests?
a. FSH level

REVISED BY TEAM “BANG JAGO” NOV 2020 UNHAS Page 1


b. Prolactin level
c. Urine pregnancy test
d. Gonorrhea/Chlamydia testing
e. Both c and d

27. 30-year-old G0 comes in for her annual examination and tells you that she plans to become
pregnant sometime in the next year. She had a LEEP procedure 5 years ago at another
facility for moderate dysplasia. You verify her Pap tests have all been negative since the
LEEP, but her last pap was 2 years ago.
What is the recommended cervical cancer screening for this patient?
a. Pap testing every 6 months
b. Pap testing and colposcopy every 6 months
c. Pap testing every 3 years
d. Pap testing and high-risk HPV testing every 5 years
e. She has been treated with the LEEP, so she no longer needs pap smears

28. Her Pap test returns HSIL and she is high-risk HPV positive. You have her return for
colposcopy. After the application of acetic acid, you see a large, dense, white area with
mosaic vessels encompassing the entire anterior cervix and extending into the endocervical
canal. You obtain a biopsy of this area and performan endocervical curettage. The
pathology report for both biopsies is CIN III. What treatment do you recommend?
a. LEEP in office
b. Cryotherapy
c. Cold-knife cone or two-stage LEEP in OR
d. Simple hysterectomy
e. Radical hysterectomy

29. A 62-year-old woman presents to the office complaining of watery vaginal discharge and
bleeding for the past 2 months. She has not had a Pap test in 14 years. She states she had a

REVISED BY TEAM “BANG JAGO” NOV 2020 UNHAS Page 2


mildly abnormal pap in her 30s, but that was treated with cryotherapy. She states she went
through menopause at age 50 and has never been on hormone replacement therapy. She
does admit to smoking one-half pack a day for 40 years. Her husband is deceased, and she
has not been sexually active in 10 years. Her examination reveals a cervical necrotic mass
approximately 5 cm in size. Rectovaginal examination is suspicious for left parametrial
involvement. There is no evidence of adnexal masses, but examination of the uterus and
adnexa is limited by the patient’s body habitus. You suspect this may be cervical cancer.
You obtain a Pap smear and take a biopsy of her cervical abnormality. The Pap test returns
with a reading of SCC, and the biopsy confirms this diagnosis. She also received a
cystoscopy for hematuria with positive urine cytology. The biopsy also shows SCC. You
order a CT scan, which shows a cervical mass measuring 7.7 3 5.0 cm as well as an avid left
internal iliac lymph node consistent with locally metastatic disease.
What is the International Federation of Gynecology and Obstetrics (FIGO) stage for her
cancer?
a. Stage I
b. Stage II
c. Stage III
d. Stage IV

30. Mrs. S, 34 years, G4P3A139 weeks gestational age, second stage of labour. The head of the
baby was delivered but the shoulder was stuck. Turtle sign (+). The mother has an
uncontrolled diabetes for 5 years. Estimated fetal weight by ultrasound was 4200 gram.
What is this condition called ?
a. After coming head
b. Compaction
c. Retention of the baby
d. Shoulder dystocia
e. Collision

31. Mrs. S, 32-year-old G4P3Ao gravid with chronic hypertension had a normal labor that
arrested in second stage at +1 station. She complained of mild dyspneu and fatigue . The
fetus had a left occiput anterior presentation and was delivered by forceps. Completion of

REVISED BY TEAM “BANG JAGO” NOV 2020 UNHAS Page 3


third stage followed quickly, and the fundus was noted to be firm . The OBGYN was
carefully examined, and no laceration were noted. The examiner then noted the lower
uterine segment was boggy.
Which of the following is suitable treatment in situation above?
a. Hemabate, 250 μg intramuscularly
b. Methergin, 0,2 mg intramuscularly
c. A 20-unit oxytocin intravenous bolus
d. Misoprostol 400 mg, intrarectally  kurang dosisnya
e. Carboprost 0,25 mg, intramuscularly

32. For Mrs. S, one dose uterotonic agent is given and the fundus is massaged. Despite this, she
continues to bleed. Which of the following is suitable treatment in this situation?
a. Administer methergin 0,2 mg intramuscularly
b. Mobilize a team that include obstetricians, nurses, and anesthesiologists
c. Perform laparotomy to prepare postpartum hysterectomy
d. Apply balloon catheter
e. Uterine and ovarian artery ligation

33. The patient continues to bleed and you have initiated whole blood transfusion. Which of
the following is suitable treatment in this situation?
a. Continue to administer Hemabate intramuscularly every 20 minutes
b. Insert Bakri postpartum Balloon or large Folley catheter balloon into the uterine cavity
and inflate the balloon
c. Consider laparotomy and uterine compression suture placement
d. all of the above
e. a and b

REVISED BY TEAM “BANG JAGO” NOV 2020 UNHAS Page 4


34. A pelvic ultrasound reveals a right-sided ectopic pregnancy as well as large amounts of
fluid, thought to be blood in the abdomen. She now has IV access and a bolus of IV fluids
is being given. Her BP is now 78/45 and her pulse rate is 112 beats perminute. Her
hematocrit returns as 27.2%. How will you proceed?
a. Administer IM methotrexate
b. Transfuse the patient with two units of packed RBCs and transfer her to the ICU
c. Proceed with a laparoscopic salpingectomy
d. Proceed with emergent laparotomy
e. Start vasopressors and transfer the patient to the ICU

35. An 89-year-old female patient with multiple, serious medical comorbidities presents to
discuss options for treatment of her high- grade prolapse. The prolapse is externalized
and becoming ulcerated from friction against her undergarments. She cannot tolerate a
pessary. Her main priority is to “fix or get rid of this thing,” but her primary care provider
has cautioned against a lengthy or open abdominal procedure. She is not interested in
future intercourse. What can you offer this patient?
a. Nothing can be done
b. Open abdominal sacral colpopexy
c. Robot-assisted laparoscopic sacral colpopexy
d. Hysterectomy with anterior and posterior colporrhaphy, vault suspension
e. Colpocleisis

36. The drug that blocks the conversion of arachidonic acid to thromboxane A2 while sparing
prostacyclin production in treatment APAs
A. Unfractionated heparin
B. Aspirin
C. Azathioprine
D. Cyclosporine
E. Cyclophosphamid

37. A 28-year-old G2P0 at 39 weeks is in early labor. She is 2 cm dilated and 90% effaced, with
contractions every 4 to 5 minutes. The fetal heart tones are reassuring. Her nurse steps

REVISED BY TEAM “BANG JAGO” NOV 2020 UNHAS Page 5


out for a moment and returns to find her having a seizure. The nurse administers a 4-g
magnesium bolus. The seizure stops. The fetal heart tone variability is flat, but there are
no decelerations. This patient is most at risk for mortality from which of the following
complications?
(A) infection
(B) uremia
(C) congestive heart failure
(D) fever
(E) cerebral hemorrhage

38. A 40-year-old woman is seen for a routine examination. Her menses have been regular,
and she has no complaints. Findings, including those on pelvic examination, are normal.
Ten days later, her Pap smear is returned as “high-grade squamous intraepithelial
lesion.” Which of the following options is the best course of action?
(A) immediate wide-cuff hysterectomy
(B) repeated Pap smears at 3-month intervals
(C) fractional dilation and curettage (D&C)
(D) punch biopsy of anterior cervical lip
(E) colposcopy with biopsy

39. A pelvic ultrasound reveals a right-sided ectopic pregnancy as well as large amounts of
fluid, thought to be blood in the abdomen. She now has IV access and a bolus of IV fluids
is being given. Her BP is now 78/45 and her pulse rate is 112 beats per minute. Her
hematocrit returns as 27.2%. How will you proceed?
A. Administer IM methotrexate
B. Transfuse the patient with two units of packed RBCs and transfer her to the ICU
C. Proceed with a laparoscopic salpingectomy
D. Proceed with emergent laparotomy
E. Start vasopressors and transfer the patient to the ICU

40. G3P2A0, 32 years old, referred from primary health care with term pregnancy and post
cesarean section. She has a history of cesarean delivery of her first child due to placenta
previa. Factors affecting the likelihood of succesful trial of labor in this patient is :
a. Prior cesarean for malpresentation
b. Spontaneous labor with advanced cervical dilatation on admission
c. Prior vaginal delivery
d. Increased maternal age
e. Recurrent indication for initial cesarean delivery

REVISED BY TEAM “BANG JAGO” NOV 2020 UNHAS Page 6


41. With regard to HIV in Pregnancy :
A. A positive HIV blood test in pregnancy is not reliable
B. A high maternal HIV RNA load decreases the mother-to-child transmission of HIV
C. Use of antiretroviral agents is always commened in the first trimester of pregnancy
D. HIV infection increases the mother to-child transmission of the hepatitis C virus
E. If there are ruptured membrane for 6 hours, there is no advantage to delivery baby by
C – Section

42. A One woman, married, G3P3A0, 43 yo, no history of abdominal surgery, has been getting
a diagnosis as cystic ovarian neoplasm with a size of 6 months pregnant. Limited mobility
mass, not pain. Hb 11,5 gr%, Ca-125 IU. Other laboratory examinations, investigations and
cytology smears perioperative smear within normal limits.
Choose the correct answer :
A. Vertical midline incision offers inferior access to the upper abdomen if greater space or
access is needed
B. Transverse incision are used commonly in gynecologic malignancy surgery.
C. Vertical midline incision was not recommended in patients who have coagulopathy,
declined transfusion, or are administered systemic anticoagulation
D. Nerve injury to the iliiohypogastric, ilioinguinal and femoral nerves in Pfannenstiel
incision was more commonly involve sensory functionand typically are transient

44. Mrs. S, 29 yo, G3P1A1 39 weeks GA, referred by midwife with prolonged second stage. In
examination, revealed normal vital sign, obstetric examination revealed contraction was
3x/10’/35’. FHR was 160 x/m, estimated fetal weight was 3100 gr, previous baby was
3000 gr. Vaginal examination revealed : full dilatation, amniotic membrane was absent,
thick and greenish, lowest part head with descent of the head was Hodge IV.
Denominator was minor fontanella at the left anterior.
What will you choose to terminate this condition ?
a. Spontaneous delivery
b. Augmentation
c. Embryotomy
d. Forcipal extraction
e. C-Sec

REVISED BY TEAM “BANG JAGO” NOV 2020 UNHAS Page 7


45. Frequent complication of the action above is :
A. Uterine rupture
B. Cephalhematoma
C. Parese N. VII
D. Erb’s Paralysis
E. Fracture of os femur.

46. FHR shows declining arises from beginning of contraction and goes to normal baseline as
soon as the contraction ends. This finding refer to ?
A. Fetal distress
B. Fetal head compression
C. Fetal hypoxia
D. Umbilical cord compression
E. Threatened uterine rupture

47. A 19-year-old primigravida at 29 weeks gestation is noted to have anemia with a


hemoglobin concentration of 8 g/dL. The peripheral blood smear below is obtained.
Which of the following laboratory findings are microcytic hypokrom ?
a. Decreased serum ferritin level (n : 41-141)
b. Elevated mean corpuscular volume
c. Decreased total iron binding capacity
d. Positive sickle-cell screen (Sickledex) result

48. Ms. Smith is a 37-year-old multigravida who presents to your office at 32 weeks' gestation
as calculated by her last menstrual period. Her hematocrit is 29 volume percent, and she
has sickle cell trait. During sonographic evaluation, the fetus has biometric values that
correlate with a 28-week fetus.
What is the most likely explanation?
a. Aneuploidy
b. Chronic hypoxia
c. Poor pregnancy dating
d. First-trimester cytomegalovirus infection

49. Your next obstetrical sonographic evaluation of the patient in Question 74-76 is
performed 4 weeks after the fust one and now at an estimated gestational age of 36
weeks. The fetus now has measurements similar to a 30-week fetus. Growth restriction
seems more likely. What is appropriate at this time?
a. Delivery

REVISED BY TEAM “BANG JAGO” NOV 2020 UNHAS Page 8


b. Strict bed rest
c. Umbilical artery Doppler velocimetry
d. Sonographic fetal biometry in 1 week

50. Solid food as one of the source of nutritional diet in post caesarean patient will be offered
within:
a. 2 hours post operative
b. 3 hours post operative
c. 8 hours post operative
d. 12 hours post operative
e. 24 hours post operative

51. A 19-year-old woman complain of a golf ball-sized mass at the entrance of her vagina. She
says that this area is “sore all the time” and began hurting “about 3 days ago”. On
examination, the patient has a tender 4 cm mass on the lateral aspect of the labia minora
at the 5 o’clock position. There is erythema and edema, and the area is very tender and
fluctuant. No cellulitis is noted.
What is the most appropriate treatment for this condition?
a. Trimethoprim/sulfamethoxazole
b. Azithromycin for the patient and any sexual partners
c. Incision and drainage of the mass followed by a course of
trimethoprim/sulfamethoxazole
d. Incision and drainage of the mass
e. Incision and drainage of the mass with placement of a Word catheter.

52. Which of the following is generally the treatment of choice for recurrent Bartholin gland
duct abscess?
a. Systemic antibiotics
b. 5-percent lidocaine ointment
c. Bartholin gland duct marsupialization
d. Warm compresses and frequent sitz baths
e. All of the above

53. The incubation period of syphilis is which of the following?


A. 1-7 days
B. 10 days
C. 3-90 days (10-90 days after initial infection)
D. 120-180 days

REVISED BY TEAM “BANG JAGO” NOV 2020 UNHAS Page 9


E. 210 days

54. A 30-year-old multigravida presents with ruptured membranes at term but without labor.
Following induction with misoprostol, her labor progresses rapidly, and she
spontaneously delivers a liveborn 3300-g neonate. Immediately after delivery, she
complains of dyspnea. She becomes apneic. Her autopsy reveal fetal squames within
pulmonary vasculature. How would her death be classified?
A. Perinatal death
B. Nonmaternal death
C. Direct maternal death
D. Indirect maternal death
E. Occasional maternal death

55. From pelvic examination findings, pelvic brim : round, diagonal conjugate 12 cm,
symphisis parallel to sacrum, subpubic angle is acute, convergent side walls, bituberous
diameter is 7 cm. By analyzing your findings, which causes bellow is unlikely to be?
A. Android pelvis
B. Anthropoid pelvic
C. High assimilation pelvis
D. Platypelloid pelvis
E. Oblique pelvis

56. Your patient has microcytic anemia with a hemoglobin 9 and normal iron stores (normal :
41-141). What is the most likely diagnosis?
(A )folate deficiency
(B) vitamin B12 deficiency
(C) thalassemia beta minor
(D) vitamin B6 deficiency
(E) acute blood loss

58. You are counseling a 30-year-old woman who wants to become pregnant. Which of the
following is the most accurate method for her to time intercourse?
(A) thermogenic shift in basal body temperature (BBT)
(B)urinary luteinizing hormone (LH) kit testing
(C)serum progesterone level
(D) profuse, thin, acellular cervical mucus
(E)mittelschmerz

REVISED BY TEAM “BANG JAGO” NOV 2020 UNHAS Page 10


60. Fecal incontinence is most likely related to which of the following?
(A) interplay between the pubococcygeus muscle and rectum
(B)innervation of the pelvic floor and the anal sphincters
(C)normal colonic transit time
(D) nulliparity
(E)urinary retention

61. What should be advised to this woman regarding her delivery plan?
A. If cesarean delivery is planned, it should be scheduled at 36 weeks gestation
B. In labor, internal monitors should be placed because fetuses of HIV-infected women
are at increased risk for distress
C. In labor, with a plan for vaginal delivery, amniotomy should be performed as soon as
possible to hasten delivery
D. Cesarean delivery is recommended for women with viral load > 1000 copies/mL
E. Cesarean delivery is only performed based on obstetrical indication

62. When creating a Pfannenstiel incision, which vessels should be anticipated halfway
between the skin and fascia, several centimeters from the midline?
a. External pudendal
b. inferior epigastric
c. Superficial epigastric
d. Superficial circumflex iliac
e. iliac vein

63. The sonographic appearance of endometrium during menstrual cycle cprrelates with the
phasic change in histological anatomy. Which phase of the cycle is depicted with the
classic trilaminar appearance shown below
A. Menstrual
B. Secretory
C. Proliferatif
D. Periovulatory

64. For surgical treatment of endometriomas which following approaches superior?


A. Drainage
B. Cystectomy
C. Cys wall ablation
D. Oovorectomy

REVISED BY TEAM “BANG JAGO” NOV 2020 UNHAS Page 11


65. Sonography endometriomas are typicaly describe as which following?
A. Solid with intracystic internal echoes
B. Cystic with hyperechoic internal choes
C. Solid diffuse internal low level echoes
D. Cystic with diffuse internal low level echoes
E. Cystic with intracystic blood flo

66. In PCOS increased testosterone production from the ovaries is secondary to stimulation
by which of the following hormone?
A. Inhibin
B. Estradiol
C. LH
D. FSH

67. Which of the following is a not risk factor for vulvar cancer?
A. Lichen planus.
B. Tobacco abuse
C. Lichen sclerosus
D. HPV
E. HSV

68. What most common site of metastatic spread of choriocarsinoma


A. Brain
B. Liver.
C. Lungs
D. Vagina

69. Regarding the coagulation system in pregnancy, which of the following statement is true.
A. Mean platelet count is 250000
B. Fibrinolityc activity is usualy reduced
C. Fibrinogen level are increased to a median of 250mg/dl
D. Decreases in platelet concentration are solely due to hemodilution

70. Compared with a normally shaped placenta, which complication of 3rd stage labor is
more common with underdiagsed succenturiate lobe?
A. Cord avultion
B. Chorioamnitis

REVISED BY TEAM “BANG JAGO” NOV 2020 UNHAS Page 12


C. Uterine invertion
D. Retained Cotelidon

71. In the US which of the following twin pregnancies would be candidate for fetoscopic laser
ablation therapy for TTTS?
A. Monochorionic diamnionic twins at age 23 week gestation with stage 1 TTTS
B. Dichorionic diamnionic twins at 19 weeks gestation with stage 11 TTTS
C. Monochorionic, diamniotic twins at 15 weeks gestation with stage IV TTTS
D. Monochorionic diamniotic twins at 21 weeks gestation with stage III TTTS

72. Which of the following defines heterotopic pregnancy?


A. One tubal and one abdominal pregnancy
B. One ectopic and one intrauterine pregnancy
C. Two pregnancy, one in each fallopian tube
D. Two ectopic pregnancys in one fallopian tube

73. This deceleration most likely reflects which of the following? (late deceleration)
A. Head compression
B. Cord compression
C. Maternal chronic anemia
D. Preeclampsia
E. Uteroplasenta insufficiency

74. When performing step down in this image as a part of a peripartum hysterectomy
particular care must be taken avoid injury to what structure?
A. Ureter
B. Bladder
C. Urethra
D. Bowel

75. Ninety five percent of patiens with SLE experienced all except which of the following
clinical manifestation?
A. Fever
B. arthralgia
C. Proteinuria
D. Weight loss
E. Myalgia
76. Which of the following clinical scenarios meets the definition of amenorrhea?

REVISED BY TEAM “BANG JAGO” NOV 2020 UNHAS Page 13


A. 12 yo with Tanner stage breast development
B. 16 yo with tanner stage II breast development (14/16+seks sek)
C. 14 yo with tanner stage III breast development
D. 18 yo with tanner stage V breast development and cessation of menses for the last one
cycles
E. 18 yo with tanner stage V breast development and cessation of menses for the last two
cycles

77. A 25 yo lady G1 term pregnancy comes to delivery room in active phase of labor. You
perform CTG, and you find the CTG as the following (di gambar : early deceleration)
This deceleration most likely reflects which of the following?
a. Head compression
b. Cord compression
c. Maternal chronic anemia
d. Severe preeclampsia
e. Uteroplacental insufficiency

78. Which of the following twin pregnancies would be candidate for fetoscopic laser ablation
therapy for TTTS?
a. Monochorionic diamnionic twins at age 23 week gestation with stage I TTTS
b. Dichorionic diamnionic twins at 19 weeks gestation with stage II TTTS
c. Monochorionic diamniotic twins at 15 weeks gestation with stage IV TTTS
d. Monochorionic di amniotic twins at 21 weeks gestation with stage III TTTS
e. Monochorionic di amniotic twins at 14 weeks gestation with stage III TTTS

79. A 55 year old healthy woman undergoes exploration for a large pelvic mass. Frozen
section analysis of her right ovary notes “mucinous low malignant potential, cannot
exclude invasion”. There is no othe obvious disease. What surgical procedures should be
performed, in addition to TAH and BSO ?
a. Pelvic washing, omentectomy, multiple peritoneal biopsy
b. Pelvic washing, omentectomy, multiple peritoneal biopsy, bilateral pelvic and oara
aortic lymph node dissection
c. Pelvic washing, omentectomy, multiple peritoneal biopsy, bilateral pelvic and oara
aortic lymph node dissection, appendectomy
d. The procedure done was completed
e. Omentectomy, multiple peritoneal biopsy, bilateral pelvic and oara aortic lymph node
dissection

REVISED BY TEAM “BANG JAGO” NOV 2020 UNHAS Page 14


80. In the post menopausal woman with uterine bleeding, evaluation of the endometrium
maybe accomplished by endometrial biopsy, hysteroscopy, or TVS. Which of the following
sonographic endometrial thickness is commonly used as a threshold to indicate low risk
for endometrial hyperplasia or cancer?
a. 1 mm
b. 5 mm
c. 7 mm
d. 10 mm
e. 12 mm

81. A defect in the anterior abdominal wall that abdominal contents of the fetus covered only
by a two layered sac of amnion and peritoneum :
a. Omplalocele
b. Gastroshisis
c. Diaphragmatic hernia
d. Duodenal atresia
e. Posturethral value

82. Which of the following is not risk factor for vulvar cancer?
a. Lichen planus
b. Tobacco abuse
c. Lichen sclerosis
d. Human papilloma virus
e. Herpes simplex virus

83. Which of the following is the most common cause of first trimester pregnancy loss?
a. Uterine anomalies
b. Incompetence cervix
c. Intrauterine infection
d. Fetal chromosomal abnormality
e. Placenta adhesive

94. A 22 year old G1P0, has just undergone a spontaneous vaginal delivery. As the placenta is
being delivered, an inverted uterus prolapses out of the vagina. The maneuver most likely
worsen the situations would be to
a. Immediately finish delivering the placenta by removing it from the inverted uterus

REVISED BY TEAM “BANG JAGO” NOV 2020 UNHAS Page 15


b. Call for immediateassistance from other medical personnel
c. Obtain intravenous access and give lactated Ringer solution
d. Apply pressure t the fundus with the palm of the hand and fingers in the direction of
the long axis vagina
e. Have anesthesiologist administer halothane anesthesia for uterine relxation

95. A 40 years old lady came to antenatal clinic for routine check up. She is G3P2, in 16 weks of
gestation. Her previous history is unremarkable, and physical examination corresponds to
16 weeks of gestation. She is really concern about her pregnancy in advanced maternal
age. Which of the following is not included in the quadruple test for aneuploidy?
a. NT examination
b. Maternal s erum AFP
c. hCG
d. Unconjugated estriol
e. Inhibin alpha

96. Which of the following symptoms of adenomyosis is correctly paired with its etiology?
a. Dysmenorrhea – Increased prostaglandine production
b. Dysmenorrhea – hemorrhage within the ecopic glandular foci
c. Menorrhagia - increased and abnormal vascularization of the adenomyotic tissue
d. Infertility – increased and abnormal vascularization of the adenomyotic tissue
e. Its severity doesn’t correlate with ectopic foci and degree of invasion

97.A 24 year old G3PP2 present with vaginal bleeding, a BhCG level 300,000 miU/mL, utetrine is
consistent with a 12 week gestation, B negative blood tupe and the sonographic correspond to
honeycomb appearance. What is the most appropriate management?
a. Plan for hysterectomy
b. Rhogam administration and bed rest
c. Plan for dilatation and curettage
d. Repeat a serum BhCG level in 48 hours
e. Repeat a serum BhCG level in 24 hours

98. A 30 yo G1 32 weeks came to policlinic with dysuria since 3 days before. Her BP was 120/80
mmHg, pulse 92 bpm, RR 20 x/I. Fever was denied, urinary frequency was present. Lower
abdominal tenderness was positive, no sign of costovertebral pain. From her complete blood
count reveals Hb 10,8 g/dL, HCT 32%, leucocyte 14000, PLT 167000. Urinary tract changes in
pregnancy:

REVISED BY TEAM “BANG JAGO” NOV 2020 UNHAS Page 16


a. During pregnancy, the rs of upper urinary tract infection is similar compared to non
pregnant women
b. Urinary tract changes during pregnancy caused by progesterone induced relaxation of
the muscularis
c. some changes in urinary tract develop after 14 weeks of pregnancy
d. Glomerular filtration rate usually decreased 20% in pregnancy
e. During pregnancy, kidney become smaller and renal calyces become dilated

REVISED BY TEAM “BANG JAGO” NOV 2020 UNHAS Page 17


AGUSTUS 2016

1. A 34 yo G3P2002 woman at 38 weeks and 6 days was admitted to labor and delivery unit
for active management of labor after it was determined that her membranes had
ruptured and she was fully dilated and the second stage begins. Which of the following is
the correct order of the cardinal movements of labor?
a. Internal rotation, engagement, descent, flexion, external rotation
b. Engagement, descent, internal rotation, flexion, external rotation
c. Internal rotation, descent, engagement, flexion, external rotation
d. Engagement, descent, flexion, internal rotation, external rotation
e. Engagement, descent, internal rotation, flexion, external rotation

A 24 years old woman at 32 weeks gestation complain of shortness of breath during her
pregnancy especially with phsival exertion . She has no prior medival history. Her respiratory
rate is 16x/m,her lungs clear to auscultation and your oxygen saturation monitor reveals her
oxygen her satutarions to be 98% room air

2. Your reassure her that this snstion is normal and explain which of the following?
a. Airway conductance is decreased during pregnancy
b. Because of enlarging uterus pushing uterus pushing up on the diagphram,her vital
capacity is decreased by 20%
c. Maximal breathing capacity is not altered by pregnancy

REVISED BY TEAM “BANG JAGO” NOV 2020 UNHAS Page 18


d. Pulmonary resistence increased during pregnancy
e. Small amniotic fluid emboli are shed throught pregnancy

3. A patient call your clinic complaining of continued heavy vaginal bleeding .She has an
uncomplicated vaginal birth 2 weeks ago of her second child. What is the most likely
diagnosis from the following differentials?
a. Coagulopaties
b. Retained placental fragments
c. uterine atony
d. Uterine rupture
e. Vaginal Laserations

4. A woman is worried because she has been taking body bulding steroids through week qo
of her pregnancy . One of the steroids has a strong propotions of androgens. You explain
that androgens can cause which of the following>
a. Paramesonephros to differentiate into the proximal urinary duct system
b. Wolffian ducts to develop
c. Mullerian duct to regress
d. the primitive vaginal tube to regress
e. The gonadal ridge to differentiate into testis

5. You are counseling a couple about infertility. In your discussion about conception,tuba
disease, and implantation, you explain to them that implantation in the uterus occurs at
which stage of development?
a. Eight-cell embryo
b. Zygote

REVISED BY TEAM “BANG JAGO” NOV 2020 UNHAS Page 19


c. Morula formation
d. Blastocyst
e. Embrionic disk late for 2 weeks and had positive

6. What is the average thereshold level of b-HCG count can exclude intrauterine pregnancy
if an empty uterus visualized on TVS
a. 25-100 mIU/ml
b. 100-500 mIU/ml
c. 500-1000 mIU/ml
d. 1000-1500 mIU/ml
e. 1500-2000 mIU/ml

A 32 years old woman, G3P2 37 wga, came to ER with contractions 5 minutes apart. On
examine ion you found that the fetus is breech, and she’s already fully dilated with breech is on
hodge IV. You asked the mother to bear down, after the umbilicus, the baby won’t decent even
after 2 times bearing down, you found nuchal arm.
7. Which is the maneuver to help release the nuchal arm?
a. Classic maneuver
b. Mau riceau-smell ie-veit maneuver
c. Lovsett maneuver
d. Brecht maneuver
e. Muller maneuver

For question no 92-93


A 63-year-old lady, Parity 5, presents to you with a complaint of LLQ pain, intermittent nausea,
abdominal pressure, and bloating. Her history is notable for mild obesity, right breast cancer,
and hypertension. She and her husband desired children but were never able to conceive. Her
family histories showing a left ovarian mass containing internal septations and papillary
excrescences. She has moderate ascites and her Ca-125 was 719
8. Which of the following is associated with an increased risk of ovarian cancer?
a. History of breast cancer
b. Breastfeeding
c. Multiparity
d. Tubal ligation

REVISED BY TEAM “BANG JAGO” NOV 2020 UNHAS Page 20


e. Obesity

9. She underwent TAH, BSO, collection of pelvic washings, omentectomy, cytoreduction or


“debulking”, and bilateral pelvic and para-aortic lymph node sampling. The mass had
spread beyond the ovary to the omentum, peritoneum, and bowel. She was found to
have ascites, and pelvic washings were positive. What stage of ovarian cancer does she
have?
a. Stage-I
b. Stage-II
c. Stage-III
d. Stage-IV
e. Stage-V

For question no 96-97


A 36-year-old G2P0 at 26 weeks gestational age presents to the office with palpitations, anxiety,
lack of sleep, and a 5 kg weight loss over the past 2 weeks. She started to develop substernal
chest pain this morning. She has no significant medical history. On examination, she is a febrile
with a heart rate of 152 and a BP of 158/82. She appears anxious and her eyes prominently
open. Her thyroid is enlarged bud. An EKG reveals sinus tachycardia without ST segment
changes.

10. Which of the following is the appropriate immediate first line therapy?
a. Radioactive iodine
b. Propanolol
c. Lorazepam
d. Propylthiouracil (PTU)
e. Hydrocortisone

11. Which clinical symptiom is not characteristic of mild thyrotoxicosis?


a. Thyromegaly
b. Tachycardia
c. Cold intolerance
d. Failure to gain weight
e. Palpitation

REVISED BY TEAM “BANG JAGO” NOV 2020 UNHAS Page 21


A couple presents because they have been trying to conceive for 18 months. During the
interview you learn that the man has fathered a child in a previous relationship and is in good
health. The woman is 28 and reports that she has had painful menses for the past 5 or 6 years.
12. You begin to suspect that she may have endometriosis. All of information below would
increase that suspicion EXCEPT :
a. She reports that a maternal cousin has a history of endometriosis
b. She has experiemced dyspareunia with deep penetration for several years
c. Her ethnicity is Caucasian
d. She report the development of abnormal bleeding in the last year
e. Her menarche began at age 9

13. After completing your history you explain to your patient that you need to perform an
examination before making any recommendations. You explain that women with
endometriosis often have a normal examination but that there are certain findings that
are associated with endometriosis. During your examination, which of the findings listed
below would NOT increase your suspicion that she has endometriosis.
a. A fixed deviated uterus
b. Uterosacral nodularity on rectovaginal examination
c. Tender adnexa
d. An enlarged irregular uterus
e. A fixed adnexal mass

A 21-years-old G2P1 with a history of asthma present to the emergency room at 25 weeks
gestational age complaining of an acute exacerbation of her astgma. On presentation she is
tachypneic and struggling to breathe.
14. All of the following are appropriate next steps to the management of this patients,
EXCEPT?
a. Arterial blood gas determination
b. Intubation with ventilator assistance
c. Oxygen therapy
d. Subcutaneus epinephrine
e. Intravenous methylprednisolone

REVISED BY TEAM “BANG JAGO” NOV 2020 UNHAS Page 22


REVISED BY TEAM “BANG JAGO” NOV 2020 UNHAS Page 23
NOVEMBER 2016

1. A 25-year-old lady G1 term pregnancy comes to delivery room In active phase of labor.
Your perform CTG, and you find the CTG as the following. This deceleration most likely
reflect which of the following? pict.
A. Head compression  early
B. cord compression  variabel
C. Maternal chronic anemia  sinusoidal
D. Severa pre eclampsia
E. Uteroplacental insufficiency  late

2. Y 29 yar old woman presents at 36 weeks of gestation will shortness of breath without 3
clear alternative diagnosis on history or examination. She is short of breath while
climbing half a flight of stairs. She has left leg oedema more than right leg oedema Your
next step in diagnostic management would be which og the following?
A. Apply the Welis dinical model for puimonary embolism
B. D-diner test
C. Bilateral leg ultrasound Imaging
D. CT scann of the chest
E. V/Q scan

REVISED BY TEAM “BANG JAGO” NOV 2020 UNHAS Page 24


3. In the presence of proteinuria and hypertension after 20 gestasional week, with
unknown previous history.
A. The differential diagnosis between PET and CKD Is always possible if prepregnancy
data are available
B. Severely Impaired utero-placental if flows and fetal growth suggest CKD
C. Normal fit PIGF ratio suggest CKD
D. Protemuria 3 day suggest CKD
E. Proteinuria above 6g/day suggest CKD

4. Which of the following drugs should be discontinued during pregnancy?


A. low dose aspirin .
B. bisoprolol
C. atenolol
D. aglotensin Converting Enzyme Inhibitors (ACEI)
E. Furosemid

5. A 75-year old woman has a bilateral, solid adnexal masses. Mammography is normal,
Gastrointestinal studies show a stomach lesion suspicinus.for malganncy. Which of the
following Is the most likely diagnosis?.
A. Pick’s adenoma
B. Krukenberg’s tumor
C. brenner tumor
D. struma ovarii
E. Carcinoid

REVISED BY TEAM “BANG JAGO” NOV 2020 UNHAS Page 25


Mrs. S, 29 years old. G3P1A1 39 weeks of gestasional age, referred by midwife with prolonged
second stage. in examination, revealed normal vital sign in. Obstetric examination recealed
contraction was 3x/10'/25*. Fetal heart rate was 160x/m. examination fetal weight was 3100
gams. Previos baby was 3000 gams. Vaginal examination revealed: full dilatation, amniotic
membrane was absent thick and greenish, lowes part was head with descents or the head was
hodge IV. Denominator was minor fontanella at the left anterior

6. What will you choose to terminate this condition


A. Spontaneous delivery
B. Augmentation
C. Embryology
D. Forceps extraction
E. Cesarean section

7. What are Indicatlon for assisted vaginal delivery?


A. Uterine rupture
B. PPROM
C. Pastpartum haemorrhage
D. Fetal anomaly
E. Fetal or maternal distress

8. Frequent compilation of the action above is


A. Uterine rupture
B. cephal haematoma
C. Erb's paralysis

REVISED BY TEAM “BANG JAGO” NOV 2020 UNHAS Page 26


D. Fracture os femur
E. Parese NVII

An 18 year old G0 F presents to your office for contraceptive counseling. She has never used
any method of contraceotion before and is engaged in a monogamous sexual relatonship.
Gynecologic history is significant for regular, heavy menstrual cycles using Up to tight pads per
day, lasting up to 7 days at a time, with severe pain (dysmenorrhea). She smokes one-half pack
of cigarettes per day and tells you that her mother and aunt have Factor V Leiden disease, but
that se has never been tested herseif or had a thromboembalic event. She will attend college
soon and has no plans for a pregnancy in the near future. She indicates her desire for the
“most reliable” method of contaceptionthat you can offer.

9. Which of the following methods of contraception has the least efficacy?


A. Orto evra patch
B. Combined oral contraceptive pills
C. Mirena IUD
D. Condoms with spermicide
E. Coitus interuptus
10. Physical examination In the office reveals a blood pressure of 140/85mmHg, pulse of 80,
and BMI Of 40, Abdomen is soft, nonender, and genitourinary exam is unremarkable
with no cervical inflamation. Which of the following would be the best Choice
contraception for this patient?
A. Orto evra patch
B. Combined oral contraceptive pilis
C. Mirena IUD
D. Condoms with spermicide
E. Coitus interuptus
11. Of course, during your discussion at this Visi, you Could encourage soking cessation and
recommend weight loss to help improve her overali health. You and the patient have
decided to proceed with IUD replacement. Prior to placement, It is important to perform
which of the following test?

REVISED BY TEAM “BANG JAGO” NOV 2020 UNHAS Page 27


A. FSH level
B. Prolactin level
C. Urine pregancy test
D. Gonorrheal/chlamydia testing
E. Both c and d

A 19 year old woman complain of a golf bali sized mass at the entrance of her vagina. She says
that this area is “store all the time” and began hurting “about 3 days go”. On examiantion, the
patient has a tender4 cm mass on the lateral aspect of the labia mnora Rt the 5 o'clock position.
There is erythema and edema, and the area Is very tender and fluctuant. No celulitis Is noted

12. What Is the most appropriate tratement for this condition?


A. Trimethoprim/suifamethoxazole
B. Azithromycin for the patient and any sexual partners
C. Incision and drainage of the mass followed by 3 course of trimethoprim /
sutfarnethoxazole
D. incisio and drainage of the mass
E. incision and drainage otf the mass with placement of Word catheter
13. Bartholin gland duct cyst form indirect response to which of the following?
A. Vulvar irritation
B. Cervical gonorrhea
C. Glanc duct obstruction
D. Chronic lichen sclerosis
E. All of the above
14. Which of the following is generally the treatment of choice for recurrent Bartholin gland
duct abscess?
A. Systemicantibiotics
B. 5-percent lidocaine ointment
C. Bartholine gland duct marsupialization
D. Warm compresses and freguent sitz baths

REVISED BY TEAM “BANG JAGO” NOV 2020 UNHAS Page 28


A 62-year old woman presents ti the office complaining of watery discharge and bleeding for
the pass 2 months. She h as not had a Pap test In 14 years. She states she had a mildiy
abnormal pap in her 30s, but that was treated with cryotherapy. She states she went trough
menopause at age 50 and has never been on hormone replacement therapy. She does admit to
smoking obe half-pack a day for 40 years. Her husband is deceased, and she has not been
sexually actibe in 10 years. Her examination reveals a cervical necrotic mass approximatety 5
cm in size. Reectovaginal examination is suspicious for left parametrial involvement. There is no
evidence of adnexal masses, but exaination of the uterus and adnexa is limited by the patient's
body habitus.

You suspect this may be cervical cancer. You obtain a Pap smear and take biopsy of her cervical
abnormality. The Pap test returns with a reading of SCC, and the biopsy confirms this diagnosis.
She also received a cystoscopy for hematuri with positive urine cytology. The biopsy also show
SCC. You order a CT scan which shows a cervical mass measuring 7,7x315.0 cm as wel as an
avid left internal iliac Iymph node consisten with localty metastatic disease.

15. What is the International Federation of Gynecology and Obstetrics (FIGO) stage for her
cancer?
A. Stage I
B. Stage II
C. Stage III
D. Stage IV
16. What do you recommend for the next step treatmen of her cervical cancer?
A. Cold knife cone
B. Simple hysterectomy
C. Chemotherapy alone
D. Palliative care/chemorad
17. She Is treated with chemoradiation anda 3 years later has a recurrence. Yoy proceed with
pelvic exenteration for her recurrent cancer. What is her 5 year survival rate after the
Pelvic exenteration? ,
A. 5%

REVISED BY TEAM “BANG JAGO” NOV 2020 UNHAS Page 29


B. 10%
C. 25%
D. 50%
E. 90%

The couple came to klinik with complaint : want to get pregnant. This couple has been married
3 years. Current wife age 36 tahun, height 151cm, and weight 73 kg, a history of sexual
intercourse 2-3 times a week. From anamnesis we found a history of menstrual pain since the
age of 20 years and intermitten treatment for vaginal discharge odor complaint and itchy since
before married

18. In this couple :

You are going to make Clinical skill training. There are some problems that you are worried, In
reaction of successtuliy of taht training approach.

19. The trairing appproach describe in this chapter Is gulded by prindpies of adult learning
These penciples are based on the assumption that people participate In traing Courses
beacuse :
A. they are Interested In the topic, wish to improve their knowledge or skilis, and thus
their jov performance. Desire to be thus their job erformance, desire to be actively
Involve in course activities
B. Uses behavior modeling Is competency-based, Incorporates humanistic traing
echniques.
C. Competency based, which means assessment is keyed to the course objective ang
emphasized acquaring the essential knowldge, atitudinal conceps and skills need to
perform a job, not simply acquiring new knowledge.
D. Less steressfull because from the outset participant, both individually and as a
group, know what they are expected to learn and where to find the information, and
have ample opportunity for discussion with the clinical trainer.

REVISED BY TEAM “BANG JAGO” NOV 2020 UNHAS Page 30


Mrs. A, 24 years promogravida, 39 weeks gestational age, undergoes caesarean delivery due to
breech presentation. She deivereg female baby, with Dody weigh 3600 gr, APGAR score was 8/9

20. Post operatively maintenance intravenous adminsteres fluids are :


A. Ringer solution or a similar crystalloid solution with 5-percent dextrose.
B. 10% dextrose
C. Colloid solution
D. Crystakoid solution with 40-percent dertrose
E. Amino acid based fluid
21. Solid food 25 one f the source Of nutritional diet in post caesarean patient will be offered
within
A. 2 hours post operative
B. 3 hours post operative
C. 8 hours postoperative
D. 12 hours post operative
E. 2 hours post operative
22. Woman undergoing caesarean delivery have an inceased risk oof venous
thromboembolism comare with those delivering. In order to decrease the risk, what is
your suggestiont these women:
A. early ambulation
B. early feeding
C. Administered ansigesic
D. Inspected the Incision each day
E. Removed the skin closure on the fourth post operative day.

Mrs. Seliy 20 year old primipara Is 36 hours post partum following caesarean delivery for allure
to progress. She is compiaining of abdominal pain and has a fever of 38C. She is not yet
tolerating oral Intake because of nausea. you diagnose metritis

23. Which of the following Is the most Imortant criterionfor the diagnosis of post partum
metritis?

REVISED BY TEAM “BANG JAGO” NOV 2020 UNHAS Page 31


A. uterine tenderness
B. fever
C. Foul-smelling lochia
D. Leucocytosis
E. Leukopenia
24. Which of the following Is the most frewuent cause metritis?
A. Group A streptococcus
B. Group B strreptococcus
C. Chlamydia trachomais
D. Mycoplasma homonis
E. Ureaplasma ureatyticum
25. A defect in the anterior abdominal wal that abdominal content of the fetus Covered only
by a two-layerd sac af amnion and peritoneum
A. Omphalocele
B. Gastroschisis
C. Diaphragrnatic hernia .
D. Duodenal atresia
E. Post uretral value
26. In case of severe hyperemesis gravidarum, all EXCEPT of the following initial Initial
complication are common?
A. Acidosis
B. dehydration
C. Hypokalemi
D. Hyponatremi
E. Mild transaminitis
27. Which of the following twin pregnancies would be candidates for fetoscopic las laser
ablation therapy for twin-twin transfusio syndrome (TTTS)?
A. Monochorionic diamniotic twins at 23 weeks of gestation with stage I TTTS
B. Dichorionic diamniotic twins at 19 weeks gestation with stage II TTTS

REVISED BY TEAM “BANG JAGO” NOV 2020 UNHAS Page 32


C. Monochorionin diamniotic twins at 15 weeks gestation with stage IV TTTS
D. Monochorion diamniotic twins at 21 weeks gestation with stage III TTTS
E. Monochorionic diamniotic twins at 14 weeks gestation with stage III TTTS,
28. Which of of the following symptoms of adenmosis is correctiyy paired with Its etiology?”
A. Dysmenorrhea incrased prostaglandin production
B. Dysmenorrhea-hemerrhage within the ectopic glandular foci
C. Menorrhagia increased and abnormal vascularization of th eadenornyotic tissue
D. Infertility Increased and abnornal vascularization of the adenomyotic tissue
E. Its severity does not correlate with ectopic foci and degree ofinvasion
29. Regarding coagulation system in preganancy, which of the following statement Is true?
A. Mean platelet counts Is 250.000/ul
B. fibbrinotytic activity Is usualy reduced
C. fibibrinogen levels are increased
D. Decreases in platelet concentration are solely due to haemodilution
E. Is responsible for dependent edema In the lower extremities
30. A 55-year old healthy woman undergoes exploration for a large pelvic mass. Frozen
section analysis of her right ovary notes “mucinous low malignant potential, cannot
exclude Invasian"”, There s no othe obvious disease. What surgical procedures should be
performed, in addition to total abdominal hysterectomy and bilateral
salpngooophorectomy?
A. Pelvic washing, omentectomy, multiple peritoneal biopsies
B. Pelvic washings, omentectomy, multiple peritoneal biopsies, bilateral pelvic and
para aortic Iymph node dissection
C. Pelvic washings, omentectomy, multiple, peritoneal biopsies, bilateral pelvic and
para aortic lymph node dissection, appendictomy
D. the procedure done was completed
E. Omentectomy, multiple peritoneal biopsies pelvic and para aortic Iymph node
dissection

REVISED BY TEAM “BANG JAGO” NOV 2020 UNHAS Page 33


Mrs. S, 32 years old G4P3A0 gravid with chronic hypertension had a normal tabor that arrested
in second stage at +1 station..She complained of mild dyspneu and fatigue. The fetus had a left
occiput anterior presentation and was delivered hy forceps. Completion ofthird stage followed
qulckiy, and the fundus was noted to be firm. The OBGYN was carefully examined, and no
laceration were noted. The examiner then noted the lower uterine segment was boggy

31. Mrs. S one dose uterotonic agent Is given and the fundus is massaged. Despited this, she
continues to bleed. Which of the following is suit. Is suitable treatment in this situation?
A. administer methergin 0,2 mg Intramuscularly
B. mobilize a team that inciude obstetrician, nurse, anf anrstesiologist
C. Perform laparotomy to prepare post partum hysterectomy
D. Apply ballon catheter
E. Uterine and ovarian artery ligation
32. The patient continues to blees and you have initiated whole blood transfusion. Which Of
the following Is suitable treatmet in this slituation?
A. Continues to adnminister hemabate Intamuscularly every 20 minutes
B. Insert bakri post partum ballon or large folle catheter ballon into the uterine cavity
and inflate the ballon
C. Cinsider laparotomy and uterine compression suture placement
D. All of the above
E. A and B

Mrs. A, 24 years primigramda, 39 weeks gestational age, undergoes caesarean delivery due to
breech presentation She delivery female baby. with body weight 3600 grams, and APGAR Score
was 8/9.

33. Post operatively maintenance intravenous administered fluids are:


A. ringer soluiton or a Similar crystalloid Solution with 5-percent dextrose.
B. 10% dextrose
C. Colloid solution

REVISED BY TEAM “BANG JAGO” NOV 2020 UNHAS Page 34


D. Crystalloid solution wih 40-percent dextrose
E. Amino acid based fluid
34. Solid food as one of the source Of nutritional diet in post carsarean patient will be
offered within
A. 2 hours post operative
B. 3 hours post operative
C. 8 hours post operative
D. 12 hours post operative
E. 24 hours post operative
35. Women undergoing cesarean delivery have an increased risk of venous
thromboembolism compared with those delivering vagnaly. In order to decrease the risk,
what is your suggestion to Ihese women
A. Early ambulation
B. Earty feeding
C. Adamnistered analgesic
D. Inspected ihe incision each day
E. Removed the skin closure on the fouth postoperatifve day

Mrs. A 24 years, G2P1A0 39 weeks gestational, admitted to your emergency room with
complained abdominal cramping with bloody show. In examination, revealed normal vital sigh,
contraction, was 3x/10/35. Fetal, heart rate was 146x/m. Estimatod fetal weight was 1 gram.
baby was 2900 gram. examination revealed: dilatation was 4 cm, effacement 100% amniotic
membrane was intake. lowest pan as head with descent of the head was Hodge II. Denominator
was minor fentanella at the left side.

36. What is the condition of this patient?


A. Not in labour yet
B. In labour, stage 1 talent phase
C. In labour, stage 1 active phase
D. In labour, stage 2
E. In labour, stage 3

REVISED BY TEAM “BANG JAGO” NOV 2020 UNHAS Page 35


37. According the vignette above, what will to happened to that mother?
A. She will continue the progress of labour and will give birth normally
B. The progress of labour will be stucked in this phase of labour
C. She will underwent prolongned second stage
D. The second stage of labour must be lerminated by vacuum or forcipal extraction
E. The mother will face the possibility of post parfum haemorrhage

Mrs. Selly 20 year-old primipara. Is 36 hours postpartum following cesarean dalivery for failure
lo progress She Is complaning of abdominal pain and has a fever of 38 C. She is not yet
tolerating oral intake because of nausea, You diagnosis metritis

38. Which of the following is the most Important criterion for the diagnosis of postpartum
metritis?
A. Uterine tenderness
B. Fever
C. Foul-smelling lochia
D. Keukocytosis
E. leukopenia
39. which of the following is the most frequent cause metritis?
A. Group A streptococcus
B. Group B streptococcus
C. Chlamydia hominis
D. Mycoplasmna hominis
E. Ureaplasma urealyticum
40. A defect In the anterior abdominal wall that abdominal contents of the fetus covered
only by a two-layered sac of amniomn and peritoneum
A. Omphalocele
B. Gastroschisis
C. Diaphragmatic hernia
D. Duodenal atresia
E. Posturethral value

REVISED BY TEAM “BANG JAGO” NOV 2020 UNHAS Page 36


41. in cases dl severe hyperemesis gravidarum, al EXCEPT which of the followng initial
complications are common?
A. Acidosis
B. Dehydration
C. Hypokalemia
D. Hyponatremia
E. Mild transminitis
42. Which of the following twin pregnancies would be candidates for fetoscopic laser
ablation therapy for twin-twin translusion syndrome (TTTS)
A. Monochorionic diamnionie twins at 23 weeks of gestation with stage I TTTS.
B. Dichorionic diamnionic twins at 19 weeks Of gestation with stage II TTTS.
C. Monochorionic diamnionic twins at 15 weeks gestation with stage IVTTTS
D. Monochorionic diamnionic twins at 21 weeks gestation with stage III TTS.
E. Monochorionic diamnionic twins at 14 weeks gestation with stage III TTTS.
43. Which of the following Symptoms of adenomyosis is correctly pared with its etiology?
A. Dysmenorrhea-increased prostaglandin production
B. Dysmenorrhea- hemorrhage within the ectopic glandular foci
C. Menorrhagia-increased and abnormal vascularization of the adenomyotic tissue
D. Infertility- increased and abnormal vascularization of the adenomyotic tissue
E. its seventy does not correlate with ectopic loci and degree of invasion
44. What is the most common sile ol metastatic spread of choriocarinoma?
A. Brain
B. Liver
C. Lung
D. Vagina
E. Tuba
45. Regarding coagulation System in pregnancy, which of the following Statements is true?
A. Mean platelet count is 250.000/uL
B. Fibrinolytic acivity is usually reduced.

REVISED BY TEAM “BANG JAGO” NOV 2020 UNHAS Page 37


C. Fibninogen levels are increased.
D. Decreases In ptatelet concentration are solely due io hemodilution.
E. Is respansibls for dependent edema in the lower extremities.
46. A 55-year-old healtty woman undergoes exploration for a large pelvic mass. Frozen
section analysis of her right ovary notes “mucinous low malignant potential, cannot
exclude invasion.” What surgical procedures should be performed, in addition to total
abdominal hysterectomy and bialteral –salpingoophoreciomy?
A. Pelvic washings, omentectomy, multiple peritoneal biopsies
B. Pelvic washings, omentectomy, multiple peritoneal biopsies, bilateral pelvic and
para-aortic Iymph node dissection
C. Pelvic washings, omentectomy, multiple peritoneal biopsies, bilateral pelvic and
para-aortic tymph node dissection appendectomy
D. The procedure done was completed
E. Omenteclomy, multiple peritoneal biopsies, bilateral pelvic and para-aortic Iymph
node dissection
47. A 32-year-old G1P1 woman presents to your office with the chief complaint of
amenorrhea Since her most recent vaginaldelivery 1 year ago. She notes that she had an
uncomplicated pregnancy, followed by the delivery of a healthy baby boy. Her delivery
was Compicaled by an Intra-amniokc Inection as well as a postpartum hemomhage
requiring a postpartum dilation and curettage. After her delivery, she breastfed for 6
months, and during this time she had scant and irregular vaginal bleeding. After stopping
breastfeeding 6 monts ago she notes the absence of menses, but instead has monthly
painful cramping, which Seems to be getting worse. She remarks that. prior to her
pregnancy, she had normal, regular menses, which were not too heavy or painful. She
and her husband would like to have another child, and have been having unprotected
intercourse for the past 6 months wihout achleving a pregnancy. Your review of Systems
Is otherwise negative. You perform a physical examination, which is normal other than a
slightly enlarged, lender uterus. A urine pregnancy test in Ihe Office is negative.
What is the most likely diagnosis?

REVISED BY TEAM “BANG JAGO” NOV 2020 UNHAS Page 38


A. Sheehan's syndrome
B. Lactational amenorrhea
C. Asherman's syndrome
D. Premature ovarian failure
E. Kallmann syndrome
48. From a hysterosalpingogram, you find out multiple synechiae within the uterus. What is
your next step in therapy of this patient?
A. Diagnostic and operative hysteroscopy
B. Provera 10 mg daily for 5 days in an attempt to achieve a withdrawal bleed
C. In vitro fertilizatio
D. Place an intrauterine device
E. Inform your patient that unfortunately, she is “barren” and will not be able lo carry a
pregnancy again
49. The following are acceptable methods for confimation of oyulation
A. A drop in basal body temperature of at least 0.5°C on day 14
B. Blood progesterone level on day 21
C. Menstruation
D. Endocervical biopsy
E. Gonadotrophin level

A 27-year-old patient and her husband present to you primary infertility. The patient reports
regular periods every 28 to 30 days. The patient has no significant medical history and does not
take any medications. Her husband is also in good health, is 30 year of age, and has two
children from a previous marriage. When you asked the patient how long they have been trying
to achive a pregnancy, they tell you 6 months.

50. Your instructions to the couple ara he following


A. They will likely need IVF to achieve a pregnancy
B. They will likely need IUI cycles
C. cotineu trying appropriately timed intercourse for 6 more months and if no
pregnancy Is achieved, come back to sae you

REVISED BY TEAM “BANG JAGO” NOV 2020 UNHAS Page 39


D. Consider donor egg
E. Consider adoption
51. The couple comes back to you after appropniately timed intercourse, not having
achieved a pregnancy. At this time you embark on a workup that inludes a semen
analysis, an HSG, and an endocrine evaluaton including FSH, E2, TSH, prolactin levels, and
ovarian reserve testing. All of the tests come back normal Your next recommendation:
A. Have 6 more months of timed intercourse and if no pregnancy is achieved, come
back to see you
B. Clomphene citrate with IUI
C. IVF
D. Donor egg
E. Human gonadotropin (hMG)
52. Which Of the following delines heterotopic pregnancy?
A. One tubal and one abdomnad pregnancy
B. One eclopic and one Intrauterine pregnancy
C. Two pregnancies, one in each fallopian tube
D. Two pregnancies in one fallopian tube
E. Two tuba and one abdominal pregnancy
53. The american College of Obstetricians and Gynecologists considers which of the
following factors the most important in selecting a suitable candidate for trial of fabor
after cesarean section (TOLAC)?
A. Prior uterine incision type
B. Infection at the time of the orginal surgery
C. Gestational age at the time of the original
D. Degree of uterine distention during the current pregnancy
E. Cervical dilatation at the timee of hospital admission
54. Post operatively, after the urinary catheter removal the most common problem in the
female bladder is
A. Urinary tract infecton

REVISED BY TEAM “BANG JAGO” NOV 2020 UNHAS Page 40


B. Bladder atony caused by over distention
C. Overactive bladder
D. Bladder and urethral trauma
E. Urge incontinence
55. The incubation period Of Syphilis Is which Of the following?
A. 1-7 days
B. 10days
C. 3-90 days
D. 120180 days
E. 210 days
56. From pelvic examination findings: pelvic brim: round: diagonal conjugale 12 cm,
symphisis parallet to sacrum, subpubic angle is acute, convergent side walls, bituberous
diameter is 7 cm. By analyzing your findings which causes below is unlikely to be?
A. Android pelvis
B. Anthropoid pelvic
C. High assimilation pelvis
D. Platypelloid pelvis
E. Obique pelvis
57. Which of the following is a factor predisposing to the development of malignant mixed
mullerian tumors?
A. Prenatal exposure to diethyistilbestrol (DES)
B. Exposure to mumps virus
C. Family history of ovarian cancer
D. Previous pelvic irradiation
E. Perineal use of talc

Mrs S, 32.year-oid G4P3A0, gravid with chronic hypertension had a normal labor that arrested
in Second stage at +1 station. She complaned ofmild dypspneu and fatigue. The fetus had a left
occiput anterior presentation and was delivered by forceps. Completion of third stage followed

REVISED BY TEAM “BANG JAGO” NOV 2020 UNHAS Page 41


quickly, and the fundus was noted to be firm. The OBGYN was carefilly axamined, and no
laceration were noted. The axaminer then notad the lower uterine segment was boggy

58. For Mrs. S, one dose uterotonic agent Is givenand the fundus is massaged. Despite this,
she continues to bleed. Which Of he following is sutable treatmen! In this situation?
A. Adminster methergin 0,2 mg intramuscularly
B. Mobilze a tearmthat include obstetricians, nurse, and anesthesiolosts
C. Perform laparotomy to prepara postpartum hysterectomy
D. Apply balloon catheter
E. Uterine and ovarian artery ligation
59. The patient continues to bleed and you have initiated whole blood transfusion. Which of
the following is suitable treatment in this situation?
A. Continue to administer Hemabale intramuscularly every 20 minutes
B. Insert Bakrie postpartum Balloon ar large Folley catheter baloon into the uterine
cavity and inflate the balloon
C. Consider laparotomy and uterine compression suture placement
D. All of the above
E. A and B

A 38-years-old G3P2002 woman presents at 40 weeks 3 days with contractions to labor and
delivery triage. Contractions started 1 hour ago and are very painful. The patient denies leaking
fluid but did notice blood and mucus on her underwear. The baby has not been particularty
active since contractions started. Her prednancy has been complicated by A2GDM. Fasting
blood glucose are usually between 80 and 90 mg/dl with 1-hour postprandial values between
120 and 140 mg/dL. Her prepregnancy weight was 130 lb and she is 5 ft 5 in (BMI 21.6). She has
gained 30 lb this pregnancy (BMI 26.6). Hemoglobin A1C is 6.0%.. Fetal ultrasound at 20 weeks
demonstrated normal fetal anatomy. Repeat ultrasound for growth at 36 weeks demonstrated
fetus with weight in the 90th percentilc and an EFW of 4,350 g. Her last pregnancy was
complicated by A1GDM and she delivered a 4.200 g infant without complications. Initial cervical
examination reveals dilation of 6 cm, 50% effacement, and 21 station. Two hours Iater the
nurse calls you to the room after the patient's water breaks. Examination shows complete

REVISED BY TEAM “BANG JAGO” NOV 2020 UNHAS Page 42


dilation and effacement. and fetus at 11 station. The patient has a strong urge to push and
begins pushing. The delivery is complicated by a second-degree perineal laceration and a
postpartum hemorrhage of 600 ml. Fetal weight Is 4.560 g and the Apgar scores are 6, 8.

60. What neonatal risks are most commonly present in macrosomic fetuses ?
1. Jaundice 4. Birth trauma
2. Hypoglycemia 5. Asthma
3. Hyperglycemia 6. Hypocalcerna
A. 1,2,4,6
B. 134,5
C. 3,4,5
D. 1,3.,5,6
E. 1,2,5,6
61. Match the following fetal heart rate tracing (Figure 11-2) with the descriptive term that
besi fits the situation.

Figure 11-2. (Reproduced wilh permission from Cunningham F, et al Williams Obstetrics,


23rd ed. New York: MoGraw-Hill 2010, p. 418)

A. Early decelerations
B. Late decelerations
C. Normal tracing
D. Poor variability
E. Sinusoidal paltiem
62. A 49-year-oid woman presents complaining of vulva pain that increases with ambulation
and intercourse. She also notes a lump on her right labia that has increased in size over
the past 48 hours and is quite painfull. The patient has had these same symptoms in the
past. On examination she has a 5-cm tender cyst on the medial aspect of her right labia,

REVISED BY TEAM “BANG JAGO” NOV 2020 UNHAS Page 43


with a surrounding erythema of the labia that extends 1 to 2 cm away. What would be
your first step in management of this patient?
A. Sitz baths
B. Insertion of Word catheter
C. Antibiotics
D. Biopsy of the cyst and insertion of word catheter
E. Marsupialization of the cyst
63. A 68-year-old postmenopausal G4 P40O4 is seen in your clinic for her annual exam, She
complains of occasional urinary incontinence with sneezing Or coughing, and also reports
heaviness in the lower abdomen and mild bulging from the vagina that is more
prominent at the end of the day. She has no fecal incontinence and no vaginal dryness.
Her PMH is significant for obesity, COPD, CHF, and poorly controlled diabetes. She has
smoked 1/2 PPD for the lasi 40 years. Three of her chidren weighed more than 9 Ibs at
birth. On pelvic examination, you note second-degree pelvic relaxation. What treatment
would you recommend for this patient??
A. Placement of 2 mesh transobturator sling for urethral support.
B. Vaginal hysterectomy and McCal Culdoplasty
C. Anterior colporrhaphy
D. Posteriar colporrhaphy
E. Placement of a pessary and encourage Kegal exercises
64. A 27-year-old G0P0 woman presents to your office with a history of amenorrhea. She has
a history ol regular menstrual cydes in high school as wed as while in college and medcal
school, when she a history of regular menstrual cycles in high school as well as while in
college and medical school when she began oral contraceptives for birth control. She
stopped her birth control pills about 7 months ago and her penod never resumed. She is
sexually active with a male partner and uses condoms for contraception. She has a
history of seasonal allergies, no prior surgeries, and no prior pregnancies. She has not
has a Pap smear in 5 years, but has no history of dysplasia and no prior known STDs.
When you ask her about her general diet and activity. she states that she is a vegetarian.

REVISED BY TEAM “BANG JAGO” NOV 2020 UNHAS Page 44


She is resident, so often eats on the run or Skips meals but does eat one to two meals
per day and has no prior history of an eating disorder. She exercises when she can, about
One io two times per week. She typically jogs Or goes for a walk with her dog. Her BMI
is , elvated at 30. Which of the following tests are most appropriate to order at this visit?
A. TSH, prolactin, and β-hCG
B. 17-OH progesterone, DHEAS, and testosterone
C. Transvaginal ultrasound
D. Hysterosalpigogram to Iook for intrauterine adhesions .
E. All of the above

65. A 24-year-old G1P0 at 28 weeks 5 days of gestation presents to routine prenatal care
with complaint of increased discharge today. She first noticed it after going to the
bathroom. When she stood up she felt as if a little urine continued to leak out
Throughout the afternoon, she has continued to feel like water is leaking from the
vagina. There is no. vaginal bleeding or abdominal pain. The discharge is clear and
odorless Her pregnancy has been Otherwise uncomplicated. Which of the following is
the first step evaluating this patient?
A. Amnion dye tes/tampon test
B. Ultrasound to check for Amniotic Fluid index (AFI)
C. Sterile speculum examination
D. AmniSure test
E. Amniocentesis to rule out chorioamnionitis
66. A 32-year-old woman comes to your clinic for preconception counseling. She was
diagnosed with epilepsy at age 12 and is currently taking phenytoin and carbamazepine.
She has been saizure-free for 11/2 years. She and her husband are planning to conceive
within the next year What should you advise to decrease the risks for the upcoming
pregnancy?
A. Siopal setue medicatons
B. Optimize her seizure regimen to include only one medication

REVISED BY TEAM “BANG JAGO” NOV 2020 UNHAS Page 45


C. Start taking a prenatal vitamin and 400 mcg of folic acid
D. Keep the same dose of both medications and start taking 4 mg of folic acid
E. Recommend she transiton off both her Current medicatons and start taking valproic
acid for monotherapy
67. You are called to see a 21-year-old G3P2002 at 28-weeks gestational age who has had
limited prenatal care. She has diftuse complants of abdominal pain, cold sweats, anxiety,
and insomnia. Upon review of her history, she tells you that she usually has two to three
vodka drinks every day. For the past 3 days, she has not been able to afford to
purchaseany alcohol. She denies any other significant medical or surgical history. You
diagnose her with alcohol withdrawal and admit her to the hospital for treatment. What
if the most significant long-term complication of alcohol dependence during pregnancy?
A. Fetal alcohol syndrome
B. Maternal withdrawal
C. Neonatal withdrawal
D. Low fetal birth weight
E. Neonatal admission to the NICU
68. 45-year-old presents for evaluation because her primary care physician has diagnosed
her with pelvic organ prolapse while performing annual care. She denies any pelvic
pressure, bulge or difficulty with urination. Her only medical comorbidity is Obesity For
asymptomatic grade 1 pelvic organ prolapse, what do you recommend?
A. Conservative management with pelvic floor muscle exercises and weight loss
B. Colpocleisis obliterative procedure
C. Gellhom pessary
D. Round ligament suspension
E. hysterectomy
69. micturition is voluntary and occurs with relaxation of the urethra and sustained
contraction of the bladder until emptying is complete. Sustained contraction of the
detrusor muscle of the bladder requires parasympathetic stimulation. Parasympathetic
control of the detrusor is supplied by which of the following nerves?

REVISED BY TEAM “BANG JAGO” NOV 2020 UNHAS Page 46


A. Hypogastric nerve
B. Pudendal nerve
C. Peroneal nerve
D. Pelvic nerve
E. Sciatic nerve
70. A 52-year-old G3P3 comes in to see you for management of her hot flashes and night
sweats. She stopped havinf menses 1.5 Years ago and still has her ovaries and uterus in
situ. She has been suffering with recurrent hot flashes and night sweats, which interfere
with her qualify of life. She had hoped to avoid taking hormones but her symptoms have
not improved over the past 1.5 years and she is now ready for treatment options for her
hot flashes and night sweats, which of the following would not be an appropriate option?
A. Oral estrogen and progesterone
B. Topical estrogen and progesterone patch
C. Low dose vaginal estrogen cream
D. A selective serotonin reuptake inhibitor (SSRI) such as paroxetine (Paxil) or
fluoxetina A (Prozac)
E. Topical clonidine (catapres) patch
71. A 27-year-old G1P1001 woman comes to your office 8 months following an
uncomplicated vaginal delivery of a healthy male newborn. She and her husband have.
Been trying to conceive a second pregnancy fcr 3 months but she has been unsuccessful
and is very anxious about this. After a through history, you find that she is still
breastfeeding her newborn routinely an is not menstruating. Assuming that she has not
yet ovulated, what is the likely underlying cause of the findings in this patient?
A. Thyroid hormone suppressian of the anterior pituitary
B. Abnormal endometial regeneration Causing failure of implantation
C. Prolectin-induced inhibiton of pulsatile GnRH from the hypothalamus
D. Reduced tubal motility secondary to elevated prolactin levels
E. Pathologically decreased sperm count of the male partner

REVISED BY TEAM “BANG JAGO” NOV 2020 UNHAS Page 47


72. A 28-year-old G2P1 woman at 30 weeks of gestational age presents to the clinic for a
routine OB visit She was diagnosed with gestational diabetes 4 weeks ago and was
started on a diabetic diet and exercise regimen. She adhered to the recommendations.
measurad her glucose levels four times a day, and recorded results for the past week.
Her average fasting glucose value is 84 mg/dl anf 1-hour postprandial values after all
three meals range from 135 to 165 mg/dl. What is the best next step in the management
of this patient
A. continue diabetic diet plus exercise
B. strat the patient on lispro and NPH in the morning and lispro at dinner
C. Start the pallent on Lispro in the morning, and lispro and NPH at dinner
D. Start the patient on metformin 500 mg PO daily
E. Start the patient on NPH in the morning and NPH at dinner
73. Laceration of abdominal wall vessets can increase blood loss and risk of postoperative
hematoma formation. The superficial epigastic, superficial circumflex iliac and superficial
external pudendal arteries all arise from which of the following?
A. Femoral artery
B. External iliac artery
C. Deep circumflex artery
D. Internal thoracic artery
74. Early treatment of hidradenitis suppurativa involves which of the following?
A. Topical corticosteroid ointment
B. Infliximab, a monoclonal antbody
C. Surgical excision of apocrine gland sinus tracts
D. Warm compresses, topical antiseptics and systematic antibiotisc
75. Which of the following is a painful, self limited mucosal lesion?
A. Vitiligo .
B. Aphthous ulcer
C. Pemphigus vulgaris
D. All of the above

REVISED BY TEAM “BANG JAGO” NOV 2020 UNHAS Page 48


76. By defnition, patients with severe Oligospermia have sperm counts less than which of the
follwing per milliliter of semen?
A. <5 million/mL
B. <15 million/mL
C. <25 million/mL
D. <35million/mL
77. Mrs. P, a 45-year-old pregnant woman at term collapses in front of the nursing staff
while waiting in the delivery room. She is unresponsive and has no pulse. The midwife
activates the emargency response System and begins chest compressions. A team of 2
doctors and 2 nurses arrive with the emergency equipment.
A. The first thing one should do is displace the uterus to the left while chest
compressions is ongoing
B. One should follow BLS guidelines and perform chest compressions on the sternum at
the inter-nipple Iine
C. One should do 2 minutes of CPR before considering defitrilation in this patient.
D. The doctors should transfer the patient to theatre for an urgent Caesarean delivery
E. Intubation should only be considered after retum of spontaneous circulation
78. A 28-year-old woman, with monochorionic twin pregnancy showed a discordant
crowlump length (CRL) at 12 weeks gestation, and a discordant amniotic fluid volume at
15 weeks with a deepest vertical pocket of 18 cm in the smaller twin and 6.3 cm in the
larger twin. Both bladders are visible. Which of the following is/are likely diagnosis?
A. Unequal placental sharing.
B. Twin-to-twin transfusion syndrome quintero stage 1
C. Complicated monochorionic twin pregnancy, suspected to develop either selective
fetal growth restriction or twin-twin transfusion syndrome (TTTS)
D. Velemantous cord insertion in one twin
E. Physiological variation
79. A women, 25-year-old, comes to your practice for preconception counseling, she plans to
married next week, and have a complaint of hypersecretive mucus from her genital,

REVISED BY TEAM “BANG JAGO” NOV 2020 UNHAS Page 49


different sex hormones have different effects on the cervicalmucus. which of the
following statements accurately describes the effect of estrogen
A. It decreases the water content of cervical mucus.
B. It decreases the palm-leaf crystallization pattern of mucus upon drying (feming).
C. It decreases formation of gilycoprotein channels, which favor sperm penetration
D. It increases cervical mucus stretchability (spinbarkeit)
E. It increases the amount of potassium Chloride in the cervical mucus

A 34-yeas-old G3P2A0 woman at 38 weeks and 6 days was admitted to labor and delivery unit
for active management of labor after it was determined that her membranes had ruptured and
she was dilated to 3 cm. Her cervix has been steadily dilating and now She is at 6 cm. She is very
uncomfortable and finds her contractions very painful.l Her partner is also very concerned that
needs pain relief.

80. You advise your patient that


A. Narotics are available, but should be reserved for closer to the time Of delivery
when her pain will be greatest
B. If she continues with natural childbirth and eventually needs a cesarean section she
will require general anesthesia
C. Spinal anesthesia is her best option because it gives a constant infusion of medicine
over a long period of time
D. She cannot have an epidural yet because she is not yet in the active phase labor
E. A variety of relaxation techniques can he incorporated into her labor in addition to
pain medication
81. With adequate pain control she dilates to 10 cm and second stage begins. Which of the
following is the correct order ol the cardinal movements of labor?
A. Internal rotation, engagement, descent, flexion, external rotation
B. Engagement, descent, internal rotation, flexion, external rotation
C. Internal rotation, descent, engagement, flexion, external rotation
D. Engagement, descent. flexion. internal rotation, external rotation
E. Engagement, descent, external rotabtion, flexion, internal rotation

REVISED BY TEAM “BANG JAGO” NOV 2020 UNHAS Page 50


82. An uncomplicated vaginal delivery typically includes which maneuver?
A. Perineal support to decrease perineal trauma
B. An episiotomy to hasten delivery
C. Vacuum extraction if the fetal station Is low
D. Forceps to aid maternal efforts
E. The McRoberts maneuver
83. Stage 3 begins following the delivery of the infant and typically which of the following?
A. Placental separation
B. Stopping oxytocin drips if they were used during stage 2
C. An abrupt increase in the size of intrauterine cavity
D. Uterine prolapse
E. A delay of 60 minutes before the placenta is delivered
84. Which of fhe following is the most common cause of first trimester pregnancy loss?
A. Uterine anomnalies
B. Incompeteni cervix .
C. Intrauterine infection
D. Fetal chromosomal abnormalities
E. Placenta adhesive
85. Which of the following NOT clinical indicator heart disease during pregnancy?
A. Cyanosis
B. clubbing of fingers
C. systolic murmur grade 2/6
D. diastolic murmur
E. cardiomegaly
86. fetal hyperinsulinemia in the second pregnancy is NOT associaled with which of the
following?
A. Macrosomia
B. Neonatal hypoglycemia .
C. Malemal hyperglycemia.

REVISED BY TEAM “BANG JAGO” NOV 2020 UNHAS Page 51


D. increased matemal weight
E. Increased chance of neonatal acidosis
87. Which ol the following statements regarding inirapartum management of HIV is true?
A. If cesarean delivery is planned, it should be schedulade at 36 weeks’ gestation
B. Cesarean delivery is recommended for women with a viral load > 1000 copies/mL
C. In labor, with a plan for vaginal delivery, amniotomy should be performed as soon as
possible to hasten delivery.
D. In labor, internal monitors should be placed because fetuses of HIV-intected women
are at inceased risk for distress.
E. No vertical transmission risk increased if there is PROM > 38 weeks
88. Increased serum free thyroxine levels in women with hidatidiform moles stem from
increaser in which of the following?
A. Maternal estrogen levels
B. Fetal thyroxine production
C. Maternal progesterone levels
D. Maternal beta human chorionic gonadotropin levels
E. Maternal estriol levels

A 33 year-old G0P0 woman comes to your office for her initial prenatal visit. She tested positive
with two home pregnancy tests and has been Experiencing breast tenderness and mild nausea
for a few weeks. She has a history ol regular menstrual periods occurring every 28 to 30 days.
this was a planned pregnancy and is the frst child for her and for her partner

89. Your patient was actively tracking her menstrual cycle and is certain that the first day of
her last menstrual period (LMP) was 12/2/11. Using Naegele rule, estimate her date of
delivery
A. 5/12/11
B. 2/11/11
C. 19/11/11
D. 19/12/11
E. 19/10/11

REVISED BY TEAM “BANG JAGO” NOV 2020 UNHAS Page 52


90. As her pregnancy continue you would expect her cardiac output to increase by which of
the gollowing mechanisms:
A. First an increase in stocke volume, then an increase in heart rate
B. A decrease in systemic, vascular resistence
C. Cardiac output would not change significantly until the thud trimester.
D. An increase in systemic vascutar resistance facilicated by elevated progesterone
levels
E. Increased heart rate alone
91. Which of the following is true regarding the Phystologic changes she might expect during
her pregnancy?
A. Gastric emptying and large bowel motility are increased in pregnancy
B. BUN and creatinine will decrease by 25% as a result of an increase in glomerular
filtration rate (GFR), which will be maintained until delivery
C. An overall decrease in the number of WBC and platelets
D. Nausea and vorniting that should be treated aggressively with antiemetics and
intravenous hydration
92. A 19-year-old G3 P1 patient with a history of a miscarriage in her last pregnancy presents
to the emergency department with some vaginal spotting. She reports that her last
menstrual period occurred 6 weeks earlier. She has had no_yaginal discharge other than
the spotting, no cramping, and no abdominal pain. Her physical examination reveals a
slightly enlarged uterus, no tendemess, and a Closed cervical os. A serum β-hCG level is
sent off and returns 346. A pelvic ultrasound shows no intrauterine pregnancy. A 2 cm
left ovarian cyst, and no free fluid. Your diagnosis of this patient is which of the
following?
A. Threatened abortion/rule out ectopic pregnancy
B. Ectopic pregnancy
C. Inevitable abortion
D. Missed abortion
E. Normal pregnancy

REVISED BY TEAM “BANG JAGO” NOV 2020 UNHAS Page 53


93. in the patient described above, when should a repeat β-hCG he drawn?
A. 24 hours
B. 48 hours
C. 72 hours
D. 1 week
E. It does need to be drawn again
94. A 25-year-old G1 P0 at 9 weeks’ gestation presents for her initial prenatal visit. She has
no medical pr surgical history and has a certain LMP that is consistent with her
examination. The patient has donated blood In the past and knows that she is Rh
negative. In which of the following situations would the patient be cared for Without
being treated with RhoGAM?
A. First trimester bleeding .
B. second trimester bleeding
C. Routinely at beginning the tthird trimester
D. Contractions at 34 weeks gestation
E. At the time of an amniocentesis

A woman 45 years old with 2 children came to the specialist clinic in the international hospital
with the complain of post coital bleeding since 3 months ago. The lady had the first
menstruation at the age of 13 years old and got married at the age 17 years aid. From the
vaginal examination, 0.5 cm mass was seen ini the upperlip of the vagina without involvement
of the vaginal mucosa.

95. What is the most likely inital procedure has to be taken after seeing the of condition c
the cervix
A. VIA
B. biopsy
C. Leep
D. Pap smear
E. Endocervical curettage wa
96. What physical diagnostic examination is important in that situation?

REVISED BY TEAM “BANG JAGO” NOV 2020 UNHAS Page 54


A. Palpation of the cervical mass
B. Rectal examination
C. Rectovaginal examination
D. Palpation of the vagina
E. Vaginal examination
97. What is the most likely stage of the cases?
A. Stage la 1?
98. Which of the following is the characteristic histologic renal lesion of preeclampsia cases?
A. Giomerutar endothelosis
B. Interstitial fibrosis
C. Crescent formation
D. Renal cortical necrosis
E. Glomenular sclerosis
99. Which of the following tests would be the most effective in identifying cause of recurrent
miscarrgiage?
A. Antithrombin III assay ,
B. Serum progesterone level
C. Lupus-antieoagulant assay
D. Luteinizing hormone assay’
E. Serum estrogen level
100.A 29-year-old G3 P1 presents complaining of no menses for 4 months after stopping her
birth control pills. She is concemed that the use of the oral birth control pais (OCPs) has
left her with amenomhea. The patient has had no recent changes in weight, exercises
two to three times a week. and notes no particuler changes in either her work or home
life. Her obstetric history includes a therapeutic abortion at age 21, a normal
spontaneous delivery at age 25, and a miscarriage at age 27. Aftaer the dilation and
curettage at the time miscarriage, the patient was hospitalized with an infection of her
uterus sine that time, she has taken OCPs given this history, which of the following is the
most likely etiology of this patient’s amenorrhea?

REVISED BY TEAM “BANG JAGO” NOV 2020 UNHAS Page 55


A. Vaginal agenesis
B. Asherman’s syndrome
C. Mayer-Rokitansky-Kister-Hauser Syndrome
D. Testicular feminization
E. Hypogonadotropic hypogonadism
101.A 65-year-old G2p2002 presents to the urogynecology clinic with complaints of urinary
incontinence. She Jeaks urine occasionally when she coughs, but also leaks urine without
any provocation. She often has difficulty making it to the restroom. In time. She has even
leaked urine shortly after having normal .emptying of her bladder, She normally gets up
at least 2 fo 3 times pernight to urinate. Urinalysis and urine culture performed last week
at her PCP’s office are both negative. What is the most likely . diagnosis and appropiate
treatment for this type of incontinence?
A. Stress urinary incontinence, Ditropan (oxybutynin chloride)
B. Detrusor overactivity Detrol (tolterodine)
C. Overflow incontinence, cholinergic agent
D. Detrusor overactivity, suburethral sling
E. Stress urinary inocntinence, suburethral sling
102.An 18-year-old G0 presents to your clinic with primary amenorrhea. She has nomal
breast developmentof, but has limited development of pubic or axillary hair. On pelvic
examinabon, you note that she has a foreshortened vagina. Transvaginal and
transabdominal ultrasound is unable to identify a uterus. Chromosomal analysis show
that the patient is 46. XY. What is this patient’s diagnosis?
A. Testicular feminization
B. Gonadal agenesis

REVISED BY TEAM “BANG JAGO” NOV 2020 UNHAS Page 56


REVISED BY TEAM “BANG JAGO” NOV 2020 UNHAS Page 57
SOAL UNAS MARET 2017

21. You are taking care for a pregna nt woman who has complaint of cough
for a month , intermittent fever and night sweat, You are concerned
about TB. Which of the following statements is most accurate
regarding TB in pregnancy?

a. All woman with those symptoms should have a chest x ray in pregnancy

b. If a patient is treated adequately during pregnancy,TB generally has no


deleterious effect in mother or child

c. Mantoux test have unusually high false positive rate in pregna ncy

d. The best single drug for therapy of TB In pregnancy is streptomicin

e. 2 drug anti TB therapy is pregnancy is contraindicated

A 21 years old at 37 weeks of gestasional age was referred to the hospital


due to profuse vaginal bleeding. Previously patient felt severe abdomi nal pain
around 2 hours ago.On physical examination patient was unstable and indelirium
stage.Her Initial BP was 130/80 mmHg and pulse rate was 115 beat/minute. Her
abdomen feels rigid

24. What is the best next step?

a. Obstetrical exami nation consist of sterile speculum followed by vaginal examination

b. Abdominal ultrasound

c. Tra nsvaginal ultrasound

d. Stabilized patient ,obtain two large bore IV and start fluid bolus with 30 cc/kgBB

REVISED BY TEAM “BANG JAGO” NOV 2020 UNHAS Page 58


e. Emergency SC

25. What is the most important laboratory test to do in this patient?

a. ·CBC

b. Urianalysis

c. PT, apt

d. AST/ALT

e. Blood type and cross match

26.These factors are precipitating or predisposing actors of Placenta abruption, EXCEPT?

a. Hypertension

b. Nullipara

c. Previous placental abruptlon

d. Abdominal trauma

e. Short umbilical cord

REVISED BY TEAM “BANG JAGO” NOV 2020 UNHAS Page 59


A 33 year old woman G1P0A0, came to hospital with major complaint, watery
leakage. She was on her 33 weeks of gestational age. Data from medical record
showed that she came previously a week ago, complaining vaginal discharge.
Vaginal swab has done.

38. In case above, what kind of examination should you performed for
establishing diagnosis.

a. Vaginal examination

b. Inspeculo

c. Blood test

d. Ultrasound

e. Simple urine test

39. You found leopold 1, hard, round with ballottement ( + ). Contraction was
infrequent and weak. What was your next plan

a. Went for labour induction

b. Immediate c-section

REVISED BY TEAM “BANG JAGO” NOV 2020 UNHAS Page 60


c. Tocolytic and corticosteroid provision

d. Performed ultrasound

e. Performed external version

Woman refer from primary health care due considerably low maternal body
weight. She is on her 35 weeks of gestation, height 150 cm weight 32 kg. She had
previous history of severe hyperemesis. Other medical problems were denied.

40. Ultrasound examination showed that trans-cerebellar diameter was proper to


gestational age, abdominal circumference was lower than 2.5 centile and
amniotic fluid deepest pocket was 1.2 cm

What is the most likely diagnosis?

a. Growth restriction with olygohydramnion

b. Normal growth with olygohydramnion

c. Growth restriction with normal amniotic fluid

d. Normal growth with normal amniotic fluid

e. Need another examination for establishing diagnosis

41. Lack of baby movement had been felt for two days,fetal hearth rate was 146
bpm. What was your next step?

a. Termination of pregnancy

b. Giving oxygenation and left lateral position

c. Ensuring fetal well-being by Manning Criteria

REVISED BY TEAM “BANG JAGO” NOV 2020 UNHAS Page 61


d. Fetal lung maturation

e. Giving intravenous fluid rehydration

42. Cardiotocography, showed low variability with checkmark pattern and no


deceleration. what was interpretation and the best manangement throuhg ?

a. Category one, continued for fetal lung maturation

b. Category two, intrauterin resusitation for 24 hours and reevaluation after

c. Category two, when for droppler velocymetry

d. Category three, went for doppler velocimetry, ultrasound exam

e. Category three, delivered the baby

A 24 year old G2P1 woment at 39 weeks and 3 days is seen in clinic. She has been
experiencing more frequent contractions and thinks she might be in labour. Her
last pregnancy ended with a caesarean delivery after stage 1 arrest. There was
no evidence of cephalopelvic disproportion. Earlier in the course of her current
pregnancy she had desired a scheduled repeat caesarian , but now she might be
in labour she would like to try and delivery vaginally.

43. what would be a contraindication to a trial of labour after caesarian


( TOLAC) ?

a. prior classical hysterotomy

b. perior kerr hysterotomy

c. small for gestational age fetus

d. olygohydroamnions

REVISED BY TEAM “BANG JAGO” NOV 2020 UNHAS Page 62


e. GBS + mother.

44. A 33 years old G8P6A1 woman with fundal height of 39 cm presents for
initial prenatal visit in your clinic. she is currently an inmate at a local jail. she is
imprisoned on charges ofn marijuana possesion. she has had no prenatal care.
she is unsure of her last period but reports that started feeling the baby
apporixmately 5 month ago. she danies vaginal bledding, leaking fluid or
contractions during the pregnancy. ultrasound performed in your clinic shows
gestational age to be approximately 40 week 3 day. what is the most common
cause of diagnosis of postterm pregnancy

a. Innacurate dating

b. Fetal anomaly

c. Delayed presentation to parenatal care

d. Advanced maternal age

e. multyparity

46.A 29 year old G3P2A0 presents to the emergency center with complains of
abdominal discomfort for 2 weeks her vital sign are BP:120/70mmHg Pulse:90
BPM, temperature 36.9,RR:18x/i. A pregnancy test is positive and an ultrasound
of the abdomen and pelvis reveals a viable 16 week gestation located behind a
normal appearing 10x6x5.5cm uterus. Both ovaries appear normal.No free fluid
is noted.Which of the following is the most likely cause of these findings?

a. Ectopic ovarian tissue

b. Fistula between the peritoneum and uterine cavity

REVISED BY TEAM “BANG JAGO” NOV 2020 UNHAS Page 63


c. primary peritoneal implantation of the fertilized ovum

d. Tubal abortion

e. Uterine rupture

47. Obesity in pregnancy increase risk of which of the following in the baby

a. An autism spectrum disorder

b. Intra-uterine growth restriction

c. Hydrocephaly

d. Type 1 diabetes in childhood

e. Childhood cancer

48. The stage of gestational development at which endometrial implantation


occur is

a. Eigh cell embryo

b. Zygote

c. Morula formation

d. Blastocyst

e. Embryonic disk

49. During early pregnancy,whuich of the following replaces the ovary as the
main source of the progesterone production

a. Amnion

REVISED BY TEAM “BANG JAGO” NOV 2020 UNHAS Page 64


b. Chorion

c. Throphoblast

d. Myometrium

e. Yolc sack

50. Anti D immunoglobulin should not be given for Rh-negative woman in which
of the following settings?

a. Threatened abortion

b. Following complete hydatiform mole evacuation

c. Before external cephalic version

d. After first trimester elective pregnancy

e. Following complete abortion

51. To preserve the pneumoperitoneum when placing secondary trocars, it is


important to:
a. Ensure the surgeon is in proper position
b. Close the stopcock on the secondary port before insertion
c. Ensure the stopcock on the secondary port is open before insertion
d. Ensure the blade on the trocar is sharp
e. Close the insufflation stopcock on the primary port
52. Clinical feature suggestive oh hyperandrogenism include all of the following,
except:
a. Facial acne
b. Seborrhea
c. Alopecia

REVISED BY TEAM “BANG JAGO” NOV 2020 UNHAS Page 65


d. Anovulation
e. Hirsutism
53. Invasive cervical cancer identified only microscopically, invasion is limited to
measured stromal invasion with a maximum depth of 3mm and no wider than
7mm should be treated with:
a. Observation
b. Tissue ablation
c. LEEP/LETZ
d. Simple total hysterectomy
e. Radical hysterectomy
54. The majority of vulvar, vaginal and cervical cancer appear to have a common
cause and usually caused by:
a. High risk of herpes simplex virus (HSV) infection
b. High risk of human papilloma virus (HPV) infection
c. Increased exposure to endogenous estrogen
d. Increased exposure to exogenous estrogen
e. Chronic bacterial and parasitic infection
55. Childhood neoplastic ovarian mass most commonly originate from:
a. Gonadal epithelium
b. Gonadal stroma
c. Sex cords
d. Germ cells
e. Metastatic disease
56. A patient returns for postoperative checkup 2 weeks after a total abdominal
hysterectomy for fibroids. She distressed because she is having continuous
leakage of urine from the vagina. The leakage is essentially continuous and
worsens with coughing, laughing or movement. Given her history and physical,
you perform both a methylene blue dye test, which is negative and
indigocarmine test, which is positive. The most likely diagnosis is:

REVISED BY TEAM “BANG JAGO” NOV 2020 UNHAS Page 66


a. Rectovaginal fistula
b. Uretrovaginal fistula
c. Vesicovaginal fistula
d. Ureterovaginal fistula
e. Impossible to distinguish
57. A 38 years old multigravida woman complains of the painless loss of urine,
beginning immediately with coughing, laughing, lifting or straining. Immediate
cessation of the activity stops the urine loss after only a few drops. This history
os most suggestive of:
a. Fistula
b. Stress incontinence
c. Urge incontinence
d. Urethral diverticulum
e. UTI

A 40 years old G2P1A0 gestational age of 12 weeks. The first child 10 years of
age. Never use contraception. Weight 80 kg, height 156 cm. has never antenatal
care

58. The conclusion below is not true:


a. High risk pregnancy due to maternal age of 40 years
b. Including overweight
c. Have history of secondary infertility
d. There is possibility of pregnancy termination by cesarean section
e. Need for ultrasound screening
59. Select the incorrect regarding guidelines for nuchal translucency (NT)
measurement is:
a. The margin of NT edges must be clear enough for proper caliper placement and
the fetus is not necessary in the midsagital plane
b. The image must be magnified so that it is filled fetal head, neck and upper thorax
c. The fetal neck must be in neutral position, not flexed and not hyperextended

REVISED BY TEAM “BANG JAGO” NOV 2020 UNHAS Page 67


d. The amnion must be seen as separate from the NT line and the + calipers must
be place on the inner borders of the nuchal space with none of the horizontal
crossbar itself protruding into the space
e. The calipers must be placed perpendicular to the long axis of the fetus and the
measurement must be obtained at the widest space og the NT
60. Choose the wrong statement about Down syndrome is
a. An increased NT thickness itself is not a fetal abnormality
b. Nuchal translucency is a marker that confers increased risk of fetal abnormality
c. Approximately one third of fetus with increased nuchal translucency thickness
will have a chromosome abnormality, nearly half of which are down syndrome
d. Down syndrome is caused by an autosomal trisomy, is the most common
nonlethal trisomy
e. All cases of down syndrome is caused by trisomy 21

One mother after normal delivery, P4 age 35 height 160 cm , having delivery one
day ago, 3000 grams birth weight, breastfeeding, want injectable contraceptive
before returning home.
61. If in the case mentioned above, the mother wanted contraception progesterone-
only contraceptive (POCs), is the correct statement below is :
a. Breastfeeding women who are <6 weeks postpartum should not use
levonogestrel (LNG) and etonogestrel (ETG) implants
b. Breastfeeding women who are <6 weeks postpartum generally should not use
progesterone-only injectable (DMPA)
c. There is no theorical concern about the potential exposure of the neonate to
DMPA/NET-EN during the first 6 weeks postpartum
d. Breastfeeding women who are <48 hours postpartum can generally use LNG-
IUDs

REVISED BY TEAM “BANG JAGO” NOV 2020 UNHAS Page 68


e. progesterone-only contraceptives (POCs) consist of pops, progestogen-
only injectables (DMPA and NET-EN) and LNG and ETG implants.
62. In Indonesia, pregnancy related morbidity and mortality risk are high,
and access to services is limited, the correct statement below is :
a. DMPA maybe one of the few types of methods widely available and
accessible to breastfeeding woman immediately postpartum
b. In Indonesia DMPA injections should not be given one day after the
delivery as early postpartum contraception
c. Breastfeeding Women who are 6 weeks to < 6 months postpartum can
use without restriction the following contraceptive methods :
progestogen-only injectables (DMPA), and LNG and ETG implants
d. Breastfeeding (and non-breastfeeding) women are generally should not
have an LNG-IUD inserted from 48 hours to <4 weeks postpartum.
e. Available data from clinical and observational trials do not suggest an
increased risk for breastfeeding performance or infant health outcomes
with use of progestogen-only injectables compared to outcomes in
studies using other progestogen-only methods.
A 27 years old P3A2 return to your clinic to follow up the result of her
annual PAP-SMEAR. She has had a full annual examination including Pap
Smear since the age of 21 and has never had abnormal cervical cytology.
Her STD screening at her last examination was normal. The cytology of
her pap smear showed high-grade squamous intraepithelial lesion
(HGSIL). You discuss the management option for the patient, and she
decides to have colposcopy. The biopsy result of the colposcopy you
perform are not insufficient to give a histology report.
63. Which Of the following option is the next best step in management for
this patient?
a. Repeat Pap smear at 6-month interval for a year
b. Repeat colposcopy to obtain better sample

REVISED BY TEAM “BANG JAGO” NOV 2020 UNHAS Page 69


c. Perform diagnostic excisional procedure such as look electrical excision
procedure (LEEP)
d. Request that the lab repeats their test on the biopsy samples given
e. Conization
64. A 32 years old present for an infertility workup. She and her partner have
been trying to conceive for 2 years without success. She has regular
menstruation, though she mention she has severe cramping during
cycles. She also notes she experiences pelvic pain during sex. On
examination, she is a thin, well development women. She is afebrile and
she experiences a great deal of pain during pelvic examination. Which of
the following test is required for diagnosis of the patients infertility?
a. Ultrasound
b. b-hcg level
c. Pap smear
d. Laparoscopy
e. Hysterosalpingogram
65. A 29 years old G2P1A0 woman is 22 weeks pregnant with reports of
pelvic bulge and bothersome pressure. Physical examination reveal
cervical prolapse, approaching the opening of vaginal introitus. She
worried that the prolapse will worsen as the pregnancy progress. What
can you offer this patient for her symptomatic prolapse?
a. Pessary trial
b. Reassessment after delivery but nothing during the pregnancy
c. Cerclage
d. Hysteropexy
e. Uterosacral ligament fixation
66. Mrs. XY is a primigravida who is 34 weeks pregnant. Her last two serial scan have

shown a small gestational age fetus growing on the 9 th centile. Her last scan

REVISED BY TEAM “BANG JAGO” NOV 2020 UNHAS Page 70


shows positive end diastolic flow with a normal PI. She reports having good fetal
movements. How should further fetal surveillance be undertaken?
a. Forthnightly umbilical artery Doppler
b. Twice weekly CTG
c. Twice weekly umbilical artery Doppler
d. Weekly CTG
e. Weekly umbilical artery Doppler + CTG
A 25 years old woman in her first pregnancy is noted to have prolonged first and
second stage of labor. She was induced at 38 weeks pregnancy. The baby was
delivered by forceps. After delivery the placenta she is noted to have heavy
vaginal bleeding. Abdominal examination demonstrated a relaxed uterus.
67. What is the most likely cause of bleeding?
a. Uterine atony
b. Uterine rupture
c. Retained placenta
d. Genital tract laceration
e. DIC
68. What should we do if the fundus not firm after placenta delivery?
a. Methylergonevine (Methergine)
b. Carboprost (Hemabate, PF 2- alpha)
c. Fundal massage
d. Misoprostol (PGE1)
e. Dinoprostone-prostaglandine E2
69. Which of the following medications would be contraindicated in the treatment
for this patient?
a. Methylergonevine (Methergine)
b. Carboprost (Hemabate, PF 2- alpha)
c. Intramuscular pitocin
d. Misoprostol (PGE1)

REVISED BY TEAM “BANG JAGO” NOV 2020 UNHAS Page 71


e. Dinoprostone-prostaglandine E2

30-year-old multiparous woman has rapid delivery soon after arriving in emergency
room. After delivery the placenta she is noted to have heavy vaginal bleeding.
Help has been summoned, abdominal examination demonstrate the fundus was
soft.
70. What is the most appropriate next step?
a. Intravenous access for fluid resuscitation
b. Uterine packing
c. Suture the laceration
d. Balloon tamponade
e. Misoprostol administration

71. What is the most likely cause of bleeding?

a. Uterine atony
b. Uterine rupture
c. Retained placenta
d. Genital tract laceration
e. DIC

71. After use of 20 IU oxytocin in 1000 ml of crystalloid solution to increase the tone
of her uterus stop the bleeding, however, you continue to notice massive
bleeding from the vagina. What is the most appropriate next steep in the
evaluation of this patient bleeding?
a. Perform a bedside ultrasound for retained products of conception
b. Perform a bedside ultrasound to look for blood in the abdomen significant for
uterine rupture
c. Perform a manual exploration of the uterine fundus and exploration for retained
clots or products

REVISED BY TEAM “BANG JAGO” NOV 2020 UNHAS Page 72


d. Examine the perineum and vaginal for laceration during delivery
e. Consult interventional radiology for uterine artery embolization

A 31 years old woman came to the hospital with chief complaint of water broke.
She is G3P2 at 38 weeks gestation with one prior caesarean delivery on second
baby due to fetal distress. She wats to try trial labor. On psychical examination,
you find 3 cm cervical dilatation, clear amniotic fluid leakage with inadequate
contraction and other examination reveal no contraindication. You will start the
oxytocin infusion and you inform to the patient about the risk ?

73. According to the large study conducted bt The Maternal Fetal Medicine Units
Networks, the use of oxytocin increase the risk for uterine rupture by what
magnitudes in women with a prior caesarean delivery ?
a. Threefold
b. Sixfold
c. Tenfold
d. Sixteenth fold
e. No difference with normal delivery

74. Labor induction and augmentation are NOT associated with which of the
following risk?
a. Postpartum hemorrhage form uterine atony is more common in women
undergoing induction or augmentation
b. Amniotic fluid embolism in a laboring patient receiving oxytocin can be occurred
c. The increased risk for caesarean delivery undergoing induction is related with
cervical favorability (Bishop Score)
d. The uterine rupture risk is increased threefold for women in spontaneous labor
with uterine scar

REVISED BY TEAM “BANG JAGO” NOV 2020 UNHAS Page 73


e. Woman whose labor is managed with amniotomy have lower incidence of
chorioamnionitis compared with those in spontaneous labour

75. According to the patient above, what is the most favorable condition for
successful trial labour after caesarean birth?
a. Water broke with clear amniotic fluid
b. A previous vaginal delivery
c. In a hospital with available facility of anesthesia
d. Only one previous caesarean delivery
e. Induction given at term pregnancy

A 32 years woman comes to your clinic due to shortness of breath, that worsen
since 2 days ago on history taking she told you that sge had ever diagnosed of
having significant mitral stenosis. She is 33 weeks pregnant. The fetus size
appropriate. She has had echocardiography showing ejection fraction of 54%
with moderate severe pulmonary hypertension

76. What is the best management for this patient currently ?


a. Performed emergency C-Section
b. Lung Maturation
c. Conservative management until term pregnancy
d. Second stage acceleration
e. Induction of labour

77. What is the most common cause of heart failure during pregnancy and the
puerperium ?
a. Chronic Hypertension with severe preeclampsia
b. Viral myocarditis
c. Obesity

REVISED BY TEAM “BANG JAGO” NOV 2020 UNHAS Page 74


d. Valvular heart disease
e. Pulmonary artery hypertension

78. For the patients with congenital heart disease, what is the most common
adverse cardiovascular event encountered in pregnancy ?
a. Chronic hypertension with severe preeclampsia
b. Arrythmyas
c. Obesity
d. Valvular heart disease
e. Heart axis changes

A 18 years old G1P0 woman at 10 weeks GA presents for an initial prenatal visit.
From history taking she had a history of IV drugs use. You ordered a rapid test
and showed reactive for HIV. She had never been on HAART medication prior to
this pregnancy. You order viral load and it returns at 10.000. Her CD4 count is
normal (500). She is otherwise healthy and has no other significant medical
history

79. When is transmission of HIV from a woman to her baby most likely to occur?
a. Intrapartum
b. Before 36 weeks gestation
c. In the days before delivery

d. Transmission occurs equally throughout gestation


e. In the first semester

80. When would you recommend she start HAART ?

a. At this visit
b. At 37 weeks in preparation for delivery

REVISED BY TEAM “BANG JAGO” NOV 2020 UNHAS Page 75


c. At the beginning of the second semester
d. Immediately after medication
e. Only in it is medically indicated for maternal health

81. Which of the following statements about the treatment of HIV pregnancy is
true?

a. Treatment is recommended for All HIV infected pregnant woman


b. All medications used for the treatmen HIV are safe in pregnancy
c. Treatmen is only required if the HIV infected woman would qualify for treatment
when not pregnant
d. Zidovudine must be added to whatever regimen the woman is already taking,
even if her viral load is adequately suppressed
e. Treatment is recommended only for woman with high viral load

82. which of the following statements regarding intrapartum management HIV is


true?

a. If cesarean delivery is planned, it should be scheduled at 36 weeks gestation


b. Cesarean delivery is recommended for woman with a viral load > 1000 copies/Ml
c. In labor, with a plan by vaginal delivery, amniotomy should be performed as soon
as possible to hasten delivery
d. In labor, internal monitors should be placed because fetuses of HIV- infected
woman are at increased risk for distress
e. The HAART should be suspended during labor

83. A 55 year old woman presents to your office for consultation regarding her
symptoms of menopause. She stopped having periods 8 months ago and is
having severe hot flushes. The hot flushes are causing her considerable stress .
what should you tell her regarding the psychological symptoms of climacteric?

a. They are not related to her changing levels of estrogen and progesterone

REVISED BY TEAM “BANG JAGO” NOV 2020 UNHAS Page 76


b. They commonly include insomnia, irritability, frustration, and malaise
c. They are related to a drop in gonadotropin levels
d. They are not affected by environmental factors
e. They are primarily a reaction to the cessation of menstrual flow

84. a 62 year old woman presents for annual examination. Her last spontaneous
menstrual period was 9years ago and she has been reluctant to use post
menopausal hormone replacement because of a strong family history of breast
cancer . she now complain of diminished interest in sexual activity. Which of the
following is the most likely cause of her complaint?

a. Decreased vaginal length


b. Decrease ovarian function
c. Alienation from her partner
d. Untreatable sexual disfunction
e. Physiologic anorgasmia

85. A 58 year old post menopause, Caucasian woman comes in to your office for
advice regarding her risk factors for developing osteoporosis. She is 5 ft 1 in tall
and weighs 195 lb. she stopped having periods at age 49. She is healthy but
smokes one pack of cigarettes a day. She does not take any medications. She has
never taken hormone replacement for menopause . her mother died at age 71
after she suffered a spontaneous hisp fracture. Which of the following will have
the least effect on this patient’s risk for developing osteoporosis?

a. Her family history


b. Her race
c. Her history of smoking cigarettes
d. Her menopause status
e. Her obesity

REVISED BY TEAM “BANG JAGO” NOV 2020 UNHAS Page 77


A 49 year old women experiences irregular vaginal bleeding for 3 months
duration. You performed endometrial biopsy, which copious tissue with a
delivery, lobulated texture. The pathologist report shows proliferation of
glandular and stromal elements with dilated endometrial glands, consistent with
simple hyperplasia. Cytologic atypia absent

86. which of the following is the best way to advised the patient?

a. She should be treated to estrogen and progestin hormone therapy


b. The tissue will progress to cancer in approximately 10 % cases
c. The tissue may be weakly premalignant and progress to cancer in approximately
of 1% cases
d. She requires a hysterectomy
e. No further therapy is needen

87. which of the following factors is a protective against endometrial


hyperplasia?

a. Obesity
b. Tamoxifen
c. Oral contraceptive pills (OCPs)
d. Early menarche or late menopause
e. Unopposed exogenous estrogen therapy

You see a 16 yo female who presented with primary amenorrhea. Breast


development was noted at 13 years but there has been no increase in breast
size. Pubic and axillary hair were noted within one year of referral. An outside
ultrasound showed no uteru no ovaries. Physical exam reveals a normal vagina
introitus with hymen present. Breast are tanner 3, but seem to be more fatty
than mammary tissue.

88. lab test were significant for absent estradiol, elevated gonadotropins and
midly elevated DHEAS with normal testosterone. A karyotype was requested by

REVISED BY TEAM “BANG JAGO” NOV 2020 UNHAS Page 78


the endocrinologist and was found to be 46 XY, (SRY gene). What is the working
diagnosis following her initial work up?

a. Androgen insensitivity
b. Disorders of testicular development
c. Mullerian agenesis
d. A dan B
e. B dan C

89. The MRI showed an infantile uterus with no discernible gonads. During a
laparascopic evaluation, two dysplastic gonads attached to small fallopian tubes
and rudimentary uterus was visualized. Yhe final diagnosis is:

a. Swyer syndrome
b. Androgen insensitivity
c. Mullerian agenesis
d. Partial gonadal dysgenesis
e. Testicular regression syndrome

90. what type of tumor is a significant risk for such condition?

a. Mature teratoma
b. Leydig cell tumor
c. Germ cell tumors
d. Stroma cell tumors
e. Granulose cell tumors
Miss 25 years old P3 comes to Gynecolog outpatientclinic with cytology result
low grade SIL. She curious about the result since her last cytology result was
normal 3 years ago. She has no complaint recently. She began sexually active
since 10 years ago and has had six partners. She smokes 10 cigarettes per day
since 4 years ago. Her mother was diagnosed for cervical cancer at 44 years old
and just died 3 months ago. Her child now is 6 years old.

REVISED BY TEAM “BANG JAGO” NOV 2020 UNHAS Page 79


91. What are patient risk factor for CIN ?
a. Her smoking habits
b. Onset of sexual activity
c. Six sex partners
d. Early childbearing
e. All mention above

92. What is next proper management for patient ?

a. IVA test
b. Colposcopy
c. No pocedur need in her treatment
d. HPV DNA test
e. Endocervical curettage

93. The result showing a condylomatous acetowhite lesion with punctuation and
atypical vessels. Biopsy result confirms CIN 1 and HPV DNA test positive. What do
you suggest for patient ?

a. LEEP procedure
b. Reevaluation of HPV DNA
c. Cold knife conization
d. Repeat cytology in 12 months
e. Repeat cytology in 6 months

Mrs. A 37 years old had just undergone laparoscopic procedures. Her chief
complaint was infertility for 6 years with history of severe dysmenorrhea. From
hyterosalpingography, both tubes were non patent. Pelvic ultrasound found
bilateral cystic mass with echo interna sized 50 and 60 mm in diameter. Her
husband sperm examination was within normal limit. From laparoscopy findings,
normal size uterus with adhesion of posterior part to colon and Douglas pouch

REVISED BY TEAM “BANG JAGO” NOV 2020 UNHAS Page 80


was also severely adhered. Both of ovaries were enlarged approximately 50-60
mm with severe adhesion at the right and left adnexa. After adhesiolysis, both
the tubes could be identified and were blocked. Chocolate fluid was spilled from
cysts. The pathology of the cysts was endometriosis cyst.

94. Which one is the TRUE statement ?

a. The patient should not undergo surgery because the best procedure to solve her
pain and infertility problem was giving GnRH agonist then performed IVF
b. Laparoscopic procedure should be done if infertility problem already more than 3
years
c. Laparoscopic cystectomy was done because the cyst have diameter 50 and 60
mm
d. GnRH agonist 1 time prior to surgery will give better results
e. None of the above

95. Which of the statement below IS NOT TRUE regarding the surgery ?

a. Bilateral cystectomy could be done without any precautions on ovarian reserved.


b. Patient has severe grade of endometriosis because its AFS score more than >40
c. Since she has not conceived yet we should conserve both of her tubes
d. A and C
e. B and C

A 19 years old woman came to clinic with chief complaint of irregular


menstruation. She had menstruation once every three months. She is also obese
(BMI 32). Physical examination reveals she has hirsutism (Ferriman Gallwey score
9), other physical examination with normal limit. Gynecologic examination within
normal limit.

96. Which of the following criteria diagnosis of polycystic ovarian syndrome (PCOS) is
not part of Rotterdam criteria ?

REVISED BY TEAM “BANG JAGO” NOV 2020 UNHAS Page 81


a. Oligo/anovulation
b. Appearance of polycystic ovaries by gynecologic ultrasound
c. Excess androgen activity
d. Ferriman Gallwey score >8
e. All statement is true

97. In PCOS, increased testosterone production from the following hormones ?

a. Inhibin
b. Estradiol
c. Prolactin
d. FSH
e. LH

A 55 years old nulliparous woman who underwent menopause at age 50 years


complaint of a 1 month history of vaginal bleeding and smells. Her medical
history reveals she has hypertension and controlled with anti hypertensive agent,
and also she has diabetes mellitus contolled with an oral hypoglycemic agent. On
examination, she weighs 89 kg and 152 cm tall. Her blood pressure is 150/90
mmHg. Heart and lung examination are normal. The abdomen is obese and no
masses are palpated. The external genitalia appear normal, and the uterus seems
to be enlarged, without adnexal masses palpated.

98. What is the probable diagnosis of this patient ?

a. Cervical cancer
b. Hyperplasia Endometrium
c. Endometrial carcinoma
d. Uterine Fibroid
e. Ovarian Cancer

99. Which of the following does not increase a woman’s risk of developing
endometrial cancer ?

REVISED BY TEAM “BANG JAGO” NOV 2020 UNHAS Page 82


a. Obesity
b. Smoking
c. Diabetes Mellitus
d. Tamoxifen
e. Unopposed estrogen

100. Which of following subtypes is NOT a type II endometrial cancer ?

a. Clear cell carcinoma


b. Papillary serous carcinoma
c. Carcinosarcoma
d. Endometrioid adenocarcinoma
e. None of the above

REVISED BY TEAM “BANG JAGO” NOV 2020 UNHAS Page 83


AGUSTUS 2017

A 28-year woman, G1 36 weeks of gestational age, went to your clinic to do routine


antenatal care. During ultrasound, the doctor told that she will be expecting baby boy
with estimated lethal Weight 2500 g, however, amniotic fluid considered to be less than
normal. Then you asked the patient to drink minimal of 2 L of water a day and get
herself another ultrasound within 3 days to evaluate the amniotic fluid
11. Oligohydramnios is defined as which of the following ?
A. Amniotic fluid index < 5 cm
B. Single deepest pocket < 2 cm
C. Amniotic fluid index < 90m percentile
D. All of the above
E. None of the above

12. Amniotic fluid volume is a balance between production and reception. What is
the primary mechanism of fluid reception ?
A. Fetal breathing
B. Fetal swallowing
C. Absorption across fetal skin

REVISED BY TEAM “BANG JAGO” NOV 2020 UNHAS Page 84


D. Absorption by fetal kidneys
E. Filtration by fetal kidneys

13. In a normal fetus at term, what is the daily volume of fetal urine that contributes
to the amount of amniotic fluid present ?
A. 200 mL
B. 250 mL
C. 500 mL
D. 750 mL
E. 1000 mL

Mrs. A, 26 years-old, G1P0A0, according to her last LMP is 34 weeks pregnant, came for
her first antenatal care. She admit to have 20 kg weight gain during pregnancy with
swelling ankles for the past 4 weeks. She never took any iron or vitamin
supplementation. From the physical findings, BP 145/95 mmHg, HR 86x/min, RR
20x/min, BMI 35 kg/m3 Ultrasound examination confirmed twins in breach
presentation. Results from urinalysis were as follows color cloudy yellow, specific gravity
1 ,013, albumin 2+, RBC 0 – 1, WBC 2-5, bacteria negative

14. What is the most likely diagnosis ?


A. Acute fatty necrosis of the liver
B. Chronic hypertension
C. Preeclampsia
D. Renal disease
E. Pyelonephritis

15. Given the history if this patient, several more laboratory and diagnostics tests
were obtained. She was stable and the fetuses have reassuring heart rate tracings.
Which of the following do you expect to see in the test results ?
A. Chest X-ray to show decreased pulmonary vascular markings

REVISED BY TEAM “BANG JAGO” NOV 2020 UNHAS Page 85


B. Urine to show infection
C. Creatinine clearance to be increased above normal pregnancy levels
D. Serum Uric acid to be increased
E. A decreased hematocrit

Mrs. B, 37 years-old came to your office at 32 weeks of gestation according to her last
menstrual period. She has no ultrasound examination before and didn’t get antenatal
care routinely. The vital sign is within normal limit. She has body mass index 19 kg/m2.
During physical examination the uterine fundal height is 22 cm. From ultrasound
examination, the fetus has biometric values that correlate with 30 weeks fetus.
16. Which of the following is the next best step in managing this patient ?
A. Antenatal care routinely for the next 2 weeks
B. Evaluate maternal status and comorbidities
C. Consider definer the baby
D. Repeat sonography for fetal growth in 2 weeks
E. Doppler velocimetry evaluation every 3 days

17. According to algorithm for management of fetal-growth restriction, you evaluate


the Doppler Velocimetry then find reversed and-diastolic flow and oligohydramnios.
What is the appropriate management at this time?
A. Regular fetal testing
B. Weekly evaluation of amniotic fluid
C. Consider corticosteroids for lung maturation
D. Deliver the baby
E. Reevaluate middle cerebral arteries and ductus venous

18. Fetal growth restriction is associated with all of the following, EXCEPT
A. Antiphospholipid antibody syndrome
B. Inherited thrombophilias

REVISED BY TEAM “BANG JAGO” NOV 2020 UNHAS Page 86


C. Infertility
D. Immunosuppressive drugs
E. Social deprivation

You see a patient in your office who had a thyroidectomy from Graves disease. She is
now hypothyroid on thyroid hormone replacement. You monitor her TSH and keep a
between 0.5 and 2,5. You increase her thyroid hormone replacement each trimester and
her TSH remains around 2.0 during the entire pregnancy
19. In addition to monitoring her TSH what other additional testing should you perform
during her pregnancy ?
A. Amniocentesis to determine at the fetus is affected by Graves Disease
B. A detailed fetal ultrasound at 18 to 20 weeks and again in the third trimester
should be performed given the increased risk of fetal goiter.
C. Fetal echocardiogram to evaluate for cardiac abnormalities
D. Umbilical Dapples to monitor for placental dysfunction
E. MCA Doppler’s to monitor for fetal anemia

20. Management of hypothyroidism in pregnancy is :


A. Women after thyroidectomy may require higher doses of levothyroxine
B. TSH levels measured weekly
C. Thyroxine dose is adjusted by 100 mgh increments until TSH values become
normal
D. Increased thyroxine requirements begin as early as 12 weeks
E. None above

Mrs. E, 12 yo referred from midwife with antepartum hemorrhage. She is G3P2 term
pregnancy On examination her blood pressure is 160/100 mmHg, HR 100 bpm. She
looks anemic , not icteric obstetrical examinations reveal contraction 4-5x/10 minutes
FHR 170 bpm, head presentation 3/5. After thorough examination it is conclude with
retro placenta hematoma size 6x5 cm. This patient planned to do cesarean section

REVISED BY TEAM “BANG JAGO” NOV 2020 UNHAS Page 87


21. It the patient above during cesarean section found to be in a atonic condition
which of the following are not a prerequisite for performing b-lynch suture ?
A. Patient in lithotomy position during operation
B. Bimanual compression reduce the amount of bleeding
C. Availability of suture material
D. Patient in stable hemodynamic condition
E. None of the above

22. If during operation the uterus is couvelaire but with good contraction, how
would you manage that condition ?
A. Perform prophylactic b-lynch suture
B. Ascending uterine artery ligation
C. Hypogastric artery ligation
D. Sub total hysterectomy
E. Uterotonic and observation

23. Postoperative period is very crucial in this patient Which of the following is not
included as a parameter needed to be evaluated in early warning system
A. Blood pressure
B. Heart rate
C. Urine production
D. Central venous pressure
E. All of the above

A 26-years-old woman, G1P0A0 was admitted to emergency room because she lost her
consciousness around 1 hour ago. According to her husband, she is 36 weeks pregnant
she performed antenatal care at scheduled time, and never missed one. Her husband
said, she never had any hypertension or any other disease before. Three days prior
hospitalization, she had severe nausea and vomiting. Physical examination reveals, BP
120/80 mmHg, pulse rate 87 x/min, RR 18x/min, Temperature 36,5 OC. You notice there

REVISED BY TEAM “BANG JAGO” NOV 2020 UNHAS Page 88


is an icteric sclera. Other physical examination was remarkable. Obstetrical examination
reveals no fetal heartbeat was detected. Laboratory examination reveals CBC
10.2/29.9/8900/263.000; Ur/Cr 18/0, 8 AST/ALT 458/878; RBG 32; Urinalysis was within
normal limit.

24. What is the best next management in this case ?


A. Abdominal ultrasound
B. Induction of labor
C. Emergency Caesarean section
D. Whole Blood transfusion
E. Injections of 40% Dextrose

25. All EXCEPT which of the following clinical characteristic that increase the risk for
acute fatty liver of pregnancy ?
A. Null parity
B. Female fetus
C. Male fetus
D. Twin gestation
E. Third trimester

26. What is the underlying pathophysiology or intrahepatic chelestasis of


pregnancy ?
A. Acute hepatocellular destruction
B. Incomplete clearance of bile acids
C. Microvascular thrombus accumulation
D. Eosinophil infiltration of the liver
E. Hepatocellular injury

REVISED BY TEAM “BANG JAGO” NOV 2020 UNHAS Page 89


A 17-year-old G1P0 woman presents at 25 weeks gestation complaining of headache for
the past 36 hours. She has had regular prenatal visits going back to her first prenatal
visit at 8 weeks gestation. A 20-week ultrasound related her pregnancy by 2 weeks as it
was 15 days earlier than her LMP dating. She has a BP of 155/104 mmHg.
27. You review her medical record and determine that she does not have chronic
hypertension. The patient denies having RUQ pain but because of your high suspicion of
severe preeclampsia you order a CBC, liver enzymes, renal function test and a 24 hour
urine protein collection. Her laboratory test results reveal a normal platelet count and
liver enzymes but a slightly elevated creatinine and proteinuria of 550 mg in 24 hours.
Her headache has resolved after a dose of acetaminophen. What is the next best step in
her management ?
A. Give her a prescription for labetatol and have her follow-up in clinic in 2 weeks
B. (a) plus bed rest
C. Hospitalization for further evaluation and treatment
D. Immediate delivery
E. Begin induction of labor

28. Over the next 12 hours, her SPBPs rise above 160 mmHg on several occasions,
most notably to 174/102 mmHg 2 hours after admission and to 168/96 mmHg 9 hours
after admission. Her headache does not return and she has no RUQ pain or visual
symptoms. A set of repeat laboratory test results are unchanged and by increasing her
labetalol dose to 400 mm TID, her BPs decrease to 140s-150s/70-90 mmHg. She is also
started on magnesium sulfate. What change in physical or laboratory examination do
you observe that would indicate delivery ?
A. Another BP of 174/102 mmHg
B. Headache returning
C. Double vision
D. Platelets of 108
E. AST of 265

A 43-year-old woman, G4P3, at 37 weeks gestation presented in hospital with a ten day
history of low extremities edema, with idiopathic hypertension for 1 year. At
presentation, she had a blood pressure of 170/100 mmHg. Laboratory findings were

REVISED BY TEAM “BANG JAGO” NOV 2020 UNHAS Page 90


normal except urinalysis (protein 2+). She was diagnosed with superimposed severe
preeclampsia. It was decided to deliver the fetus by means of a C-section by indication
transverse lie. Blood pressure measurement was 150/100 mmHg. She lost consciousness
for 30 seconds five hours after operation. The laboratory studies gave the following
result serum aspartate aminotransaminase (AST), 225 IU/L, serum alanine
aminotransaminase (ALT), 140 IU/L. serum urea and creatine were normal, hemoglobin,
10.6 mg/dL, platelet count, 50 x 103 /ml. A brain computed tomography (CT) scan was
performed on patient which revealed the left frontal lobe lacunar infarction. The patient
was transferred to intensive care unit.
29. What is the most appropriate diagnosis
A. DIC
B. Acute Fatty liver in Pregnancy
C. HELLP Syndrome
D. Severe puerperal infection
E. Thrombotic thrombositopenic purpura

30. What is the best management after for this case


A. Fresh-frozen plasma and trombocytes concentrates
B. Anti-platelets
C. Anti-oxidant
D. Corticosteroid
E. Magnesium sulfate

31. Twelve hours observation showed urine production was 100 ml


A. Immediately giving diuretics bolus-iv
B. Immediately giving diuretics maintained by syringe-pump
C. Check albumin level, giving diuretic justified after hipoalbuminemia condition
had bean distinguished
D. Renal failure due to micro thrombopathy suspected, heparin provision could be
considered

REVISED BY TEAM “BANG JAGO” NOV 2020 UNHAS Page 91


E. Immediately step on fluid rescucitation

A 33-year-old woman, G1P0A0, came to hospital with major complaint, watery leakage.
She was on her 33 weeks of gestational age. Data from medical record showed that she
came previously a week ago, complaining vaginal swab has done.
32. In case above, what kind of examination should you performed for establishing
diagnosis
A. Vaginal examination
B. Inspeculo
C. Blood test
D. Ultrasound
E. Simple urine test

33. You found on Leopold 1, hard, round with ballottement (+). Contraction was
infrequent and weak. A Shat was your next plan ?
A. Went for labour induction
B. Immediate C. Section
C. Tocolytic and corticosteroid prevision
D. Performed ultrasound
E. Performed external-version

34. Ultrasound examination showed that trans-cerebellar diameter was proper to


gestational age. Abdominal circumference was lower than 2.5 centile and amniotic fluid
deepest pocket was 1.2 cm. What is the most likely diagnosis?
A. Growth restriction with olygohydramnion
B. Normal Growth with olygohydramnion
C. Growth restriction with normal amniotic fluid
D. Normal growth with normal amniotic fluid
E. Need another examination for establishing diagnosis

REVISED BY TEAM “BANG JAGO” NOV 2020 UNHAS Page 92


35. Lack of baby movement had been felt for two days, lethal heart rate was 146
bpm. What was your next step ?
A. Termination of pregnancy
B. Giving oxygenation and left lateral position
C. Ensuring fetal welt-being by Manning criteria
D. Fetal lung maturation
E. Giving intravenous fluid rehydration

36. Cardiotocography, showed low variability with checkmark pattern and no


desceleration. What was your interpretation and the best management through ?
A. Category one continued for fetal lung maturation
B. Category two, intrauterine resuscitation for 24 hours and reevaluation after
C. Category two, went for Doppler velocymetry
D. Category three, went for Doppler ultrasound ultrasound exam
E. Category three, delivered the baby

37. A patient wishes to consider pregnancy after treatment for her breast cancer.
What is the most important predictor of a good prognosis ?
A. Young age
B. Herceptin positivity
C. Estrogen receptor positivity
D. BRCA gene positivity
E. Family history of treatable breast cancer

38. A primigravida at 36 weeks gestation is measuring large for dates. Ultrasound


shows AC > 97 th canticle GTT performed shows poorly controlled gestational diabetes.
What is the immediate management plan?
A. Give steroids

REVISED BY TEAM “BANG JAGO” NOV 2020 UNHAS Page 93


B. Start induction
C. Start hypoglycaemics
D. Wait and watch
E. Start sliding scale and deliver

39. A 36 year old G2P1 presents to the antenatal clinic. She had an emergency
caesarean section for sudden onset hypertension and placental abruption at 30 weeks in
her previous pregnancy. She is currently 20 weeks of gestation and enquires about
further plan of fetal monitoring in this pregnancy. What is the most appropriate advice?
A. No extra monitoring is required
B. Uterine artery Doppler at 22 weeks
C. Serial scans starting form 24 weeks
D. Serial cardiotocograph monitoring from 28 weeks
E. Serial scans from 28 weeks

40. A 34 year old pregnant lady, G2P1 has been diagnosed with ductal carcinoma of
the right breast (Stage 1). She is currently 22 weeks pregnant. What is the initial
treatment of choice for her ?
A. Termination of pregnancy
B. Local mastectomy with reconstruction
C. Local mastectomy without reconstruction
D. Local radiotherapy
E. Single close chemotherapy with frastuzumab

Ada yang sama d shef life tapi choisenya di putar

42. Women with one or more previous caesarean section scars and an anterior
placenta are at of placenta accrete. Which test has been shown in recent research to
provide the highest sensitivity and specificity for antenatal diagnosis of placenta
accrete?

REVISED BY TEAM “BANG JAGO” NOV 2020 UNHAS Page 94


A. Colour Doppler
B. 3D power Doppler
C. Contrast CT
D. Gadolinium contrast MRI
E. Grey scale ultrasound

43. A 22 year-old unhooked primigravida presents to the Emergency Department at


26 weeks of gestation with a history of spontaneous painless bleeding at about 500 ml.
What is the best investigation to clinch a diagnosis ?
A. MRI scan
B. Transabdominal scan
C. CTG
D. Transvaginal scan
E. CT-scan

44. A 41-year-old primigravida with DCDA twins following a successful IVF treatment
presents with a booking BMI of 42 at 13 weeks of pregnancy and is seen in the antenatal
clinic. What is the most appropriate medication that you would recommend ?
A. Folic acid 5 mg/day
B. Vitamin D 10 mcg/day
C. Aspirin 75 mg/dya
D. Metformin 500 mg TDS
E. Omega 3 fatly acid

45. A primigravida at 35 weeks of gestation presents with pain in the right


hypochondrium and right side of her back. There is no history of nausea or vomiting.
Hypertension, urinary symptoms and bowel problems. Vital signs: pulse-106,
temperature 38,1 BP 128/75. Abdominal examination is unremarkable. Chest is clear.
Fetal monitoring is normal. Urine shows 2+ leucocytes and 1+ blood. White cell count
16x10 lt. What is the most likely diagnosis ?

REVISED BY TEAM “BANG JAGO” NOV 2020 UNHAS Page 95


A. Appendicitis
B. Cholecystitis
C. Pyeloneghritis
D. Abruption
E. Right basal pneumonilis

46. A 35-year-old P2 is 36 weeks pregnant clinically there is a suspicion of left calf


DVT, CTG is normal. What is the next step in the immediate management ?
A. Plan delivery
B. Therapeutic dose of i tinzaparin
C. Prophylactic dose of tinzaparin
D. FBC, coagulation screen LFTs
E. Thrombophilia screen

47. A 24-year-old G2P1 woman at 39 weeks and 3 days is seen in clinic. She has been
experiencing more frequent contractions and thinks she miht be in labor. Her last
pregnancy ended with a caesarean delivery after a stage 1 arrest. There was no evidence
of cephalopelvic disproportion. Earlier in the course of her current pregnancy she had
desired a scheduled repeat cesarean, but now that she might be in labor she would like
to try and delivery vaginally.
What would be a contra indication to a trial of labor after cesarean (TOLAC) ?
A. Prior classical hysterotomy
B. Prior Kerr hysterotomy
C. Small for gestational age fetus
D. Oligohydraminios
E. GBS + righter

48. An 18-year-old G1 sees you in your office and tells you she missed her last period
and had a positive home urine pregnancy test. You perform your normal first-obstetrical
examination and obtain basic prenatal labs as well as a first trimester viability

REVISED BY TEAM “BANG JAGO” NOV 2020 UNHAS Page 96


ultrasound. The radiologist calls you to discuss the findings, which demonstrate a
gestational sac with a fetus measuring approximately 8 weeks, gestation There is no
fetal heartbeat demonstrated. The placenta is markedly thickened and echogenic more
than would be expected in the first trimester. There are multiple areas of small cystic
spaces within the placenta as well the most likely diagnosis and corresponding
karyotype is:
A. Complete Mole, 46 XX
B. Incomplete Mole, 46 XY
C. Incomplete Mole, 69 XXY
D. Complete Mole, 69 XXX
E. Incomplete Mole, 69 XYY

49. A 38-year-old woman presents to your emergency department with complaints


of irregular vaginal bleeding for the past 1 year. Upon evaluation of her surgical history,
she fells you she had a procedure outside of the country for an abnormal pregnancy
about 1 year ago. She has no follow-up since then. On review of systems. She tells you
she has been coughing un blood for the past 1 week. Physical examination reveals old
blood in the vaginal vault. You lab data are significant for a serum -hCG of 112.000 ml
U/ml. the most likely diagnosis is :
A. Complete molar pregnancy
B. Metastatic persistent GTD
C. Placental site trophoblastic tumor
D. Dysfunctional uterine bleeding
E. Partial molar pregnancy

50. A 21-year-old woman undergoes hystertoscopy an currtage for persistent


irregular uterine bleeding after her term vaginal delivery 8 months ago. Final pathologic
lindings form the D&C are consistent with chonocarcinoma. Baseline serum-hCG is
200.2000 ml uteri. When of the following is not currently indicated ?
A. Imaging for distant melastastic lesions
B. Surgical intervention (hysterectomy)
C. Chemotherapy

REVISED BY TEAM “BANG JAGO” NOV 2020 UNHAS Page 97


D. Clos surveillce of serum -hCG
E. Reliable contraception

51. A 25-year-old G1P0 presents to the emergency room with vaginal beeling. Her
last normal menstrual period was 6 weeks earlier. She reports that she is sexually active
with male partners and does not use any hormonal of burner methods for
contraception. On arrival her temperature is 37oC. blood pressure is 115/80 pulse is 75
beats per minute respiratory rate is 15 breaths per minute, and she has 100%oxygen
saturation on room air. A pelvic examination reveals a small amount of dark blood in the
vagina. The external cervical as appears 1 to 2 cm dilated. Her uterus is mildly enlarged,
anteverted and nontender. A pelvic ultrasound is obtained and shows an intrauterine
gestational sac with a yolk sac. No fetal pole or cardiac motion is seen. Bilateral adnexa
are normal what is her diagnosis ?
A. Incomplete abortion
B. Threatened abortion
C. Ectopic pregnancy
D. Missed abortion
E. Inevitable abortion

52. A 21-year-old G0 presents for her first gynecologic examination. She states that
she became sexually active 2 weeks ago for the first time. She has no significant medical
history. She has regular menses with some mild dysmenorrheal. During the speculum
examination, you observe a small raised 0.5 cm. it is smooth and light-bluish in color
with the appearance of a bubble under the epithelial surface and a blood vessel running
over the top. What is your diagnosis ?
A. Barthoilin’s gland cyst
B. Cervical dysplasia
C. Nabothian cyst
D. Skene’s gland cyst
E. Cervical cancer

REVISED BY TEAM “BANG JAGO” NOV 2020 UNHAS Page 98


53. A 68-years-old woman presents with vulvar pruritus since the previous year that
has been increasing over the last few months. She has tried antifungal medications
which seem to help but the symptoms keep recurring. She went through menopause at
age 49 and has not been sexually active for 10 years. She does not use any douching
products and is not taking any antibiotics. On physical examination you note thin white
epithelium of the labia minora with red oval shaped erosions varying in size from 0,5 to
1,5 cm. ho would you proceed?
A. Culture the vagina and treat with high-dose antifungal
B. Wide local excision of the lesions
C. Cryotherapy to eradicate the lesions
D. Punch biopsy of the vulvar lesions
E. Treat with moderate-high potency topical steroids

54. A 27-year-old woman presents to her gynecologist for evaluation of new vulvar
lesions. She first noticed several “bumps” on her outer labia 3 weeks ago. She is
otherwise without any associated symptoms On pelvic examination there are six flesh-
colored, nontender, 1 to 3 mm verrucous papules. Which of the following is the most
likely cause of the patient’s vulvar lesions ?
A. Human papilomavirus (HPV) serotype 16
B. Herpes simplex virus (HSV) type 1
C. Pox virus
D. Human papilomavirus (HPV) serotype 6
E. Haemophilus ducrey

55. A 45-year-old presents for evaluation because her primary care physician has
diagnosed her with pelvic organ prolapsed while performing annual care. She denies any
pelvic pressure, bulge, or difficulty with urination. Her only medical comorbidity is
obesity. for asymptomatic grade 1 pelvic organ prolapsed, what do you recommend ?
A. Conservative management with pelvic floor muscle exercises and weight loss
B. Colpocleisis obliterative procedure
C. Gellhom pessary

REVISED BY TEAM “BANG JAGO” NOV 2020 UNHAS Page 99


D. Round ligament suspension
E. Hysterectomy

56. A postmenopausal woman with symptomatic vaginal vault prolapsed elects to


process with surgical correction of the descensus. You determine that she is an
appropriate candidate for an uterosacral ligament suspension. After completion of the
surgery. Successful anatomic restoration is noted. Before you conclude the surgery,
what additional procedure is indicated?
A. Hysteroscopy
B. Defecography
C. Cystoscopy
D. Episiotomy
E. Anterior cotporrhaphy

57. A 62-year-old G2 P2 presents to the urogynecology clinic ith complaints of


urinary incontinence. She has urinary urgency and can’t make it to the bathroom before
leaking a large amount of urine. She gets up two to three times per night to urinate A
urinalysis and urine culture done 1 week ago at her PCP’s office are both negative. What
is the most likely diagnosis and appropriate treatment option for this type of urinary
incontinence ?
A. Stress incontinence, mid-urethral sling
B. Urgency incontinence oxybutynin (anticholinergic medication)
C. Overflow incontinence oxybutynin fanticholinergic medication)
D. Urinary fistula surgical repair
E. Functional incontinence bladder suspension

58. A 29-year-old G3P2A0 presents to the emergency center with complaints of


abdominal discomfort for 2 weeks. Her vital signs are blood pressure 120/70 mmHg
pulse 90 beats per minute temperature 36,9oC respiratory rate 18 breaths per minute. A
pregnancy test is positive and an ultrasound of the abdomen and pelvis reveals a viable
16-week gestation located behind a normal appearing 10 x 6 x 5,5 cm uterus Both

REVISED BY TEAM “BANG JAGO” NOV 2020 UNHAS Page 100


ovaries appear normal no free fluid is noted. Which of the following is the most likely
cause of these findings ?
A. Ectopic ovarian tissue
B. Fistula between the peritoneum and uterine cavity
C. Primary peritoneal implantation of the fertilized ovum
D. Tubal abortion
E. Uterine rupture of prior cesarean section scar

59. Invasive cervical cancer identified only microscopically, invasion is limited to


measured stromal invasion with a maximum depth of 3 mm and no wider than 7 mm
should be treated with;
A. Observation
B. Tissue ablation
C. LEEP/LLETZ
D. Simple total hysterectomy
E. Radical hysterectomy

60. The majority of vulvar, vaginal and cervical cancers appear to have a common
cause and usually caused by
A. High risk types of herpes simplex virus (HSV) infection
B. High risk types of human papiloma virus (HPV) infection
C. Increased exposure to endogenous estrogen
D. Increased exposure to exogenous estrogen
E. Chronic bacterial and parasitic infection

61. Childhood neoplastic ovarian masses most commonly originate from


A. Gonadal epithelium
B. Gonadal stroma

REVISED BY TEAM “BANG JAGO” NOV 2020 UNHAS Page 101


C. Sex cords
D. Germ cells
E. Metastatic disease

62. During a routine return OB visit, an 18-years-old G1P0 patient at 23 weeks


gestational age undergoes a urinalysis. The dipstick done by the nurse indicates the
presence of trace glucosuria. All other parameters of the urine test are normal. Which of
the following is the most likely etiology of the increased sugar detected in the urine ?
A. The patient has diabetes
B. The patient has a urine infection
C. The patient’s unnalysis is consisten with normal pregnancy
D. The patient’s urine sample is contaminated
E. The patient has kidney disease

63. A maternal lethal medicine specialist is consulted and performs an in depth


sonogram. The sonogram indicates that the fetuses are both male and the placenta
appears to be diamniotic and monochronic. Twin B is noted to have oligohydramnios
and to be much smaller than twin A. in this clinical picture all of the following are
concerns for twin Except
A. Congestive heart failure
B. Anemia
C. Hypervolemia
D. Polycythemia
E. Hydramnion

64. you are called in to evaluate the heart of a 18-year-old primigravida at term.
Listening carefully to the heart, you determine that there is a split S1 normal S2, S3
easily audible with a 2/6 systolic ejection murmur greater during inspiration, and a soft
diastolic murmur. You immediately recognize that
A. The presence of the S3 is abnormal

REVISED BY TEAM “BANG JAGO” NOV 2020 UNHAS Page 102


B. The systolic ejection murmur is unusual in a pregnant woman at term
C. Diastolic murmur are rare in pregnant women
D. The combination of a prominian and soft diastolic murmur is a significant
abnormally
E. All findings recorded are normal changes in pregnancy

A 24-year-old G1P0 woman presents for prenatal care at 8 weeks by LMP. She has
regular menses every 28 to 30 days and you confirm her gestational age with an
ultrasound today in the office. She has no past medical or surgical history. She and her
husband of 6 months planned the pregnancy and they have both been reading about
pregnancy and prenatal care. You discuss the prenatal tests for the first visit as well as
the plan throughout the rest of the pregnancy.
65. As part of this discussion, you offer her which of the following prenatal
screening/diagnostic tests ?
A. CVS
B. Amniocentesis
C. First-trimester screening
D. Quad screening
E. All of the above

66. The patient opts to undergo first trimester screening, which returns with a risk
for Down syndrome of 1 in 1,214 and risk of trisomy 18 of 1 in 987. At 18 weeks, she
gets a quad screen, and her estriol, -hCG, and -fetoprotein (AFP) were all low She has
an ultrasound, with 15 weeks size, increased amniotic fluid. Club oat omphalocele,
choroid plexus cyst and possible heart defect On the basis of the patient’s history and
data prewired. What is the most likely diagnosis ?
A. Trisomy 21
B. Trisomy 18
C. Trisomy 13
D. Turner syndrome
E. Klinefelter syndrome

REVISED BY TEAM “BANG JAGO” NOV 2020 UNHAS Page 103


67. Your next patient is a 13-year-old adolescent girl who presents with cyclic pelvic
pain. She has never had a menstrual cycle. She denies any history of intercourse. She is
afebrile and her vital signs are stable. On physical examination, she has age-appropriate
breast and pubic hair development and normal external genitalia. However, you are
unable to locate a vaginal introitus instead, there is a tense bulge where the introitus
would be expected you obtain a transabdominal ultrasound, which reveals a
hematocolpos and hematometra. What is the most likely diagnosis ?
A. Transverse vaginal septum
B. Longitudinal vaginal septum
C. Imperforate hymen
D. Vaginal atresia (MRKH)
E. Bicornuate uterus

68. An 18-year-old nulligravid woman presents to the student health clinic with a 4
week history of yellow vaginal discharge. She also reports vulval itching and irritation.
She is sexually active and monogamous with her boyfriend. They use condoms
inconsistently on physical examination, she is found to be nontoxic and afebrile On
genitourinary examination, vulvar and vaginal erytherna is noted along with a yellow,
frothy malodocous discharge with a pH of 6,5. The cervix appears to have erythematous
punctuations. There is no cervical. Uterine of adnexal tenderness The addition of 10%
KOH to the vaginal discharge does not produce an amine odor Wet prep microscopic
examination ot the vaginal swabs is performed. What would you expect to see under
microscopy ?
A. Branching hyphae
B. Multinucleated giant cells
C. Scant WBC
D. Flagellated, motile organisms
E. Epithelial cells covered with bacteria

69. An 89-year-old female patient with multiple, serious medical comorbidities


presents to discuss options for treatment of her high-grade prolapsed. The prolapse is
externalized and becoming ulcerated from friction against her undergarments. She

REVISED BY TEAM “BANG JAGO” NOV 2020 UNHAS Page 104


cannot tolerate a pessary. Her main priority is to fix or get rid of this thing but her
primary care provider has cautioned against a lengthy of open abdominal procedure.
She is not interested in future intercourse. What can you offer this patient ?
A. Nothing can be done
B. Open abdominal sacral colpopexy
C. Robot assisted laparoscopic sacral colpopexy
D. Hysterectomy colporrhaphy with anterior and posterior colporrhaphy
E. Colpocleisis

70. A 51-year-old woman presents to your office amenorrhea for the past year. Since
she has not had any hot flashes, she is wondering if she is menopausal. What blood test
would confirm the diagnosis of menopause ?
A. FSH
B. Estroten
C. Testosterone
D. Human chorionic
E. Prolactin

71. A 37-year-old female G5P5 has come to your office to discuss long term
contraception A thorough history and office examination reveal a healthy, thin female,
in no acute distress. She reports seasonal allergies and has undergone one cesarean
section in the past as well as four spontaneous vaginal deliveries and no additional
surgeries. She has never had an STI and is a nonsmoker. Her Pap Smears and HPV
screens have all been normal and are up to date. She asks you to explain which methods
will be effective for a long period of time, as she has trouble remembering to take a pill
daily with five small children at home you inform her that many methods can be
considered “long term” except which of the following?
A. 1,5 mg Levonorgestrel pill
B. Etonogestrel rod (Nexplanon)
C. Copper IUD (ParaGard)
D. Levonorgestrel IUD (Mirena)

REVISED BY TEAM “BANG JAGO” NOV 2020 UNHAS Page 105


E. Tubal Ligation/Occlusion

72. A patient returns for a postoperative checkup 2 weeks after a total abdominal
hysterectomy for fibroids. She is distressed because she is having continous leakage of
urine from the vagina. Her leakage is essentially continous and worsens with coughing,
laughing, or movement. Given her history and physical, you perform both a metthylene
blue dye test, which is negative and an indigo carnine test. Which is positive. The most
likely diagnosis is:
A. Rectovaginal fistula
B. Uretro vaginal fistula
C. Vesico vaginal fistula
D. Uretero vagina fistuta
E. Improssible to distinguish
73. A 38-year old multigravida woman complains of the painless loss of urine,
beginning immediately with coughing laughing, lathing or straining immediate cessation
of the activity stops the urine loss after only a lwe drops. This history is most suggestive
of
A. Fistula
B. Stress incontinence
C. Urge incontinence
D. Urethral diveriticulum
E. UTI

74. Select the incorrect regarding guidelines for Nuchal Translucency (NT)
measurement is
A. The margins of NT edges must be clear enough for proper caliper placement and
the fetus is not necessary in the midsagittal plane
B. The image must be magnified so that it is filled by the fetal head, neck, and
upper thorax
C. The fetal neck must be in a neutral position, not flaxed and not hyperextended

REVISED BY TEAM “BANG JAGO” NOV 2020 UNHAS Page 106


D. The amnion must be seen as separate from the NT line, and the + calipers must
be placed on the inner borders of the nuchal space with none of the horizontal crossbar
it self protruding into the space
E. The calipers must be placed perpendicular to the long axis or the fetus and the
fetus and the measurement must be obtained at the widest space of the NT

75. A 32-year-old presents for an infertility workup She and her partner have been
trying to conceive for 2 years without success. She has regular menstruation though she
mentions she has severe cramping during her cycles. She also notes she experiences
pelvic pain during sex. On examination, she is a febrile and she and she experiences a
great deal of pain during the pelvic examination. You do not note discharge on
examination Which of the following tests is required for diagnosis of the patient’s
infertility ?
A. Ultrasound
B. -hCG level
C. Pap smear
D. Laparoscopy
E. Hysterosalpingogram

76. Mrs. XY is a primigravida who is 34 weeks pregnant. Her last two serial scans
have shown a small gestational age fetus growing on the 9th centile. Her last scan shows
positive end diastolic flow with a normal PI. She reports having good fetal movements.
How should further fetal surveillance be undertaken ?
A. fortnightly umbilical artery Doppler
B. twice weekly CTG
C. twice weekly umbilical artery Doppler
D. weekly CTG
E. weekly umbilical artery Doppler CTG

A 25-year-old woman in her first pregnancy is notes to have prolonged first and second
stages of labour. She was induced at 38 weeks’ pregnancy. The baby was delivered by

REVISED BY TEAM “BANG JAGO” NOV 2020 UNHAS Page 107


forceps. After delivery the placenta she is noted to have heavy vaginal bleeding.
Abdominal examination demonstrates a relaxed uterus.
77. What is the most likely cause of bleeding
A. Uterine atony
B. Uterine rupture
C. Retained placenta
D. Genital tract laceration
E. DIC

78. What should we do if the fundus not firm after placental delivery
A. Methylergonovine (Mehergine)
B. Carboprost (Hemabate, PGF2-alpha)
C. Fundal Massage
D. Miscoprostal (PGE1)
E. Dinoprostone-prostaglandin E2

A 30-years-old multiparous woman has rapid delivery soon after arriving in emergency
room. After delivery the placenta she is noted to have heavy vaginal bleeding. Help has
been summoned. Abdominal examination demonstrates the fundus was soft
79. What is the most appropriate next step?
A. Intravenous access for fluid resuscitations
B. Uterine packing
C. Balloon tamponade
D. Suture the laceration
E. Misoprostal administration

80. After use of a 20 units of oxylocin in 1000 mL of crystalloid solution to increase


the tone of her uterus stop the bleeding. However you continue to notice a massive

REVISED BY TEAM “BANG JAGO” NOV 2020 UNHAS Page 108


bleeding form the vagina. What is the most appropriate next step in the evaluation of
this patient’s bleeding ?
A. Perform a bedside ultrasound for retained products of conception
B. Perform a bedside ultrasound to look for blood in the abdomen significant for
uterine rupture
C. Perform a manual exploration of the uterine fundus and exploration for retained
clots or products
D. Examine the perineum and vaginal for laceration during delivery
E. Consult interventional radiology for uterine artery embolization

81. Labor induction and augmentation are NOT associated with of the following
risk ?
A. Postpartum hemorrhage from uterine atony is more common in going induction
or augmentation
B. Amniotic fluid embolism in a laboring patient receiving oxytocin can be occurred
C. The increased risk for cesarean delivery undergoing induction is related with
cervical favorability (Bishop Score)
D. The uterine rupture risk is increased threefold for women in spontaneous labor
with uterine scar
E. Women whose labor is managed with amniotomy have lower incidence of
chorioamnionitis compared with those in spontaneous labor

82. According to the patient above, what is the most favorable condition for a
successful trial of labor after cesarean birth ?
A. Water broke with clear amniotic fluid
B. A previous vaginal delivery
C. In a hospital with available facility of anesthesia
D. Only one previous cesarean delivery
E. Induction given at term pregnancy

REVISED BY TEAM “BANG JAGO” NOV 2020 UNHAS Page 109


A 32-year-old woman comes to your clinics due to shortness of breath, that worsen since
2 days ago. On history taking, she told you that she had ever diagnosed of having
significant mitral stenosis. She is 33 weeks pregnant. The fetus is size-date appropriate.
She has had a recent echocardiography showing ejection fraction of 54% with moderate-
severe pulmonary hypertension
83. What is the best management for this patient currently ?
A. Perform emergency C section
B. Lung maturation and C section
C. Conservative management until term pregnancy
D. Second stage acceleration
E. Induction of labor

84. What is the most common cause of heart failure during pregnancy and the
puerperium ?
A. Chronic hypertension with severe preeclampsia
B. Viral myocarditis
C. Obesity
D. Valvular heart disease
E. Pulmonary Artery Hypertension
85. For patients with congenital heart disease, what is the most common adverse
cardiovascular event encountered in pregnancy ?
A. Heart failure
B. Arrhythmia
C. Thromboembolic event
D. Cerebrovascular hemorrhage
E. Heart axis changes

86. A 55-year-old woman presents to your office for consultation regarding her
symptoms of menopause. She stopped having periods 8 months ago and is having

REVISED BY TEAM “BANG JAGO” NOV 2020 UNHAS Page 110


severe not flushes. The hot flushes are causing her considerable stress. What should you
tell her regarding the psychological symptoms of the climacteric ?
A. They are not related to her changing levels of estrogen and progesterone
B. They commonly include insomnia, irritability, frustration, and malaise
C. They are related to a drop in gonadotropin levels
D. They are not affected by environmental factors
E. They are primarily a reaction to the cessation of menstrual flow

87. A 62-year-old woman present for annual examination. Her last spontaneous
menstrual period was 9 years ago and she has been reluctant to use postmenopausal
hormone replacement because of a strong family history of breast cancer. She now
complains of diminished interest in sexual activity. Which of the following is the most
likely cause of her complaint ?
A. Decreased vaginal length
B. Decreased ovarian function
C. Alienation from her partner
D. Untreatable sexual dysfunction
E. Physiologic anorgasmia

A 49-year-old woman experiences irregular vaginal bleeding for 3 months duration. You
performed endometrial biopsy, which obtains copious tissue with a velvety, lobutated
texture. The pathologist report shows proliferation of glandular and stroma elements
with dilated endometrial glands, consistent with simple hyperplasia Cytologic atypia is
absent
88. Which of the following is the best way to advise the patient ?
A. She should be treated to estrogen and progestin hormone therapy
B. The tissue will progress to cancer in approximately 10% of cases
C. The tissue may be weakly premaligmant and progress to cancer in approximately
1% of cases
D. She requires a hysterectomy

REVISED BY TEAM “BANG JAGO” NOV 2020 UNHAS Page 111


E. No further therapy is needed

89. Which of the following factor is protective against endometrial hyperplasia ?


A. Obesity
B. Tamoxifen
C. Oral contraceptive pills (OCPs)
D. Early menarche or late menopause
E. Unopposed exagenous estrogen therapy

You see a 19 o female who presented with primary amenorrhea. Breast development
was noted at 13 years but there has been no increase in breast size. Public and axillary
hair was noted within one year of referral. An outside ultrasound showed no uterus or
ovaries. Physical exam reveals a normal vaginal introitus with hymen present. Breasts
are Tanner 3 but seem to be more fatly than mammary tissue
90. Lab test were significant for absens estradiol, elevated gonadotropins and mildly
elevated HDEAs. With normal testosterone testosterone. A karyotype wan requested by
the endocrinologist and was found to be 46, XY (SRY gene +) What is the working
diagnosis following her initial workup ?
A. Androgen insensitivity
B. Disorders of testicular development
C. Mullerian agenesis
D. A and B
E. B and C

91. The MRI showed an infantile uterus with no dissemble gonads. During a
laparoscopic evaluation, two dysplastic gonads attached to small fallopian tubes and a
rudimentary uterus was visualized. The final diagnosis is
A. Swyer syndrome
B. Androgen insensitivity
C. Mullerian agenesis

REVISED BY TEAM “BANG JAGO” NOV 2020 UNHAS Page 112


D. Partial gonadal dysgenesis
E. Testicular Regression Syndrome

92. What type of tumor is a significant risk for such condition ?


A. Mature teratoma
B. Leydig cell tumor
C. Germ cell tumor
D. Stromal cell Tumors
E. Granulose cell tumors

Miss 25 years-old P3 comes to Gynecology outpatient clinic with cytology result low
grade SIL. She curious about the result since her last cytology result was normal 3 years
ago. She has no complaint recently. She began sexually active since 10 years ago and has
had six partner. She smokes 10 cigarettes per day since 4 years ago. Her mother was
diagnosed for cervical cancer at 44 years old and just died 3 months ago. Her child now
is a years old
93. What are patient risk factor for CIN ?
A. Her smoking habits
B. Onset of sexual activity
C. Six sex partners
D. Early childbearing
E. All mention above

94. What is next proper management for patient ?


A. IVA test
B. Colposcopy
C. No procedure need in her treatment
D. HPV DNA test
E. Endocervical curettage

REVISED BY TEAM “BANG JAGO” NOV 2020 UNHAS Page 113


95. The result showing a condylomatous acetowhite lesion with punctuation and
atypical vessels. Biopsy result confirms CIN I with HPV DNA tes positive. What do you
suggest for patient ?
A. LEEP procedure
B. Reevalutation of HPV DNA
C. Cold knife conization
D. Repeat cytology in 12 months
E. Repeat cytology in 6 months

A 16-year-old woman came to clinic with what complaint of irregular menstruation. She
had menstruation active every week months. She is also obsesis (BM 32) Physical
examination reveals she has (Ferriman Gateway 9) other physical examination within
normal limit Gynecologic examination within normal limit
96. Witch of the following criteria diagnosis of polycystic ovarian syndrome (PCOS)
at not part of the Rotterdam criteria ?
A. Oligo and volution
B. Appearance of polycystic ovaries by gynecologic ultrasound
C. Excess androgen activity
D. Ferriman Gateway score > 8
E. All statement is true

97. In PCOS increased testosterone production from the ovaries is secondary to


stimutation by which of the following hormones ?
A. Inhibin
B. Estradiol
C. Prolactin
D. Follicle stimulating hormone (FSH)
E. Lutenizing hormone (LH)

REVISED BY TEAM “BANG JAGO” NOV 2020 UNHAS Page 114


A 55 year-old nulliparous woman who underwent menopause at age 50 years complaint
of a 1 month history of vaginal bleeding and smells. Her medical history reveals she has
hypertension and controlled with anti hypertensive agent and also she has diabetes
mellitus contribute with oral hypoglicemic agent how examination she weight 50 kg and
152 cm. he her blood pressure in 150/90 mmHg. Heart and long examination are normal
The abdomen is obese and no masses are palpated. The external genitalia appear
normal and the uterus scretis to be entarged without adnexal masser palpable
98. What is the probable diagnoses of the patient ?
A. Cervical cancer
B. Hyperplasia endometrium
C. Endometrial carcinoma
D. Uterine fibroid
E. Ovarian cancer

99. Which of the following does not increase a woman’s risk of developing
endometrial cancer ?
A. Obesity
B. Smoking
C. Diabetes mellitus
D. Tamoxifen
E. Unopposed estrogen

100. Which of the following subtypes is NOT a type II endometrial cancer?


A. Clear cell carcinoma
B. Papillary serous carcinoma
C. Carcinosarcoma
D. Endometrioid adenocarcinoma
E. None of the above

REVISED BY TEAM “BANG JAGO” NOV 2020 UNHAS Page 115


REVISED BY TEAM “BANG JAGO” NOV 2020 UNHAS Page 116
NOVEMBER 2017

30. A 31-year -old G lPO woman at 39 week s and 4 days presents to labor and delivery
unit, with regular contractions occurring every 3 to 5 minutes. Her contracti on last 30
to 90 seconds . She not s ure If she's been leaking any fluid from her vagina . You take
her history and conduct a physical examination. ROM would be supported b which
of the allowing?

A. Nitrazine paper remaining orange when exposed to fluid In the vagina

B. negative fern test

C. An ultras ound with a normal AFI

D. A negative tampon test

E. Speculum examination with evidence of pooling in the vagina

31. On examination you attempt to determine the presentation of the fetus. Which
of the following presentations and positions would be most favorable to achieve a vaginal
delivery?
A. Breech
B. Transverse
C. Vertex with occiput posterior
D. Vertex with occiput anterior
E. Vertex with occiput transverse

33. A 26-year-old G1P0 woman presents for a prenatal visit at 34 weeks’ gestation.
She complains of some mild nausea and vomiting over the past 3 days. She has no
headache and no visual changes. Her BP is 142/83 mm Hg. On examination, she has 21
lower extremity pitting edema, and 3+ reflexes bilaterally with four beats of clonus. A

REVISED BY TEAM “BANG JAGO” NOV 2020 UNHAS Page 117


urinalysis dip has 1+ protein. Which of the following laboratory tests would NOT be
helpful at this point?
A. Platelet
B. WBC
C. LFTs
D. LDH
E. Obstetric ultrasound

34. A 24-year-old G2P0010 woman at 8 weeks’ GA presents for an initial prenatal


visit. Her pregnancy is complicated by a history of IV drug use as a teenager and HIV. She
had never been on HAART medication prior to this pregnancy. You order a viral load and it
returns at 10,000. Her CD4 count is normal (.500). She is otherwise healthy and has no
other significant medial history. When would you recommend she start HAART?
A. At this visit
B. At 37 weeks in preparation for delivery
C. At the beginning of the second trimester
D. Immediately after delivery
E. Only if it is medically indicated for maternal health

35. A 28-year-old G2P1001 woman who is 18 weeks pregnant presents to the ED


with increasing left lower extremity swelling, redness, and pain. She first noticed these
symptoms 2 days ago and has tried elevation and warm compresses, which have not
helped. She has no personal or family history of blood clots. She is otherwise healthy and
her only medication is a prenatal vitamin. In the ED, her vital signs are normal. Physical
examination is significant for left lower extremity edema, calf tenderness, and erythema.
You suspect she has a lower extremity DVT and you plan to start her on anticoagulation
therapy. Which of the following is the most important test to confirm your diagnosis?
A. Venography
B. D-dimer
C. Left lower extremity venous Doppler ultrasound
D. MRI of the left lower extremity

REVISED BY TEAM “BANG JAGO” NOV 2020 UNHAS Page 118


E. No further testing is indicated

36. A 36-year-old G7P50015 woman has just delivered a 4,500 g female infant at 39
weeks gestation. She underwent induction of labor with oxytocin for severe preeclampsia
diagnosed with systolic BPs elevated to 160 mm Hg. Her pregnancy was complicated by
uncontrolled gestational diabetes and resultant polyhydramnios. She was placed on
magnesium throughout her induction for seizure prophylaxis. She had an epidural placed
during the first stage of labor and remained on a normal labor curve throughout. Her
second stage of labor lasted 3½ hours; she was, however, able to deliver vaginally with
preemptive McRoberts maneuvers and steady traction. The third stage of labor lasted 10
minutes and the placenta was delivered intact. Immediately after the third stage her
bleeding was significant with the expulsion of blood clots and a fundus that was notable
for bogginess. Which of the following are not risk factors for postpartum hemorrhage?
A. Advanced maternal age
B. Grand multiparity
C. Prolonged use of oxytocin during labor
D. Polyhydramnios
E. Prolonged exposure to magnesium during labor

48. A 58-year-old G3P3003 Caucasian, postmenopausal woman comes to your office. She
has been menopausal since age 50. She has a negative past medical and surgical history.
She took hormone replacement for about 2 years but stopped due to concerns of an
increased risk of cancer that she heard about from friends. Prior to the onset of
menopause, she had a history of normal and regular menses. She has had annual GYN
care with you, and has never been diagnosed with cervical dysplasia. Her last Pap smear
with complaint of nipple discharge milky in color, comes from both breasts, and is

REVISED BY TEAM “BANG JAGO” NOV 2020 UNHAS Page 119


present even when she doesn’t express it 372 HPV was obtained last year and both
were negative. She has recently become sexually active with a new partner and has noted
some spotting with intercourse as well as that she notices on wiping for the past
associated with occasional mild lower abdominal cramping. She complains of a general
feeling of vaginal dryness and does have pain and dryness with intercourse. She has no
other complains what so ever. Her most likely diagnosis is

A. Endometrial cancer

B. cervical cancer

C. Urogenital atrophy

D. bleeding dyscrasia

E. uterine fibroids

49. The patient comes back to your office and she is pregnant. What hormone does the
developing trophoblast produce?

A. Human chorionic gonadotropin (hCg)

B. Progesterone

C. Androstenedion

D. LH

E. Estrogen

50. A 34-year-old G2P2 woman comes to the emergency department with 8 hours of . She
is and uses Depo-Proverafor contraception. She was . Her temperature is 38.5°C (101.3°F),
pulse rate is 114/min, respirations are 22/min, and BP is 110/70 mm Hg. On examination,
her abdomen is soft with right lower quadrant tenderness. Voluntary guarding is present

REVISED BY TEAM “BANG JAGO” NOV 2020 UNHAS Page 120


without rebound. Pelvic examination shows no cervical motion tenderness or uterine
tenderness. The right adnexa is exquisitely tender and fullness is appreciated. Her and
there are 15% bands. . What is the most likely diagnosis?

A. Ovarian torsion

B. Appendicitis

C. Tubo-ovarian abscess

D. Cervicitis

E. Ectopic pregnancy

52. A 36-year-old G1P0 is 31 weeks and 5 days by LMP and is sure of her dates. Her
pregnancy has been complicated by persistent nausea and vomiting, back pain, and lower
extremity swelling. She comes to you for a routine prenatal visit. She had a quad screen at
16 weeks that was normal. She is having a girl. Her low back pain is no longer relieved
with a heating pad and she finds that she needs pain relief to make it through each work
day. Which of the following options would be safest for her?

A. Ibuprofen

B. Aspirin

C. Oxycodone

D. Flexeril

E. Tylenol

53. A G3P2002 woman at 35 weeks is seen in your office for her prenatal visit. She is
concerned because she has not felt her baby moving as much as she used to. Her
pregnancy has been uncomplicated and her past two pregnancies ended in full term,

REVISED BY TEAM “BANG JAGO” NOV 2020 UNHAS Page 121


normal spontaneous vaginal deliveries. When formal antenatal testing is done, which of
the following is most reassuring?

A. Late decelerations on fetal monitoring

B. A contraction stress test (CST) with variable fetal heart rate (FHR) decelerations
with contractions, but moderate variability

C. A nonstress test (NST) with two accelerations of the FHR in 20 minutes that are
at least 15 beats above baseline and last for at least 15 seconds

D. An increase in the systolic to diastolic ratio in the umbilical artery blood flow

E. A score of 6 on a BPP

54. A 28-year-old P0010 woman presents to the emergency department with abdominal
pain since the past day. She reports a 1-week history of nausea with occasional
vomiting. She has noticed some breast tenderness as well. She denies dysuria, vaginal
bleeding, or any bowel symptoms. She reports that her last period was 4 weeks ago, but
was lighter than normal. She has been using condoms for contraception. On arrival, her
vital signs include a temperature of 37°C, BP of 117/68, pulse rate of 78 beats per minute,
and respiratory rate of 16 breaths per minute. Cardiovascular and respiratory
examinations are normal. She notes some suprapubic abdominal discomfort with
palpation, but she does not have rebound tenderness or guarding. A speculum
examination reveals a closed cervix without bleeding. A pelvic examination is mildly
uncomfortable and reveals a normally sized, anteverted uterus, and palpably normal
adnexa. A urine pregnancy test is positive. The quantitative β-hCG level is 1,300
mIU/mL. The patient reveals that this was an unplanned, but desired pregnancy. What
follow-up recommendations do you give this patient?

A. Make an appointment with her primary OB/GYN for an initial prenatal visit

B. This is likely an ectopic pregnancy and she should proceed with

REVISED BY TEAM “BANG JAGO” NOV 2020 UNHAS Page 122


methotrexate therapy

C. She should undergo urgent laparoscopy for evacuation of an ectopic pregnancy

D. She should return in 48 hours for a repeat β-hCG

E. She has likely had a SAB and does not need further follow-up

55. A 40-year-old G2P0 woman at 7 weeks GA by LMP presents for her first prenatal
visit. She spontaneously conceived after 18 months of trying. She is excited about the
pregnancy, but at the same time is concerned about potential risks for herself as well as
the baby because of her age. Her husband is 52 years old, healthy, and has fathered two
children from a prior marriage. The week prior to the visit, she experienced spotting that
lasted 3 days and then resolved. Currently, she has no complaints. She has no past
medical or surgical history except for a miscarriage 3 years ago. She has regular periods
every 30 days. You offer her which of the following prenatal screening/diagnostic tests?

A. CVS

B. Amniocentesis

C. First-trimester screening

D. Quad screening

E. All of the above

56. You are providing prenatal care to a 22-year-old G1P0 woman at 16 weeks GA by LMP.
She has had a relatively smooth pregnancy without complications thus far. At 5950 and
215 lb she has an obese BMI, otherwise without medical or surgical history. She
presented to prenatal care at 14½ weeks, and so missed first-trimester screening. She
undergoes the quad screen and has an elevated level of maternal serum alpha-
fetoprotein (MSAFP). The ultrasound reveals a myelomeningocele. The increased
incidence of this finding is associated with which of the following medications when used

REVISED BY TEAM “BANG JAGO” NOV 2020 UNHAS Page 123


in pregnancy?

A. Valproic acid

B. Lithium

C. Fluoxetine

D. Prednisone

E. Acetaminophen

58. A 34-year-old G3P2002 woman at 38 weeks and 6 days was admitted to labor
and delivery unit for active management of labor after it was determined that her
membranes had ruptured and she was dilated to 3 cm. Her cervix has been steadily
dilating and now she is at 6 cm. She is very uncomfortable and finds her contractions very
painful. Her partner is also very concerned that she needs pain relief. With adequate pain
control she dilates to 10 cm and second stage begins. Which of the following is the
correct order of the cardinal movements of labor?

A. Internal rotation, engagement, descent, flexion, external rotation

B. Engagement, descent, internal rotation, flexion, external rotation

C. Internal rotation, descent, engagement, flexion, external rotation

D. Engagement, descent, flexion, internal rotation, external rotation

E. Engagement, descent, internal rotation, flexion, external rotation

59. A 22-year-old G1P0 African American woman at 36 weeks by LMP consistent with 12-
week ultrasound with limited prenatal care presents via ambulance to the L & D triage
unit complaining of severe abdominal pain and profuse vaginal bleeding. The patient is

REVISED BY TEAM “BANG JAGO” NOV 2020 UNHAS Page 124


unstable and unable to communicate coherently. The EMT reports that initially her BP
was 180/100 mm Hg and pulse rate was 110 bpm, but she has lost at least 500 mL blood
in route. On examination her BP is 90/50 mm Hg, pulse rate is 120 bpm, she appears to be
in significant pain, is unable to answer questions, and her abdomen feels rigid. What is
the most important laboratory test to order for this patient emergently?

A. Type and cross

B. CBC

C. Urine drug screen

D. Chemistry

E. PT, PTT, INR

60.

61. All of the following are risk factors for uterine fibroids except:
A. African American heritage
B. Multiparity
C. Early menarche
D. Perimenopause
E. Hypertension

62. A 29-year-old G3P1102 woman at 29 weeks 3 days presents to labor and delivery
triage for evaluation of abdominal pain. Her pain started 2 hours ago and comes and goes
every 5 minutes. She denies any leaking fluid, change in vaginal discharge, or vaginal
bleeding. Her baby has been active. Her pregnancy is complicated by a history of a urinary
tract infection at 10 weeks with GBS and a history of preterm birth at 31 weeks with her
last child. She is currently taking progesterone injections weekly and a prenatal vitamin.
On vaginal examination, her cervix is closed, 25% effaced, and 23 station. Uterine
contractions are noted every 4 to 5 minutes with a category 1 tracing. A fetal fibronectin

REVISED BY TEAM “BANG JAGO” NOV 2020 UNHAS Page 125


test returns positive. Repeat examination after 1 hour shows dilation of 1 cm, 50%
effacement, and 2/3 station. The team decides to start her on magnesium sulfate for
tocolysis and administers the first dose of betamethasone. What side effects or
complications should you counsel your patient that she might experience from
magnesium sulfate?
A.Flushing, diplopia, headache
B.Flushing, dizziness, headache
C.Headache, tachycardia, anxiety
D.Headache, flushing, dizziness
E.Constriction of the ductus arteriosis in the neonate

63. You are working in the emergency department when an 18-year-old Caucasian
woman arrives via ambulance. EMS reports that she was found seizing in a local drug
store approximately 10 minutes ago. She appears to be 7 to 8 months pregnant. She had
no family or friends with her, but police have contacted family who are on the way to the
emergency department. Here vital signs on arrival are as follows: BP, 180/116 mm Hg;
heart rate, 76 bpm; respiratory rate, 16 bpm; oxygen saturation, 98%. Her pants are
soiled and she is not responding to questions at this time. Bedside ultrasound
demonstrates fetal cardiac activity in the 130s. Quick bedside biometry estimates
gestation age to be 32 weeks 1 day. You begin empiric magnesium sulfate therapy. What
is the most appropriate next step in management?
A. Intubation to protect airway
B. IV labetalol
C. Head CT
D. Lumbar puncture to rule out infection
E. Delivery

64. A 38-year-old G3P2002 woman presents at 40 weeks 3 days with contractions to labor
and delivery triage. Contractions started 1 hour ago and are very painful. The patient
denies leaking fluid but did notice blood and mucus on her underwear. The baby has not
been particularly active since contractions started. Her pregnancy has been complicated
by A2GDM. Fasting blood glucose are usually between 80 and 90 mg/dL with 1-hour
postprandial values between 120 and 140 mg/dL. Her prepregnancy weight was 130 lb

REVISED BY TEAM “BANG JAGO” NOV 2020 UNHAS Page 126


and she is 5 ft 5 in (BMI 21.6). She has gained 30 lb this pregnancy (BMI 26.6).
Hemoglobin A1C is 6.0%. Fetal ultrasound at 20 weeks demonstrated normal fetal
anatomy. Repeat ultrasound for growth at 38 weeks demonstrated fetus with weight in
the 90th percentile and an EFW of 4,350 g. Her last pregnancy was complicated by
A1GDM and she delivered a 4,200 g infant without complications. Initial cervical
examination reveals dilation of 6 cm 50% effacement, and 21 station. Two hours later the
nurse calls you to the room after the patient’s water breaks. Examination shows complete
dilation and effacement, and fetus a 1/1 station. The patient has a strong urge to push
and begins pushing. The delivery is complicated by a second degree perineal laceration
and a postpartum hemorrhage of 600 mL. Fetal weight is 4,560 g and the Apgar scores are
6, 8. Which of the following is most likely the cause of fetal macrosomia in this case?
A. Maternal weight gain in pregnancy
B. Gestational diabetes
C. Poor glycemic control
D. Advanced maternal age
E. Postterm pregnancy

65. which of the findings listed below would NOT increase your suspicion that she
has endometriosis.
A. A fixed deviated uterus
B. Uterosacral nodularity on rectovaginal examination
C. Tender adnexa
D. An enlarged irregular uterus
E. A fixed adnexal mass
66. A 36-year-old G2P1001 woman at 12 weeks' gestation presents to clinic for routine
prenatal visit. She reports her nausea has resolved and denies vaginal bleeding. Her
pregnancy has been uncomplicated. Her prior pregnancy 2 years ago was complicated by
the diagnosis of GH that led to an induction of labor and cesarean delivery. She has no
other medical history. On examination her BP is 138/84 mm Hg, her body mass index
(BMI) is 36 kg/m, and a urine dip shows trace protein. The patient's 24-hour urine shows
100 mg of protein. You counsel the patient on the pregnancy complications associated
with chronic hypertension as WON as management. What is the next best step in her
managernent?

REVISED BY TEAM “BANG JAGO” NOV 2020 UNHAS Page 127


A. Expectant management with close Observation for early signs Of preeclampsia
and fetal growth restriction
B. Start antihypertensiuc therapy
C. Hospitalization for lurther maternal and fetal testing
D. Bed rest
E. Recommend pregnancy termination

67. A 32-year-old G0 woman with type 1 diabetes mellitus (T1DM) presents for a
preconception visit. She was diagnosed with T1DM at age 4, and other than some
challenges with glucose control during her teen age years, she generally has good control
per her report. She uses a subcutaneous insulin pump. She has no history of retinopathy,
renal disease, heart disease, proteinuria, peripheral neuropathy, or any other medical
conditions. On examination, she is 5960 tall and weighs 122 lb. Her BP is 128/76 mm Hg.
You order a laboratory test and her HgbA1c returns at 11. You advise her which of the
following?
A. She should go ahead and start trying to get pregnant
B. She should aggressively try to lower her HgbA1c to less than 9 to reduce the risk
of preterm birth
C. She should aggressively lower her HgbA1c to less than 5 in order to reduce her
risk of preeclampsia
D. She should aggressively lower her HgbA1c to less than 7 to reduce her risk of
congenital anomalies
E. She should go ahead and start trying to get pregnant, but also slowly reduce her
blood glucoses with a HgbA1c target of less than 9

68. A 46-years-old G2P2 obese woman is referred her primary physician because of
increasingly heavy and painful menses over the last 18 months, She has tried oral
Contraceptive with improvement in her pain. She reports no other history of pelvic pain
or abnormal bleeding in past, She has never had an abnormal Pap smear and stales she
has never had any infections “down there.” Her only medical problems are her obesity
and gastroesophageal reflux disease. On examination, you note normal external genitalia,
vagina, end cervix. However, her uterus is enlarged, mildly and softer than you expected.
She has no adnexal rnass or tenderness. Which of these diagnoses is the least likely choice
to keep in your differential?

REVISED BY TEAM “BANG JAGO” NOV 2020 UNHAS Page 128


A. A Leiomyoma
B. Adenomyosis
C. Irritable bowel syndrome
D. Endometrial hyperplasia
E. Endometriosis

69. An 18-year-old nulligravid woman presents to the student health clinic with a 4-
week history of yellow vaginal discharge. She also reports vulvar itching and irritation. She
is sexually active and monogamous with her boyfriend. They use condoms inconsistently.
On physical examination, she is found to be nontoxic and afebrile. On genitourinary
examination, vulvar and vaginal erythema is noted along with a yellow, frothy,
malodorous discharge with a pH of 6.5. The cervix appears to have erythematous
punctuations. There is no cervical, uterine, or adnexal tenderness. The addition of 10%
KOH to the vaginal discharge does not produce an amine odor. Wet prep microscopic
examination of the vaginal swabs is performed. What would you expect to see under
microscopy?
A. Branching hyphae
B. Multinucleated giant cells
C. Scant WBC
D. Flagellated, motile organisms  tricomonas vaginalis
E. Epithelial cells covered with bacteria

70. A 36-year-old G2P2 woman presents to her gynecologist with a 3-week history of
vaginal irritation and fish-smelling vaginal discharge. She recently tried an over-the-
counter antifungal treatment without any improvement in her symptoms. She is sexually
active in a monogamous relationship with a male partner of 5 years and she uses a
contraceptive ring (NuvaRing). Genitourinary examination shows a thick white discharge.
The remainder of her examination is normal. Microscopic evaluation of a saline “wet
prep” of the vaginal secretions reveals decreased lactobacilli, a few WBCs, and vaginal
epithelial cells with stippled appearance. Risk factors for developing this condition include
all of the following except:
A. multiple sexual partners
B. cigarette smoking

REVISED BY TEAM “BANG JAGO” NOV 2020 UNHAS Page 129


C. douching
D. contraceptive ring (NuvaRing)
E. lack of vaginal lactobacilli

71. A 28-year-old G1P1 presents to the emergency department 4 days after primary
cesarean section with complaints of fever, malaise, and increased lower abdominal
pain for the past 6 hours. Her labor course was complicated by prolonged rupture of
membranes and stage 2 arrest due to cephalopelvic disproportion resulting in a
cesarean delivery. Her postoperative course was uncomplicated and she had been
discharged home stable the day prior to presentation. Her temperature is 38.1°C
(100.6°F), pulse rate is 102/min, respirations are 20/min, and BP is 110/70 mm Hg.
Abdominal examination shows fundal tenderness. The incision is intact without
erythema, warmth, or discharge. On pelvic examination, there is foul-smelling lochia.
Her WBC count is elevated and there is moderate blood on urine analysis. Which of
the following is the most appropriate next step in management?

a. Dilation and curettage

b. Discharge home with oral doxycycline

c. Exploratory laparotomy

d. CT scan of abdomen and pelvis

e. Administer IV clindamycin and gentamicin

72. During what time period is vertical transmission of HIV most 'likely to occur?

a. Early antepartum

b. Late antepartum

c. Intrapartum/labor and delivery

REVISED BY TEAM “BANG JAGO” NOV 2020 UNHAS Page 130


d. Postpartum

e. All of the above confer equal risk of vertical transmission

73. An 89-year-old female patient with multiple, serious medical comorbidities presents
to discuss options for treatment of her high-grade prolapse. The prolapse is externalized
and becoming ulcerated from friction against her undergarments. She cannot tolerate a
pessary. Her main priority is to "fix or get rid of this thing," but her primary care provider
has cautioned against a lengthy or open abdominal procedure. She is not interested in
future intercourse. After the patient about counseling colpocleisis, she expresses
concem about losing her potential for future intercourse. She is not ready to
proceed with any surgical repairs. She asks about the use of topical estrogen cream.
Which of the following is NOT a estrogen contraindication toadministration?

a. Endometrial cancer
b. Stroke
c. Active arterial thromboembolic disease (i.e., myocardial infarction)
d. Administration of progestin
e. Hormone receptor positive breast cancer

74. An 82-year-old G3P2 woman is brought to your office by her caregiver from a local
retirement home. She has a diagnosis of dementia that has been worsening over the past
year. She is followed closely by her PCP and saw him recently for her general conditions.
She is communicative but has a poor memory. Her history is obtained from her caregiver.
She is ambulatory with minimal assistance and is able to follow commands. Her BMI is
23.5. Her caregiver tells you that over the past year she has had an increase in the
number of urinary leakage episodes. She wears adult diapers (Depends), which have to be
changed at least two times per day due to leakage. In the morning she wakes up with a
wet pad. Her pelvic examination is normal except for atrophic vaginitis (consistent with
menopause). Urinalysis with culture and sensitivity are negative. What is the initial
management of this patient?

a. Pelvic floor muscle exercises, 3 sets of 10 per day

b. Bladder training, she should be instructed to empty her bladder every 2 to 3 hours

c. Weight loss

d. Incontinence pessary

e. Expectant management

REVISED BY TEAM “BANG JAGO” NOV 2020 UNHAS Page 131


75. Bartholin gland duct cysts form indirect response to Which Of the following?

a. Vulvar irritation

b. Cervical gonorrhea

c. Gland duct Obstruction

d. Chronic lichen sclerosis

e. All of the above

76. A 21 years old G4P2 at 17 weeks gestation presents for her first prenatal care visit. She
has a history of prostitution, but she denies engaging in such activities for the past month.
During examination, a painless lesion is noted on the right labia. The most likely diagnosis
is which of the following?

a. Chancroid

b. Primary syphilis

c. Bartholin gland duct abcess

d. Herpes simplex virus infection

e. Condiloma acuminate

77. During a physical examination myrtiform caruncles may be noted. What are
they?

REVISED BY TEAM “BANG JAGO” NOV 2020 UNHAS Page 132


a. circumferential nodules in the areola of the breast

b. healing Bartholin’s cysts

c. remnants of the Wolffian duct

d. remnants of the hymen

e. remnants of the Müllerian duct

78. During delivery, which of the following muscles is most likely to be obviously torn?

a. ischiocavernosus muscle

b. bulbocavernosus muscle

c. superficial transverse perineal muscle

d. levator ani muscle

e. coccygeus

79. A 38-year-old African American woman presents with heavy menses and an enlarged
uterus. After an examination the clinical diagnosis is leiomyoma of the uterus. Which of
the following best describes this finding?

a. soft, interdigitating mass of the uterine wall

b. a premalignant papule of the uterine wall

c. a rapidly dividing necrotic malignancy

REVISED BY TEAM “BANG JAGO” NOV 2020 UNHAS Page 133


d. a rounded, smooth, firm, well-circumscribed mass

e. erythematous, tender, and hereditary

80. An ovary is removed for frozen section pathologic examination. The ovary is enlarged,
with small surface excrescences. Pathologic examination reveals numerous cysts lined by
serous epithelium with six to eight cell layers piled on top of one another to form the cyst
walls. The cells show marked cytologic atypia, and nests of similar cells are present in the
ovarian stroma. Round laminated calcium bodies are also seen. What diagnosis does this
histologic description indicate?

a. normal proliferative phase follicle

b. corpus luteum cyst

c. ovarian endometriosis

d. borderline ovarian carcinoma

e. cystadenocarcinoma

81 Histologic examination of the normal breast from a postmenopausal woman as compared


to the breast from a premenopausal woman would show which of the following?
a. A Decreas in the number and size of acinar glands and ductal elements, with decreased
density of the breast parenchyma
b. An increase in breast size and turgidity because of an increase in the density of the
parenchyma
c. Increase in number and size of acinar cells and a widening of the ductal lumens
d. Significant atrophy of the adipose tissue of the breast with little change in the actual
breast parenchyma
e. No significant change in hystologi

82 An amniocentesis results show a fetus with 45XO. In counseling the parents, how would
you explain that the genetic sex is determined?
a. At ovulation
b. At conception

REVISED BY TEAM “BANG JAGO” NOV 2020 UNHAS Page 134


c. By the presence or absence of testosterone
d. In the absence of Mullerian-inhibiting factor
e. Psychosocially after birth

83 Which of the following is the result of lack of fusion of the Mullerian duct system?
a. Uterine didelphys
b. Transverse vaginal septum
c. Unilateral renal agenesis
d. Imperforate hymen
e. Ovarian remnant syndrome

84 A couple is concerned about the safety of antenatal ultrasound. What should you counsel
them regarding the procedure?
a. Ultrasound has been in use for almost 40 years with noted side effects
b. The reason it is called ultrasound is that it is safe, as verified by the Food and Drug
Administrastion (FDA)
c. Ultrasound is done only by trained sonographers to ensure safe practice
d. Ultrasound may be associated with cataracts and hearing loss in animals, if used
continually, and thus it is used only when indicated
e. Antenatal ultrasound may be associated with heating in tissue and thus is used only when
indicated
85 A baby presents with ambiguous genitalia. A full chromosome count is sent and will
return in 72 hours. Your laboratory can perform a test for barr body so you can provide a
preliminary answer sooner. What is the Barr body?
a. The condensed, nonfunctioning X chromosome
b. The darkest, widest band found on chromosomes
c. An extra lobe on the female polymorhonuclear leukocytes
d. Found only in the female
e. The largest chromosome in the female genotype

86 It is now possible to adrnirister GnRH in either a brief pulse or continuously. This allows
diagnostic and therapeutic interventions in the dihipothalamic- pituitary axis. To
anticipate a normal response to GnRH stimulus one must understand how GnRH controls
LH and FSH release. Which of the following is true concerning GnRH – stimdated LH
secretion?
a. Associated with steady LH release
b. Enhanced by gonadotrope exposure to coninuous GnRH
c. Enhanced by gonadotrope exposure to estrogen

REVISED BY TEAM “BANG JAGO” NOV 2020 UNHAS Page 135


d. Enhanched by gonadotrope exposure to progesterone
e. Increased by gonadotrope exposure to testosterone

87 A 22 year old G3P2A0 who had a hematocrit of 36% at her initial obstetrical examination
at 12 weeks is found to have a hematocrit of 30% at 28 weeks when checked along with
her 1 hour glucola. Based on the indices of the red blood cells on the CBC, you diagnose
iron deficiency. She ask why that occurred since she has been taking her prenatal
vitamins. As part of the explanation, you note that. Which of the following maternal
measurements or findings is first decreased by the iron requireents of pregnancy?
a. Bone marrow iron
b. Hemoglobin
c. Jejunal absorption of iron
d. Red cellsize
e. Serum iron binding capacity

88 Apoorly controlled class D diabetic patient desired a repeat cesarean section. An


amniocentesis to verify pulmonary maturity was done prior to scheduled surgery at 37
weeks gestation. The L/S Ratio was 2: 1 phosphatidyl-glycerol was absent. An infant was
delivered who developed infant respiratory distress syndrome (IRDS). What was the most
likely reason?
a. Diabetic patients do not produce lecithin
b. Fetal lung maturation may be delayed in maternal diabetes
c. Foam test not done
d. The L/S test was done on fetal urine
e. Maternal blood was present in the specimen

89 A 32 year old presents for an infertility workup. She and her partner have been trying to
conceive for 2 years without success. She has regular menstruation, thought she
mentions she has severe cramping during her cycles. She also notes she experiences
pelvic pain during sex. On examination, she is a thin, well- developed woman. She is a
febrile, and she experiences a great deal of pain durung the pelvic examination. You do
not note discharge on examination. Which of the following test is required for diagnosis
of the patient’s infertility?
a. Ultrasound
b. B-hCG level
c. Pap smear
d. Laparascopy
e. Hysterosalpingogram

REVISED BY TEAM “BANG JAGO” NOV 2020 UNHAS Page 136


90 A 45 year old patient post op day 6 from total abdominal hysterectomy presents to the
emergency room (ER )complaining of serous fluid that is oozing from her incision. The
patient’s BMI is 33. She has diabetes and required an open laparotomy for her surgery.
On her examination, the skin incision is separated, and a Q tip can be placed past the
fascial layer. What is the most likely cause of this patient’s condition?
a. Fascial dehiscence
b. Hematoma formation
c. Normal healing
d. Infection development
e. Seroma formation

REVISED BY TEAM “BANG JAGO” NOV 2020 UNHAS Page 137


18 yo nulliparous patient attends the gynaecology clinic for haevypainful menstrual bleeding. She is in a
sexual relationship UNAS MARET 2018

1. A 62 Yo woman present for annual examination. Her last spontaneous


menstrual period was 9 year ago, and she has been reluctant to use
postmenopausal hormone replacement thearpy because if strong family history
of breast cancer. She now complain of diminished interest in sexual activity.
Which of the following of the most likely cause of her complaint?
a. Decreased vaginal length
b. Alienation from her partner
c. Untreatable sexual dysfunction
d. Decreased ovarian function
e. Physiology anorgasmia
2. A 39 year old patient presents with symptoms of leakage of urine upon
coughing, sneezing and during exercise. The symptoms started following the
birth of her second child 18 months ago. What would be the first line of
management?
a. Biofeedback/electrical stimulation
b. Bladder retraining
c. Insertion of a midurethralretropubic tape
d. Pharmacotherapy with duloxetine
e. Supervised pelvic floor muscle training
3. You were asked to perform a total abdominal hysterctomy because of a large
right broad ligament fibroid. How best should you avoid ureteric injury?
a. Ask a urological surgeon to assist you to help prevent injury
b. Dissection and visualization of the ureter
c. Introperative insertion of a ureteric catheter to identify the ureter

REVISED BY TEAM “BANG JAGO” NOV 2020 UNHAS Page 138


d. Intraoperative insertion of illuminating ureteric catheters
e. Preoperative intravenous urography to identify the course of the ureter
4. Which of the following vulval skin disorders is associated with the highest risk of
developing malignant disease?
a. Contact irritant dermatitis
b. Lichen planus
c. Lichen sclerosus
d. Squamous cell hyperplasia
e. Psoriasis
5. A woman is found to have a unilateral invasive vulvar carcinoma that is 2 cm in
diameter but not associated with evidence of lymph node spread. Initial
management should consist of
a. Chemotherapy
b. Radiation therapy
c. Simple vulvectomy
d. Radical vulvectomy
e. Radical vulvectomy and bilateral lymphadenectomy
6. An 18 Yo girl present with a large abdominal mass with abdominal pain. She
claims the mass has increase in size within the last three months. A laparotomy
and unilateral salpingo oophorectomy is perfomed. The histology report shows a
mesodermal core with a central capillary schiller duval body. Which ovarian
tumor is the most likely to be?
a. Choriocarcinoma of the ovary
b. Dysgerminoma
c. Embryonal carcinoma
d. Endodermal sinus tumour
e. Serous adenocarcinoma
7. A 21 Yo G1P0 woman presents at 25 weeks gestation complaining of headache
for the past 36 hours. She has had regular prenatal visit since first trimester. A

REVISED BY TEAM “BANG JAGO” NOV 2020 UNHAS Page 139


20 week ultrasound redated her pregnancy by 2 weeks as it was 15 days earlier
than her LMP dating. She has a BP of 150/105 mmHg. She does not have chronic
hypertension. The patient denies having right upper quadrant (RUQ) pain but
because of your high suspicion ov severe preeclampsia you order a CBC, Liver
enzymes, renal function test, and a 24 hour urine protein collection. Her
laboratory test results reveal a normal platelet count and liver enzymes. Protein
urine level is 600 mg in 24 hours. Her headache has resolved after a dose of
acetaminophen. What is the next best step in her management?
a. Begin induction of labor
b. Give her a prescription fo labetalol and have her follow up clinic in 2 weeks
c. Immediate delivery
d. Give her nifedipine and have her follow up in clinic in 1 weeks
e. Hospitalization for further evaluation and treatment
8. A 34 Yo G2P1 woman with obesity came to your clinic. She presents at
approximately 10 weeks estimated gestational age. Her BMI was 32 kg/m 2
before get pregnant. In her previous pregnancy, sherequired insulin thearapy for
GDM, and she has family history for type 2 diabettes mellitus (T2DM). she
delivered at 38 minggu and her baby boy weighed 4500 g. in addition to the
routine prenatal laboratory test, what other testing do you also obtain at this
time?
a. An ultrasound to estimate fetal weight’random blood glucose
b. Random blood glucose
c. The best time to perform glucose challenge test is at 24-28 weeks gestation, so no
testing needed at this time
d. A 75 g load glucose challenge test with fasting and a 2 hour blood glucose
e. She has GDM so no other testing needed
9. A 32 YO primigravida commences a planned delivery in a rural stand alone mid
wifery unit. After 8 hoyrs of established labour, the second stage of labour is
diagnosed. The woman develops an urge to push 1 hour later and she

REVISED BY TEAM “BANG JAGO” NOV 2020 UNHAS Page 140


commences pushing. After 30 minutes late decelerations are heard on
intermittent auscultation. On examination the fetus is cephalic, 2/5 palpable per
abdomen, fully dilated, direct OP and at station spines -1. There is 3+ caput and
3+ moulding. A decision is made for transfer to hospital, although this is delayed
because of treacherous snow covered and ice covered roads. On arrival at
hospital 3 hours later the CTG is severely abnormal, with examination findings
unchanged and a category 1 cesarean section is performed. Ten minutes after
delivery resuscitationis stopped for a few seconds while the neonate is
reassessed. The fetus is still extremely floppy, pulse 45 bpm, no response to
stimulation and no spontaneous breathing. It is blue. What is the infants’s 10
minute apgar score?
a. 0
b. 1
c. 2
d. 3
e. 4
10. You see a 16 yo female who presented with primary amenorrhea. Breast
development wa noted at 13 years but there has been no increase in breast size.
Pubic and axillary hair was noted within one year of referral. An outside
ultrasound showed no uterus or ovaries physical exam reveals a normal vaginal
introitus with hymen present. Breasts are tanner 3, but seem to be more fatty
than mammary tissue. Lab test were significant for absent estradiol, elevated
gonadotropins and mildly elevated DHEAs with normal testosterone. A
karyotype was requested by the endocrinologist and was found to be 45, XY
(SRY gene +). The MRI showed an infantile uterus with no discernible gonas.
During a laparascopic evaluation, two dysplastic gonad attached to small
fallopian tubes and a rudimentary uterus was visualized. The final diagnosis is:
a. Swyer syndrome
b. Androgen insensitivity

REVISED BY TEAM “BANG JAGO” NOV 2020 UNHAS Page 141


c. Mullerian agenesis
d. Partial gonadal dysgenesis
e. Testicular regression syndrome
11. A 28 yo amenorrhoeic woman who wishes to become pregnant attends the
fertility clinic complaining of galactorrhea and mild visual disturbance. Her
serum prolactin level was found to be elevated. An MRI scan of the head is
performed, which showed the presence of a mocroprolactinoma, but without
suprecellar extension. What is the most appropriate first line management?
a. Bromocriptine
b. Cabergoline
c. Guinagolide
d. Radiotherapy
e. Trans spenoidal surgical excision of the prolactinoma
12. A 31 yo patient infertile for 4 years and prefferd for start in vitro fertilization
(IVF) because of obstructed fallopian tubes. On hysterosalpingogram (HSG), it is
noted that she has large dilated hydrosalpinges present bilaterally. What should
be your next step?
a. The patient should begin her IVF treatment cycle
b. The patient should repeat the HSG to confirm the result
c. The patient should not be offered the opportunity to have IVF
d. Her hydrosalpinges should be drained via transvaginal aspiration prior to starting IVF
e. Bilateral salpingectomies should be done prior to starting IVF
13. A 24 yo G1P0 at 13 weeks gestation who has sickle cell anemia requests genetic
counseling to learn about the risk of transmission to her fetus. If her partner is a
heterozygous carrier for this condition, what is the risk that their offspring will
be affected?
a. 25%
b. 50%
c. 75%

REVISED BY TEAM “BANG JAGO” NOV 2020 UNHAS Page 142


d. 80%
e. 100%
14. A 23 yo G2P0 at 25 weeks presents with a complaint of loss of fluid vaginally, the
fluid was copious and clear. Assessment by sterile speculum examination reveals
vaginal pooling; ferning was noted under the microscope and nitrazine paper
turned blue. She is very worried because she lost her first son about 4 weeks
after he was born at 26 weeks due to largely in part to severe repiratory
distress. To reassure her, you discuss using antenatal corticosteroids. Which of
the following is the most correct statement with respect to antenatal
corticosteroid?
a. They are only helpful if given at least 24 hours prior to delivery
b. Men have better outcomes than women
c. Maternal infections are increased
d. Surfactant alone is more effective than in combination with corticosteroids
e. They are less effective in cases of ruptured compared with intact membranes
15. A 41 yo primigravida presents to your office during the ninth week of pregnancy.
Because of her age, so they are extremely anxious and would like prenatal
diagnosis as early as possible. They have many questions about chorionic villus
sampling (CVS). You explain to the couple that complications oc CVS include
which of the following?
a. Amniotic band syndrome
b. Vaginal bleeding
c. Vater syndrome
d. IUGR
e. Rupture of the umbilical cord
16. A 33 yo G2P1 woman at 10 weeks gestation presents to clinic for routine
prenatal visit. Her pregnancy has been uncomplicated. Her prior pregnancy 2
years ago was complicated by the diagnosis of gestational hypertension that led
to an induction of labor. She has no other medical history. On examination her

REVISED BY TEAM “BANG JAGO” NOV 2020 UNHAS Page 143


BP is 138/88 mmHg, her Body mass index (BMI) is 37 kg/m, and a utine dip
shows trace protein. You counsel the patient on the pregnancy complication
associated with chronic hypertension as well aa management. What is the next
best step in her management?
a. Recommend termination of pregnancy
b. Start antihypertensive therapy
c. Bed rest
d. Hospitalization for futher maternal and fetal testing
e. Expectant management with close observation for early signs of preeclampisa and
fetal growth restriction
17. Mrs. Y, 37 yo, G2P1 37 wga desire VBAC (Vaginal Birth after caesarean section)
but she worries about its complication. Her baby is in a vertex presentation and
her previous low transverse procedure was for postterm and failed of induction.
In providing informed consent, in which of the following ways do you explain
the risk of uterine rupture?
a. Less than 1%
b. Between 2% and 5% - klo di soal post sc 2x
c. Dependent on the length of her labor
d. 15-20%
e. Dependent on the location and proximity of the scar site to the placental
implantation
18. A 27 yo G2P1 woman at 8 weeks GA comes to your clinic for an initial prenatal
visit. A medical history reveals a diagnosis of lupus 2 years ago. She is in normal
condition right now. What is the most significant neonatal complication of
maternal lupus?
a. Congenital abnormalities
b. Neonatal thrombosis
c. Neonatal heart block
d. Acute respiratory distress syndrome (ARDS)

REVISED BY TEAM “BANG JAGO” NOV 2020 UNHAS Page 144


e. Feeding difficulties
19. A 31 yo G4P0A3 woman presents to the hospital with vaginal bleeding and
abdominal pain. She appears pale and states that she feels lightheaded when
sitting up or standing. She reports that she is currently 8 weeks pregnant. On
arrival, her temperature is 37oC, BP is 80/50, pulse rate is 115 beats per minute,
and respiratory rate in 20 breaths/min. abdominal examination reveals a rigid
abdomen with rebound tenderness to pappation. Pelvic examination reveals a
small amount of vaginal bleeding, a 6 week size uterus, and fullness at the right
adnexa. A urine b-hCG confirms that she is pregnant. A pelvic ultrasound reveals
a right sided ecrtopic pregnancy as well as large amounts of fluid, thought to be
blood in the abdomen. She now has IV access and a bolus of IV fluids is being
given. Her BP is now 75/45 and her pulse rate is 120 beats/min/ her hematocrit
returns at 25.2%. how is the next management for this patient?
a. Proceed with emergency laparascopic salpingectomy
b. Proceed with emergency laparotomy
c. Administer IM methotrexate
d. Transfuse the patient with two units of packed RBCs and transfer her to the ICU
e. Start vasopressors and a transfer the patient to the ICU
20. A patient presents for her routine prenatal visit at 30 weeks EGA. Her pregnancy
up to now has been uncomplicated. Her BMI is 25. Her laboratory testing is
normal including a glucosechallenge screen. Anatomic ultrasound done at 12
weeks was normal and confirmed her dating. Her fundal height is 37 cm today. A
brief bedside ultrasound reveals an amniotic fluid index (AFI) of 30 cm. which of
the following situations is most likely to be the etiology of polyhydramnions in
this case?
a. Renal atresia
b. Pulmonary hyperplasia
c. Duodenal atresia
d. Gestational diabetes

REVISED BY TEAM “BANG JAGO” NOV 2020 UNHAS Page 145


e. Anencephaly
21. A 37 yo G6P3205 at 34 weeks of gestational age with gestational diabetes
moderately controlled with insulin presents for her scheduled obstetric (OB)
appointment. During the course of the visit, the patient notes that the fetus is
moving, but sleeps a lot. Her previous pregnancy was also complicated by
gestational diabetes and fetal macrosomia. Her bp is 138/85. Fetal heart tones
(FHTs) are in the 130s. fundal height measures 36 cm. her last ultrasound was at
28 weeks and showed a fetus in the 93rd percentile.
What is the next step in the management of this patient for this visit?
a. Schedule biweekly NSTs
b. Schedule a growth ultrasound
c. Draw Preeclampsia labs and send her to triage fpr evaluation
d. Reassure her and return to clinic in 2 weeks
e. Refer to a maternal fetal specialist for further management
22. Your patient is a 21 yo nulligravida currently desiring pregnancy and attempting
conception. Her last menstrual period was 6 weeks ago. She presents with
complaints of vaginal spotting. Urine b-hCG positive, vital signs are normal,
hematocrit is 36 volume percent. From sonogram[hic findings found the
trilaminar endometrial lining. Other findings include normal myometrium, a cul
de sac without free fluid, and normal adnexa. Her serum b-hCG level is 1200
miU/mL. she is planned to have another b-hCG measurement in 48 hours. She
returns in 48 hours with minimal right lower quadrant pain. Her vital signs are
normal, and spotting has decrease. Her clinical and laboratory findings are
unchanged except for a serum b-hCG level now measuring 2300 miU/mL. what
is the next best clinical step for this patient?
a. methotrexate
b. Repeat transvaginal sonography
c. Perform dilatation and curettage
d. Schedule repeat serum b-hCG level in 48 hours

REVISED BY TEAM “BANG JAGO” NOV 2020 UNHAS Page 146


e. Emergency laparotomy
23. A 30 yo woman has a twin pregnancy at 12 weeks gestation. During her initial
prenatal care visit, you review risks for multifetal pregnancies. Which of the
following statements reflects the most frequent risks in twin pregnancies?
a. cesarean delivery is necessary in all of twin deliveries
b. Shoulder dystocia occurs more in the aftercoming vertex twin, as compared to a
singleton
c. Pregnancy induced hypertension occurs at a highe rate than in singletons
d. Perinatal death rate is less than that of singletons
e. Congenital anomalies occur at the same rate as a singletons
24. A 30 yo G2P1 woman at 28 weeks GA comes to your office for a routine prenatal
visit. Her child was recently sent home with a rash and fever. She states that the
child had a rash on both cheeks and the pediatrician said it was a vira;infection
called fifth disease. Her baby is moving well and denies any vaginal bleeding,
abnormal vaginal discharge or contractions but she wonders if she needs any
more testing to see if she has been affected. What is the most likely causative
organism of the child’s infection?
a. Varicella
b. CMV
c. Parvovirus
d. Toxoplasmosis
e. Listeriosis
25. A pair of monochorionic twins presents at 21 weeks gestation with sonographic
findings that suggest twin-twin transfusion syndrome. There is significant
growth discordance, no bladder is visualized in the smaller twin, neither twin
has ascites or hydrops and umbilical doppler studies are normal. What would be
the assigned quintero stage?
a. Stage 1 – oligo dan poli
b. Stage 2 – no bladder

REVISED BY TEAM “BANG JAGO” NOV 2020 UNHAS Page 147


c. Stage 3 – doppler abnormal
d. Stage 4 - hydrops
e. Stage 5 - mati
26. You are providing prenatal care to a 24 yo G1P0 woman at 17 weeks GA by LMP.
She has had a relativelu normal pregnancy thus far. She has an obese BMI,
otherwise without medical or surgical history. She presented to prenatal care at
15 weeks, and so missed first trimester screening. She undergoes the quad
screen and has an elevated level of maternal serum alpha fetoprotein (MSAFP).
Given the elevation in MSAFP, her pregnancy is at increased risk for which of the
following?
a. Gestational diabetes
b. Gastrochisis
c. Down syndrome
d. Klinefelter syndrome
e. Potter syndrome
27. A 27 yo, 32 weeks gestation complains of shortness of breath during her
pregnancy, especially with physical exertion. She has no prior medical history.
Her respiratyory rate is 16x/m. lung auscultation are clear and her oxygen
saturation to be 98% on room air. You reassure her that this sensation is normal
and explain which of the following?
a. Because of enlarging uterus pushing up on the diaphragm, her vital capacity is
decrease by 20%
b. Airway conductance is decrease during pregnancy
c. Pulmonary resistance increases during pregnancy
d. Small amniotic fluid emboli are shed throughout pregnancy
e. Maximal breathing capacity is not altered by pregnancy
28. A G3P2A0 woman at 35 weeks is seen in your office for her prenatal visit. She is
concerned because she has not felt her babu moving as much as she used to.
Her pregnancy has been uncomplicated and her past two pregnancies ended in

REVISED BY TEAM “BANG JAGO” NOV 2020 UNHAS Page 148


full term, normal spontaneous vaginal deliveries. When forntal antenatal testing
is done, which of the following Is most reassuring?
a. Late decelerations on fetal monitoring
b. A CST with variable FHR decelerations with contractions, but moderate variability
c. A NST with two accelerations of the FHR in 20 min that are at least 15 beats above
baseline and last for at least 15 seconds
d. An increase in the systolic to diastolic ratio in the umbilical artery blood flow
e. A score of 6 on a BPP
29. A 24 yo woman presents to clinic with an atypical squamous cells of
undertermined significance (ASCUS) pap she had a pap about 2 to 3 years ago.
And it was normal. She became sexually active at age 17 and has had two
lifetime partners. She and her male partner have been together for over a year.
She started having periods at age 13 and has regular cycles on her own. Her last
period was 1 week ago on oral contraceptives. She has a history of herpes
simplex virus well controlled with daily acyclovir. Her physical examination is
normal. What is the next step?
a. Repeat the pap
b. Get high risk HPV typing – soal Shelf Life
c. Get low risk HPV typing
d. Perform colposcopy
e. Perform a conization
30. A 26 yo P2A0 presents for her annual gynecologic examination and would like to
discuss her risk of ovariu cancer. Menarche occurred at age 14. She used oral
contraceptive pills for 3 years prior to the birth of her first child when she was
23. She breastfed both of her children for 1 years each. Her mother is 46 and
currently is undergoing chemotherapy for ovarian cancer, and she had a
maternal aunt who passed away in her 50 from breast cancer. What is this
woman’s greatest potential risk factor for developing ovarian cancer?

REVISED BY TEAM “BANG JAGO” NOV 2020 UNHAS Page 149


a. BRCA (breast cancer gene) mutation – ada tipe soal lain History of breast cancer
jawabannya
b. Hereditary nonpolyposis colorectal cancer (HNPCC) mutation – Lynch syndrome
c. Family history
d. Early menarche
e. Late childbearing
31. A 27 yo, P1, woman comes to see you to discuss a recent decrease in sexual
desire. The patient is in a 7 year monogamous sexual relationship with her
husband and states that otherwise they have a great relationship. You start by
discussing the 4 phases of the femae sexual response; desire, arousal, orgasm,
and resolution.
What is the most likely problem noted in evaluating sexual dysfunction?
a. The sequence stated above is out of order
b. Ther is a missing step in the sequence
c. There is a lack of satisfaction with the sexual encounter
d. The patient reports pain with intercourse
32. An 18 yo nulligravid woman presents to the student health clinic with a 4 week
history of yellow vaginal discharge. She also reports vulvar itching and irritation.
She is sexually active and monogamous with her boyfriend. They use condoms
inconsistently. On physical examination, vulvar and vaginal erythema is noted
along with ayellow, frithy, malodorous discharge with a pH of 6.5. the cervix
appears to have erythematous punctuations. There is no cervical, uterine, or
adnexal tenderness. The addition of 10% KOH to the vaginal discharge does not
produce an amine odor. Wet prep microscopic examination of tge vaginal swabs
is perfomed. What would you ecpect to see under microscopy?
a. Branching hyphae
b. Multinucleated giant cells
c. Scant WBC
d. Flagellated motile organisms

REVISED BY TEAM “BANG JAGO” NOV 2020 UNHAS Page 150


e. Epithelial cells coverd with bacteri
33. A 36 yo G3P2 woman at 39 weeks and 4 days was admitted to labor and delivery
unit for active management of labor. After her membranes has ruptured, she is
very uncomfortable and finds her contractions very painful. You perform cervix
examination is at 6 cm. which of the following presentations and positions
would you expected be most favorable to achieve a vaginal delivery?
a. Vertex with occiput anterior
b. Vertex with occiput posterior
c. Vertex with occiput transverse
d. Breech with sacrum anterior
e. Breech with sacrum posterior
34. A 41 yo P2 woman comes to see you with a complaint of 14 months of irregular
bleeding =. She previously had regular menses but over the past 2 years they
have become totally unpredictable. The doctor said that it might be caused by
hormonal imbalance. Which of the following hormones can have a negative as
well as a positive feedback at the level of the pituitary?
a. Cortisol
b. Oxytosin
c. Estradiol
d. Progesterone
e. Insulin
35. A 23 yo G1 woman at 8 weeks GA comes to your clinic for an initial prenatal
visit. She had a history of cardiac disease since 5 years ago, but nor she didn’t
have any complaint and not taking any medication since 1 year ago, but nor she
didn’t have any complaint and not taking any medication since 1 year ago. She
asks if she could continue her pregnancy, since many people said that pregnancy
could put her in danger. You explain that it may be due to the increase of blood
volume and cardiac changes in pregnancy. The increase in blood volume in
normal pregnancy is made up of which of the following?

REVISED BY TEAM “BANG JAGO” NOV 2020 UNHAS Page 151


a. Erythrocytes
b. More erythrocytes than plasma
c. More plasma than erythrocytes
d. Neither plasma nor erythrocytes
e. Plasma only
36. A woman presents for her new obstetrical vist at 12 weeks EGA. Her medical
history is complicated by graves thyroiditis that has been treated with
radioactive iodine a few years prior. The patient is currently being maintained
on thyroid replacement. She is worried that this will compromise the fetus. She
is told that the interaction between maternal and fetal physiology relative to
thyroid function is complex. Which of the following is an accurate description of
this interaction?
a. Maternal thyroid hormones (T4 and T3) readily cross the placenta
b. Maternal thyrotropin easily crosses the placenta
c. The athyroid fetus is growth retarded at birth
d. The fetal thyroid concentrates iodide
e. The placenta serves as a barrier to maternal iodine crossing to the fetus
37. A 26 yo G3P2A0 come to your clinic for oreconceotion counselling. She has a
medical history of significant fo igA glomerulonephritis. Creatinine result is 2.0
she is otherwise helathy. Which of the following you tell her to except regarding
her kidney function if she were to get pregnant?
a. It will stay the same
b. It will improve
c. She will need kidney transplant in order to have a successful pregnancy
d. It will get worse
e. It is unlikely she will need dialysis bt the end of pregnancy
38. You are asked to see a 28 yo woman who is day 2 following a cesarean section
for unsuccessfull induction of labour at 27 weeks gestation. She was diagnosed
with mild preeclampisa at 36 weeks gestation. She was diagnosed with mild

REVISED BY TEAM “BANG JAGO” NOV 2020 UNHAS Page 152


preeclampisa at 36 weeks gestation. Her blood preasure has been 155/100
mmHg on two occasions today, although she is asymptomatic and her deep
tendon reflexes are normal. She is breast feeding. You decide to start her on the
antihypertensive agent. Which of the following antihypertensive agents has
insufficient evidence on infant safety to recommend or use in breast feeding
mothers?
a. Amlodipine
b. Atenolol
c. Captopril
d. Enalapril
e. Metoprolol
39. A 28 yo G1P0A0 woman at 38 weeks and 5 days presents to labor and delivery
unit, with regular contractions occurring 3-4x/10 minutes. Her contractions last
30 to 90 seconds. On vaginal examinations you found 10 cm dilatation and now
in the second stage of labor. She is pushing effectively, but during contractions
you notice decelerations on fetal heart tracings. Which of the following would
be most dangerous?
a. Absence of decelerations
b. Isolated early decelerations
c. Repetitive variable decelerations that resolve quickly after each contraction
d. Repetitive early decelerations and variable decelerations
e. repetitive late decelerations and loss of variability between contractions
40. After a prolonged secod stage, a 28 yo woman delivers the vertex with an
immediate turtle sign with the head retracting against the perineum. Mc
Robbert’s maneuver does not affect delivery. Which of the following would be a
helpful meneuver in managing this shoulder dystocia?
a. Fundal pressure
b. Internal podalic version
c. Increased maternal pushing effort

REVISED BY TEAM “BANG JAGO” NOV 2020 UNHAS Page 153


d. Wood’s crew maneuver
e. Ritgen maneuver
41. A 36 yo G2P1 with one prior cesarean delivery presents at 36 weeks gestation
with active vaginal bleeding and now requires emergency cesarean hysterctomy
due to placenta previa with accrete. Compared with patients who have elective
surgery, this woman is at increased risk or which of the following complications?
a. Bowel injury
b. Urinary tract injury
c. Venous thromboembolism
d. External iliac vessels injury
e. Hypogastric nerve injury
42. A 30 yo G1P0 woman at 40 weeks and 4 day presents to labor and delivery unit
with second stage of labor. She pushes the head to the perineum and you
deliver the bay without complication. You examine her for lacerations.
Classification of perineal tear caused by either tearing or episiotomy in which
injury to the perineum involves less than 50% of the external anal spincter
thickness torn, is classified as
a. Second degree
b. Third A degree (3A)
c. Third B degree (3B)
d. Third C degree (3c)
e. Fourth degree
43. A 28 yo G1P0 woman at 39 weeks and 6 days presnts to labor and delivery unit,
with regular contractions occurring every 3 minutes. Her contractions last 30 to
50 seconds. She not sure is she’s been leaking any fluid from her vagina. You
take her history and conduct a physical examination. Rupture of membrane
would be best supported by which of the following?
a. Nitrazine paper remaining orange when exposed to fluid in the vagina
b. A negative fern test

REVISED BY TEAM “BANG JAGO” NOV 2020 UNHAS Page 154


c. An ultrasound with a normal AFI
d. A negative tampon test
e. Speculum examination with evidence of pooling in the vagina
44. A 22 yo primiparous woman develops postpartum hemorrhage unresponsive to
oxytocin and uterine massage. The uterine contraction was good. Her infant
was 3800 grams. She has bled 750 cc. what is the most possible diagnosis?
a. Laceration of cervix or vagina
b. Placenta accrete
c. Uterine inversion
d. Ruptured uterus
e. Coagulopathy
45. A 38 yo Asian mother has delivered her fourth baby normally. She is a known
type 2 diabetic and was taking methformin prior to pregnancy for glycemic
control. From 32 weeks gestation, isophane insulin was added twice daily in
addition to methformin to achive glycemic control. The woman is planning to
breast feed. What advise should be given regard to a hypoglycemic agent in the
postnatal period?
a. Continue all the medications for the first 24 hours after delivery and then resume
methformin as per prepregnancy with monitoring of blood sugar
b. Step all medications and follow diet control with monitoring of blood sugar
c. Stop insulin and advise methformin as per prepregnancy with monitoring of blood
sugar
d. Stop methformin and continue isophane insulin twice daily until breast feeding has
stopped
e. Stop methformin and continue isophane insulin at half the dose used during
pregnancy until breastfeeding is stopped
46. A 28 yo nulliparous patient attended for routine cervical screening which has
shown borderline changes and HPV inadequate results. She is not currently

REVISED BY TEAM “BANG JAGO” NOV 2020 UNHAS Page 155


sexually active and has had normal smear results before this test. What is the
next appropriate step in management?
a. Refer to colposcopy
b. Repeat cytology only in 3 months
c. Repeat cytology and HPV in 3 months
d. Repeat cytology and HPV in 6 months
e. Repeat cytology and HPV in 12 months
47. An 18 yo nulliparous patient attends the gynaecology clinic for haevypainful
menstrual bleeding. She is in a sexual relationship with a new partner and both
have tested negative dor STIs, though she has had a previously treated
chlamydia infection. She does not want any children in the near future. She
taken lamotrigine for epilepsy control and has not suffered any seizures for 3
years. General and pelvic examinations are normal, and her BMI is 30. What is
the best appropriate method of contraception in her situation?
a. Combined oral contraceptive pill
b. Depot medrocyprogesterone acetate
c. Combined estrogen and progestin patch
d. Levonogetrel releasing intrauterine system
e. Ethynilestradiol ring
48. A 28 yo P2 woman comes to the emergency department with 6 hours of
increasing left lower quadrant pain, inability to tolerate orals, and nausea. She is
sexually active and uses depo provera for contraception. She was treated for
gonorrhea and reports compliance with treatment. Her temperature is 38.5oC,
Pulse rate 112x/min, respirations 22x/m, and BP is 100/70 mmHg. On
examination, her abdomen is soft with leftlower quadrant tenderness.
Woluntary guarding is present without rebound. Pelvic examination shows no
cervical motion tenderness or uterine tenderness. The left adnexa is exquisitely
tender and fullness is appreciated. Her WBC count is 21000 cells/µL. urine HCG

REVISED BY TEAM “BANG JAGO” NOV 2020 UNHAS Page 156


is negative. Which of the following is the most appropriate next step in
mamagement?
a. Pelvic ultrasound
b. Abdominal X ray
c. Measurement of serum b-hCG concentration
d. Dilatation and curettage
e. Discharge home on oral antibiotics
49. A 30 year old recently diagnosed with acquiuured immunodeficiency syndrome
(AIDS). She has not consumeany medicationsyet. She presents with genital
lessions that have been present for 1 week. She reports having had a single sore
4 months ago that was not painful and subsided without treatment after
approximately 2 weeks. Which of the following test specific to these lessions do
you expect to be positive?
a. PCR
b. Wright-giemsa stain
c. Herpex simplex antibody assay
d. Psiitive skin punch biopsy staining for candida
e. VDRL – RPR
50. A 45 yo patient with prolonged menstruation and dysmenorrhea from pelvic
examination found enlarged uterus, both adnexa within normal limit. Which
study listed below would best differentiate between adenomyosis and uterine
fibroids?
a. Pelvic ultrasound
b. Pelvic MRI
c. Pelvic CT
d. Sonohysterogram
e. Hysterosalpingogram

REVISED BY TEAM “BANG JAGO” NOV 2020 UNHAS Page 157


51. a 27 yo nulligravida presents with 6 month of amenorrhea and diagnosed with
hyperprolactinemia. Laboratory test measuring which of the following should
also be obtained?
a. Total testosterone
b. 24 hour urinary free cortisol
c. Insulin like growth factor II
d. Thyroid stimulating hormone
e. Oxytocin
52. A 51 year old woman presents to your office due to amenorrhea for the past
year but she denied had any hot flashes, she is wondering if she is menopausal.
She asking you how to make sure, what blood test would confirm the diagnosis
of menopause?
a. Progesterone
b. Testosterone
c. FSH
d. hCG
e. Prolactin

AGUSTUS 2018

An 18-year-old young woman presents to your office with a complaint of amenorrhea.


She notes that she has never had a menstrual period, but that she has mild cyclic
abdominal bloating. She reports normal breast development starting at age 12. She
reports she has become sexually active, but she finds intercourse painful. Her past
medical and surgical history is unremarkable. On physical examination, you note
normal apprearing axillary and pubic hair. Her breast development is normal. Pelvic
examination reveals normal appearing external genitalia, and a shortened vagina
ending in a blind pouch.
REVISED BY TEAM “BANG JAGO” NOV 2020 UNHAS Page 158
1. Which of the following tests would be your first step in determining the diagnosis?
a. Karyotype
b. Pelvic ultrasound
c. Serum FSH
d. Diagnostic laparoscopy

2. You perform a bedside ultrasound and find normal appearing bilateral ovaries as well
as an absent uterus and fallopian tubes. What is your most likely diagnosis?
a. Imperforate hymen
b. Transverse vaginal septum
c. Müllerian agenesis
d. Androgen insensitivity syndrome

3. Which additional organ system should you be evaluating in a patient with this
disorder?
a. Pancreas and duodenum
b. Cerebral circulation
c. Olfactory system
d. Renal and urinary collecting system

4. Your patient’s concerned mother calls you later that afternoon. “How will my daughter
be able to start a family?” You wisely counsel her about her options, which include:
a. Intrauterine insemination
b. Uterine transplant
c. In vitro fertilization with gestational carrier
d. The patient will not be able to reproduce using her genetic material
A 23-year-old G1P0 woman at 30 weeks 3 days presents to clinic for routine prenatal
care. Her pregnancy is dated by last menstrual period (LMP) consistent with 10 weeks
ultrasound. She has had three prenatal visits, and her pregnancy has been complicated
by vaginal bleeding in the first trimester and the development of heartburn at 25
weeks. She has no complaints today. She continues to smoke half a pack of cigarettes
daily, which has decreased from one pack per day at the beginning of her pregnancy.
Her medical history is significant for asthma. Ultrasound at 20 weeks’ gestation
showed no evidence of fetal abnormality, posterior placenta, AFI of 10.6 cm, and fetal
growth in the 20th percentile. Her current weight is 130 lbs, and her height is 5 ft 6 in.
She has gained 10 lbs so far in pregnancy. Urine dip is negative for protein, glucose,
ketones, and leukocytes. BP is 112/64 mm Hg and heart rate is 80 beats per minute.
Fetal heart tones are in the 130s. Fundal height measures 25 weeks. Of note, at her
last visit at 25 weeks, fundal height measured 23 weeks, and she had a normal glucose
tolerance test and complete blood count.

REVISED BY TEAM “BANG JAGO” NOV 2020 UNHAS Page 159


5. Which of the following is the next best step in managing this patient’s pregnancy?
a. Nonstress test (NST)
b. Fetal ultrasound
c. Group B streptococcus culture
d. TORCH titers
e. Amniocentesis

6. Which of the following is the most likely risk of factor for this patient’s small for
gestational age (SGA) fetus?
a. Congenital anomaly such as cardiac anomaly
b. Congenital cytomegalovirus (CMV) infection
c. Tobacco abuse
d. History of chemotherapy exposure as a child
e. Genetic potential

7. Ultrasound demonstrates that the fetus measures less than the 10th percentile for
head circumference, femur length, and abdominal circumference. Doppler velocimetry
of the fetal umbilical artery was normal. Amniotic fluid index (AFI) was 11.2 cm. Which
of the following is the most appropriate component of the treatment strategy at this
time?
a. Induction of labor
b. Continue routine prenatal care
c. Fetal ultrasound every 2 to 3 weeks
d. Admission to the hospital for daily NST and biophysical profile (BPP)
e. Daily Doppler velocimetry

8. The patient has a repeat ultrasound performed at 33 weeks of gestation. Fetal growth
is noted to be at the 4th percentile with intermittently elevated umbilical cord
Dopplers and an AFI of 8.4 cm. What do yo recommend at this time?
a. Repeat growth ultrasound in 4 to 6 weeks
b. Amniocentesis for fetal lung maturity and delivery if mature
c. Admission to hospital for continuous fetal monitoring until delivery
d. Induction of labor
e. Betamethasone administration

9. What neonatal outcome must be anticipated in macrosomic fetuses?


a. Hypoglycemia
b. Hyperglycemia
c. Hypocalcemia
d. Asthma
e. None of the above

REVISED BY TEAM “BANG JAGO” NOV 2020 UNHAS Page 160


A 35-year-old woman with a history of primary infertility presents with her partnes for
initial prenatal visit. The had been attempting pregnancy for the past 3 years and have
now conceived through IVF. She had one embryo transferred 7 weeks ago. The
patient’s medical history is significant for rheumatoid arthritis and polycystic ovarian
syndrome. She is overweight. In viewing their family history, she tells you that her
grandmother was a twin and there is no family history of congenital abnormalities or
known genetic disorders in their family. Today, she reports nausea and vomiting, which
is worse in the morning. She has not had any vaginal bleeding or cramping. You
perform a transvaginal ultrasound and notice not one but two embryos. There appears
to be a thin dividing membrane between the two. Heart rate for each embryo is
around 150 beats per minute.
10. At what stage of division in the embryonic disk does monochorionic-diamniotic
twinning occur?
a. Before the differentiation of the trophoblast
b. After trophoblast differentiation and before amnion formation
c. After amnion formation
d. Not until after day 15 of development
e. At the stage of blastocyst

11. Postterm pregnancy is associated with all the following except...


a. Transient tachypnea of the newborn
b. Oligohydramnions
c. Macrosomia
d. Meconium aspiration
e. Intrauterine fetal demise
A 32 years old woman, G3P2 37 wga, came to ER with contractions 5 minutes apart. On
examination you found that the fetus is breech, and she’s already fully dilated with
breech is on hodge IV. You asked to mother to bear down, after umbilicus, the baby
won’t decent even after 2 times bearing down, you found nuchal arm.
12. Which is the maneuver to help release the nuchal arm?
a. Classic maneuver
b. Mauriceau-smellie-veit maneuver
c. Lovsett maneuver
d. Bracht maneuver
e. Muller maneuver

13. If aftercoming head happened, what should we do?


a. Naegel forceps
b. Downward traction

REVISED BY TEAM “BANG JAGO” NOV 2020 UNHAS Page 161


c. Lovsett maneuver
d. Modified Prague maneuver
e. Hyperextension of fetal head

14. Lung surfactant is critical to pulmonary functioning by keeping surface tension in the
alveoli low and thereby decreasing the occurence of atelectasis and atrioventricular
(AV) shunting. Surfactant is formed in which of the following?
a. Epithelium of the respiratory bronchi
b. Hilum of the lung
c. Placental syncytiotrophoblasts
d. Type I pneumocytes of the lung alveoli
e. Type II pneumocytes of the lung alveoli

15. A 33-year-old patient has been diagnosed as having adenomyosis. Which of the
following symptoms is most consistent with this diagnosis?
a. Dyspareunia
b. Mood swings
c. Painful defecation
d. Secondary dysmenorrhea
e. Infertility

16. A 53-year-old woman is diagnosed with anovulatory dysfunctional bleeding. Which of


the following is the most appropriate medical therapy?
a. Orally administered estrogen for the first 25 days of each month
b. Vaginal estrogen cream two to three times per week
c. Orally administered progesterone 5 to 10 mg daily for 10 days each month
d. Testosterone tablets 10 mg/d
e. Estrogen 20 mg administered intravenously

17. When performing a hysterectomy, the surgeon must be aware that at its closest
portion to the cervix, the ureter is normally separated from the cervix by which of the
following distances?
a. 0.5 mm
b. 1.2 mm
c. 12 mm
d. 3 cm
e. 5 cm
A 32-year-old G3P2002 woman presents for routine prenatal care at 37 weeks. Her
pregnancy is complicated by Rh-negative status, depression, and a history of LSIL Pap
smear with normal colposcopy in the first trimester. Today she reports good fetal

REVISED BY TEAM “BANG JAGO” NOV 2020 UNHAS Page 162


movement and denies leaking fluid or contractions. During your examination you
measure the fundal height at an appropriate 37 weeks, and find fetal heart tones
located in the upper aspect of uterus. A bedside ultrasound reveals frank breech
presentation.
18. Which of the following is the next step in management of this patient?
a. Schedule a cesarean delivery at 39 weeks
b. Return visit in 1 week to reassess fetal position
c. Schedule an external cephalic version
d. Offer a trial of vaginal breech delivery
e. Offer emergency cesarean delivery

19. Prior to discharging the patient from labor and delivery triage after her successful
external cephalic version, which of the following should you do first?
a. Schedule induction for 39 weeks
b. Palce abdominal binder to help hold fetus in cephalic presentation
c. Prescribe tocolytic
d. Give RhoGAM
e. Check fetal position with ultrasound

20. Which of the following findings would deter you form offering this patients a trial of
breech delivery?
a. Frank breech presentation
b. Fetal weight of 3200 g
c. Complete breech presentation
d. Fetal weight of 4100 g
e. Footling presentation
A 31-year-old G1P0 woman at 39 weeks and 4 days presents to labor and delivery unit,
with regular contractions occuring every 3 to 5 minutes. Her contraction last 30 to 90
seconds. She not sure if she’s been leaking any fluid from her vagina. You take her
history and conduct a physical examination.
21. Preterm rupture of the membranes is most strictly defined as spontaneous rupture at
any time prior to which of the following?
a. A stage of fetal viability
b. Second stage of labor
c. 32 weeks of gestation
d. 37 weeks of gestation
e. Onset of labor

REVISED BY TEAM “BANG JAGO” NOV 2020 UNHAS Page 163


22. You determine her membranes have ruptures and admit her for active management of
labor. The first stage of labor...
a. Includes an active and latent phase
b. Begins when the cervix has completely dilated
c. Is considered prolonged if its duration is longer than 2 hours in nulliparous woman
d. Begins with the onset of Braxton Hicks contractions
e. Is commonly associated with repetitive early and variable decelerations

23. On examination you attempt to determine the presentation of the fetus. Which of the
following presentations and position would be the most favorable to achieve vaginal
delivery?
a. Breech
b. Transverse
c. Vertex with occiput posterior
d. Vertex with occiput anterior
e. Vertex with occiput transverse

24. The patient is 5 cm dilated with regular contractions. However on the CTG you find
several variable decelerations. What is pathophysiology of variable decelerations?
a. Normal labor
b. Head compression
c. Cord compression
d. Maternal chronic anemia
e. Uteroplacental insufficiency

25. She pushes the head to the perineum and you deliver the head and the shoulders
without complication. The cord is clamped and the placenta delivered. You examined
her for lacerations. A second degree laceration...
a. Involves the anal mucosa
b. Is commonly associated with buttonhole lacerations
c. Involves the mucosa or the skin only
d. Will heal well without repair
e. Extends into the perineal body, but does not involve the anal sphingter
You are providing prenatal care to a 22-year-old G1P0 woman at 16 weeks GA by LMP.
She has had a relatively smooth pregnancy without complication thus far. At 5950 and
215 lb she has obese BMI, otherwise without medical or surgical history. She
presented to prenatal care at 14 weeks and so missed first-trimester screening. She
undergoes the quad screen and has an elevated level of maternal serum alpha-
fetoprotein (MSAFP).

REVISED BY TEAM “BANG JAGO” NOV 2020 UNHAS Page 164


26. Given the elevation in MSAFP, her pregnancy is at increased risk for which of the
following?
a. Gestational diabetes
b. Gastroschisis
c. Down syndrome
d. Klinefelter syndrome
e. All of the above

27. You discuss the potential meaning of the elevated MSAFP. After a long conversation,
the patient decides to undergo her second trimester ultrasound. The ultrasound
reveals a myelomeningocele. Which of the following is true and may be used in
counseling?
a. This is generally a lethal anomaly
b. Delivery must be by cesarean to protect the baby
c. Fetal surgery includes laser therapy
d. Fetal surgery has been shown to improve some outcomes
e. Fetal surgery is experimental and has no known benefits

28. The increased incidence of this finding is associated with which of the following
medications when used in pregnancy?
a. Valproic acid
b. Lithium
c. Fluoxetine
d. Prednisone
e. Acetaminophen

29. In a subsequent pregnancy, prevention of reccurence would include:


a. Low-dose aspirin
b. Low molecular weight heparin
c. Prenatal vitamin taken twice per day
d. 4 mg folic acid
e. Increased dietary calcium
A 32-year-old woman is 36 weeks pregnant in first pregnancy with DCDA (dichorionic-
diamniotic) twins and is being reviewed in the antenatal clinic. A recent obstetric
growth scan confirm both fetuses are normally grown. Both twins are longitudinal lie,
and cephalic presentation. She has no other complicating medical or obstetric
disorders. She is deciding between planned vaginal or elective cesarean modes of
delivery.
30. Which of of the following is correct in relation to the counseling she will receive?
a. About 10% of twin pregnancies result in spontaneous birth before 37 weeks

REVISED BY TEAM “BANG JAGO” NOV 2020 UNHAS Page 165


b. Continuing twin pregnancies beyond 38 weeks increases the risk of fetal death
c. Maternal antenatal corticosteroids are routinely recommended for all twin
pregnancies
d. Offer elective birth from 37 weeks after a course of maternal corticosteroids has been
administered
e. There is strong evidence to show cesarean delivery is safer for mother an fetuses than
vaginal mode of delivery

31. Which of the following statements about twinning is true?


a. The frequencies of monozygosity and dizygosity are the same
b. Division after formations of the embryonic disk results in conjoined twins
c. The incidence of monozygotic twinning varies with race
d. A dichorionic twin pregnancy always denotes dizygosity
e. Twinning causes no appreciable increase in maternal morbidity and mortality over
singleton pregnancy

32. You are following a 38-year-old G2P1 at 39 weeks in labor. She has had one prior
vaginal delivery of a 3800 g infant. One week ago, the estimated fetal weight was 3200
g by ultrasound. Over the past 3 hours her cervical examination remains unchanged at
6 cm. Fetal heart rate tracing is reactive. An intrauterine pressure catheter (IUPC)
reveals two contractions in 10 minutes with amplitude of 40 mm Hg each. Which of the
following is the best management for this patient?
a. Ambulation
b. Sedation
c. Administration of oxytocin
d. Cesarean section
e. Expectant

33. A primipara is in labor and an episiotomy is about to be cut. Compared with midline
episiotomy, which of the following is an advantage of mediolateral episiotomy?
a. Ease of repair
b. Fewer breakdown
c. Less blood loss
d. Less dyspareunia
e. Less extension of the incision

34. A 24-year-old primigravid woman, at term, has been in labor for 16 hours and has been
dilated 8 cm for 3 hours. The fetal vertex is in the right occiput posterior at +1 station
and molded. There have been mild late decelerations for the past 30 minutes. Twenty
minutes ago, the fetal scalp pH was 7.27, it is now 7.20.
For above clinical description, select the most appropriate procedure.

REVISED BY TEAM “BANG JAGO” NOV 2020 UNHAS Page 166


a. External version
b. Internal version
c. Midforceps rotation
d. Low transverse cesarean section
e. Classic cesarean section

35. Variable decelerations are caused by which fetal situation?


a. Hypoxia
b. Acidemia
c. Cord compression
d. Head compression

36. With increasing gestational age, the fetal heart rate baseline undergoes which of the
following trends?
a. Increases
b. Remains unchanged
c. Variability changes become more closely tied to activity changes
d. Decreases

37. Fetal bradycardia typically may result from which of the following?
a. Maternal fever
b. Fetal head compression
c. Maternal atropine use
d. None of the above

38. A 20-year-old G1 at 38 weeks gestation presents with regular painful contractions


every 3 to 4 minutes lasting 60 seconds. On pelvic examination, she is 3 cm dilated and
90% effaced; an amniotomy is performed and clear fluid is noted. The patient receives
epidural analgesia for pain management. The fetal heart rate tracing is reactive. One
hour later on repeat examination, her cervix is 5 cm dilated and 100% effaced. Which
of the following is the best next step in her management?
a. Begin pushing
b. Initiate Pitocin augmentation for protracted labor
c. No intervention labor is progressing normally
d. Perform cesarean delivery for inadequate cervical effacement
e. Stop epidural infusion to enhance contractions and cervical change

A 63-year-old G4P4 woman presents to your office with a chief complaint of vaginal
spotting. She reports an isolated episode 1 week prior to presentation that consisted of
scant vaginal bleeding. She denies any associated symptoms including pelvic pain,

REVISED BY TEAM “BANG JAGO” NOV 2020 UNHAS Page 167


pressure, or early satiety. She also denies any family history of gynecologic malignancy.
Her past medical history is significant for morbid obesity, hypertension, and
inflammatory bowel disease.
39. What is the most likely diagnosis?
a. Atrophic endometrium
b. Endometrial cancer
c. Endometrial polyp
d. Ovarian cancer
e. Adenomyosis

40. After obtaining a through history and performing a physical examination (including a
pelvic exam), what is the next best step in evaluation?
a. CA 125
b. MRI
c. Cervical cytology
d. Transvaginal ultrasound and possible endometrial biopsy
e. Mammography

41. What is the patient’s most significant risk factor for endometrial cancer?
a. Multiparity
b. History of prior tobacco use
c. Remote history of oral contraceptive use
d. Inflammatory bowel disease
e. Morbid obesity

A 16-year-old girl presents to the physician for annual examination. She denies any
current symptoms or concerns. She has been sexually active for 1 year and is using
Depo-Provera for contraception. She does not use condoms. On urogenital
examination, she has a moderated amount of yellow mucopurulent discharge from
endocervix. There is no cervical motion, adnexal, or uterine tenderness. Microscopy of
vaginal discharge was normal except greater than 10 WBCs per high power field.
Vaginal pH is normal.
42. What is the most appropriate next step in the management?
a. Perform a Pap smear with reflex HPV testing
b. Order a pelvic ultrasound
c. Trat for presumptive chlamydial infection and gonorrhea
d. Send vaginal discharge for culture and Gram stain
e. Perform nucleic acid amplification test (NAAT) for chlamydial infection and gonorrhea

REVISED BY TEAM “BANG JAGO” NOV 2020 UNHAS Page 168


43. Test results confirm the diagnosis of gonorrhea. What is the best initial treatment for
this patient?
a. Ceftriaxone 125 mg IM once
b. Ceftriaxone 125 mg IM once plus azithromycin 1 g orally once
c. Doxycycline 100 mg orally twice daily for 7 days
d. Ciprofloxacin 500 mg orally once
e. Clindamycin 300 mg orally twice times daily for 7 days
A 22-year-old G3P1021 woman recently delivered and is now attempting to breastfeed
her baby. Her pregnancy and delivery were uncomplicated. She denies any medical
history for social history significant for drug use. She is frustrated by her lack of
volume, worried that her son will not gain weight, and is now requesting a bottle and
prepared formula.
44. Which of the following statements about the benefits of breastfeeding is false?
a. Breastfed children are more resistant to disease and infection early in life than
formula-fed children
b. Breastfeeding women have a lower risk of breast, uterine, and ovarian cancer if they
have breastfed for at least 2 years cumulatively
c. Children who are breastfed are significantly less likely to become obese later in
childhood
d. Oxytocin released during breastfeeding causes the uterus to return to its normal size
more quickly
e. None. All of the above are true statements

45. Which of the following would be an appropriate form of contraception for this
breastfeeding patient?
a. Progesterone-eluting intrauterine device (IUD)
b. Combined oral contraceptive pills
c. Contraceptive vaginal rings
d. Contraceptive patch
e. None of the above

46. Your patient is admitted to the hospital 2 weeks later with rigors and chills and
complaint of a swollen and reddened right breast. She has been breastfeeding
throughout the last 2 weeks. Her vitals are significant for a fever up to 38.4˚C and
tachycardia with pulse of 112; all other vital signs are normal. Her physical examination
is significant for cracked nipples and engorged breasts bilaterally; her right breast is
particularly tense, notable for erythema and increased temperature compared with
the left breast without masses. What is the appropriate therapy for the above
condition?

REVISED BY TEAM “BANG JAGO” NOV 2020 UNHAS Page 169


a. Dicloxacillin 3, 10 to 14 days
b. Dicloxacillin until afebrile for 48 hours
c. Reassurance, ice, breast support, and breast pumping
d. Protective nipple shields and soothing ointments
e. Ultrasound-guided localization of abscess and aspiration
You are seeing a 20-year-old single G1P1 woman who is postpartum day 2 after a
normal spontaneous vaginal delivery of a healthy female infant. Her pregnancy and
delivery were uncomplicated. She is notable for being teary and anxious when you
begin providing her discharge instructions. She explains that she has been unable to
get any sleep between her baby crying, breastfeeding every 3 hours, and her constant
worries about whether she will be able to handle a baby at home by herself. She is
particularly bothered by pervasive thoughts that her daughter could roll onto her
stomach and be unable to breathe or that she could choke while breastfeeding without
her mother recognizing it because of her inexperience and sleep deprivation. She is
concerned about the overwhelming responsibility of raising a baby by herself, and that
she may never be able to go back to school; she expresses that “this may all have been
a bad idea.” She denies any history of depression.
47. Which of the following is the most important next step in evaluating the patient?
a. Reassure the patient that she is likely experiencing a common condition called “the
baby blues”
b. Contact the father of the baby to ensure that the patient will have an alternative
source of childcare when she needs to care for herself
c. Offer a prescription sleep aid to help the patient get a full night’s rest
d. Prescribe the patient an SSRI for new diagnosis of postpartum depression
e. Tell the patient that she likeluy has postpartum depression and should be seen by a
counselor while in the hospital

48. The patient sees you again 6 weeks later for her postpartum appointment and still
reports difficulty coping with her new baby. She is still having difficulty with sleep but is
now unable to stay asleep even when the baby has been sound asleep. She has been
avoiding phone calls from her friends because she does not want them to see her in
this state. She has a limited appetite, decreased interest in her normal sources of
entertainment, and she reports just generally being sad since the baby was born.
Although she has taken her baby to the pediatrician as needed and notes interval
weight gain, she reports having ignored her crying baby on more than one occasion
over the last few weeks. Which of the following is the most important next step in
evaluating the patient?
a. Tell the patient that she likely has postpartum depression and should be seen by a
counselor as soon as can be arranged
b. Prescribe the patient an SSRI for new diagnosis of postpartum depression

REVISED BY TEAM “BANG JAGO” NOV 2020 UNHAS Page 170


c. Provide careful reassurance and arrange for follow-up appointment in 2 weeks to asses
for resolution of symptoms
d. Contact the Departement of Human Services for your concerns regarding child neglect
e. Assess the patient for any current or historical thoughts of harming herself or her baby

You see a patient for preconception counseling. She is a 24-year-old G3P0020 woman,
and her medical history is significant for IgA glomerulonephritis and a recent creatinine
of 2.0. She is otherwise healthy.
49. What can you tell her to expect regarding her kidney function if she were to get
pregnant?
a. It will improve
b. It will stay the same
c. It will get worse
d. It is unlikely she will need dialysis by the end of pregnancy
e. She will need kidney transplant in order to have a successful pregnancy

50. She returns 6 months later and is 8 weeks pregnant. Her creatinine at this time is 1.8.
Her baseline 24-hour urine protein is 1,200 mg, and she has mildly impaired creatinine
clearance. You again counsel that this is a high-risk pregnancy and will need close
monitoring. Which of the following is she not at risk for during this pregnancy?
a. Preeclampsia
b. Preterm delivery
c. IUGR
d. Fetal cardiac malformations
e. Worsening renal disease

51. A patient present with amenorrhea and galactorrhea. Her PRL level are elevated. She is
not and has never been pregnant. In addition to evaluating her for a prolactinoma, one
also needs to evaluate for other causes that would increase PRL such as elevated level
of which of following?
a. Corticotropin-releasing hormone (CRH)
b. Dopamine
c. Gamma-aminobutyric acid (GABA)
d. Histamine type II receptor activation
e. Thyrotropin-releasing hormone (TRH)

52. A 24-year-old G1P1 female comes to the office colposcopy due to LSIL pap 1 month
prior. Her pap was normal 3 years ago. She began sexual activity at age 16 and had five
partners. She has had chlamydia and cigarettes per day since she was 15. She uses

REVISED BY TEAM “BANG JAGO” NOV 2020 UNHAS Page 171


Depo-Provera for contraception. Her mother died last year of cervical cancer at age 44.
What are her risk factors for CIN?
a. Age 16 at onset of sexual activity
b. Five sex partners
c. Her history of STDs
d. Smoking 15 cigarettes a day
e. All of the above

53. A 67-year-old woman present with abdominal discomfort and bloating, 10 Kgs weight
loss, decrease appetite, and fatigue. Vital signs are stable. Physical demonstrates a
menopausal woman with large left adnexal mass detected on bimanual examination.
You perform a transvaginal ultrasound that shows an 8 cm complex mass with solid
and cystic examination, thick irregular septations, and irregular surface. Based on your
clinical assumptions, what is the most likely course of management for this patient?
a. Chemotherapy
b. Abdominal exploration with surgical resection
c. Neoadjuvant chemotherapy, abdominal exploration, and surgical resection
d. Abdominal exploration with surgical resection followed by chemotherapy
e. Abdominal exploration with surgical resection followed by radiation

54. A 15-year-old girl is seen in the emergency departement. She has sudden onset of
heavy vaginal bleeding. She has noted irregular, painless vaginal bleeding of 6 months
duration. Her medical history is unremarkable, and she is not sexually active. Physical
and pelvic examinations are normal, but blood is coming through the cervical os. A
serum pregnancy test is negative and complete blood cell count has hematocrit of 37%
and normal white blood cell and platelet counts. Which of the following is the best
course of immediate action?
a. Observation
b. Estrogen therapy
c. Progesterone therapy
d. Nonsteroidal anti-inflammatory therapy
e. D&C

55. A 27-year-old P3A1 return to your clinic to follow-up on the result of her annual pap
smear. She has had a full annual examination including pap smear since the age of 21
and has never had abnormal cervical cytology. Her STD screening at her last
examination was normal. The cytology of her pap smear showed high-grade squamous
intraepithelial lesion (HGSIL). You discuss the management options to the patient, she
decides to have colposcopy. The biopsy result of the colposcopy you perform are
insufficient to give a histology report. Which of the following options is the next step in
management for this patient?

REVISED BY TEAM “BANG JAGO” NOV 2020 UNHAS Page 172


a. Repeat Pap smear at 6-month interval for a year
b. Repeat colposcopy to obtain better sample
c. Perform diagnostic excisional procedure such as loop electrical excision procedure
(LEEP)
d. Request that the lab repeats their test on the biopsy samples given

56. A 44-year-old P5 patient who is currently using oral contraceptive pills to control
menorrhagia had a hysterectomy for uterine enlargement. You sucpect adenomyosis
by history. Which histological description supports the diagnosis of adenomyosis?
a. The metaplastic change of glandular epithelium to muscle fibers in the uterus
b. The same pattern and location as endometriosis
c. The presence of endometrial glands and stroma deep within uterine muscle
d. A premalignant change of the endometrium
e. A premalignant change of the uterine muscle

57. A. 38-year-old African American woman present with heavy menses and an enlarged
uterus. After an examination the clinical diagnosis is leiomyoma of the uterus. Which
of the following best describes this finding?
a. A soft, interdigitating mass of the uterine wall
b. A premalignant papule of the uterine wall
c. A rapidly dividing necrotic malignancy
d. A rounded, smooth, firm, well-circumscribed mass
e. Erythematous, tender, and hereditary

58. A 45-year-old woman undergoes an abdominal hysterectomy for a large fibroid uterus.
She is found to have a fibroid in broad ligament, and there is a concern that her ureter
may have been damage during the difficult surgery. Which of the following is the least
possible site of ureter injury in this surgery?
a. At the level of the uterosacral
b. Lateral to the uterine vessels
c. Renal pelvicureteric junction
d. The area of the ureterovesical junction close to the cardinal
e. The base of the infundibulopelvic ligament as the ureters cross the pelvic brim at the
ovarian fossa

59. A mother brings her 2 month-old daughter to pediatrician because she feels a lump in
her abdomen. On pelvic ultrasounds the infant has an ovarian mass. When counseling
the mother, you inform her that the most common ovarian lesion associated with the
transient elevated gonadotropins in a female newborn during the first 6 to 12 months
of life is which of the following?
a. Granulosa cell tumor

REVISED BY TEAM “BANG JAGO” NOV 2020 UNHAS Page 173


b. Leiomyoma
c. Serous cystadenoma
d. Single large follicular cyst
e. Theca cell tumor

60. A woman has had a previous child with renal agenesis. She is a middle school biology
teacher and wants to understand more about development of kidneys. Which of the
following best describe the function of pronephros?
a. They begin the developmental sequence that forms the permanent excretory ducts
and kidneys
b. They are primitive kidney and ureter that will mature into the adult urinary tract
c. They develop as the primitive kidney and migrate caudally and laterally to form the
mesonephros
d. They will serve as the fetal kidney until 16 weeks and development of metanephros
e. They form the primitive kidney and primitive upper genital ducts

61. A 7-year-old girl is brought in for evaluation. On examination, she has well developed
hair and breasts and she is 99% of height for her age. Her mother recently noted some
bloodstains on her underwear. Which of the following conditions is most likely the
cause of these findings?
a. Estrogen-producing ovarian cyst
b. Hepatoma
c. Hypothalamic tumor
d. Sex steroid-containing medication
e. Thecal / Leydig cell tumor

62. A 57-year-old, post menopausal woman complain of something coming out of her
vagina, and difficulty in defecating. She had four children uneventfully with vaginal
births. Her last child birth was 18 years back. The most likely diagnosis is?
a. Anterior vaginal wall prolapse
b. Genuine stress incontinence
c. Overflow incontinence
d. Posterior vaginal wall prolapse
e. Stress incontinence
63. After trying behavioral and lifestyle modifications, the patient continues to be
symptomatic. Urodynamic studies are done to further evaluate the incontinence.
Cystometry reveals leakage of urine with increase in intra-abdominal pressure during
valsava effort and coughing. No involuntary detrusor contractions are seen. Bladder
compliance is normal. The pressure flow pattern is continuous and normal with a void

REVISED BY TEAM “BANG JAGO” NOV 2020 UNHAS Page 174


initiated by urethral relaxation and normal detrusor contraction. What type of urinary
incontinence does this patient most likely have?
a. Stress incontinence
b. Urgency incontinence
c. Overflow incontinence
d. Mixed incontinence
e. Functional incontinence
A 20-year-old nulligravid young woman presents to the emergency departement 4
hours after the onset of nausea, vomiting, and moderate lover abdominal pain. Her last
menstrual period was 2 weeks ago. She reports three new sexual partners in the last 6
months and uses condoms intermittently. She denies any history of sexually
transmitted infections. Her temperature is 38.0˚C (100.4˚F), pulse rate is 96/min,
respirations are 20/min, and BP is 110/60 mm Hg. Examination shows a soft abdomen
and lower quadrant tenderness without guarding or rebound. On pelvic examination,
there is a mucopurulent cervical discharge, moderate cervical motion tenderness, and
uterine tenderness. Bilateral adnexa are nontender and without enlargement.
64. What is the most likely diagnosis?
a. Ectopic pregnancy
b. Pelvic inflammatory disease
c. Ovarian torsion
d. Pyelonephritis
e. Cervicitis

65. What is this patient’s future risk of infertility?


a. 1%
b. 12%
c. 20%
d. 40%
e. 60%

66. Which method of contraception is not recommended in this patient?


a. Combined oral contraceptive pills
b. Injectibles
c. Condoms
d. Intrauterine contraceptive device
e. Progestin implants
A 28-year-old G2P1 at 36 weeks was referred to your clinic from a midwife, due to
previous cesarean section. She had a cesarean section due to breech presentation. She
asks if she could deliver vaginally.

REVISED BY TEAM “BANG JAGO” NOV 2020 UNHAS Page 175


67. What is a contraindication to a trial of labor after cesarean (TOLAC)?
a. Prior low transverse uterine incision
b. Prior intramural myomectomy
c. Small-for-gestational age fetus
d. Oligohydramnions
e. Prior median abdominal skin incision

68. In which of the following cases might internal podalic version be indicated?
a. Vertex delivery of the first twin and transverse lie of the second twin
b. Term transverse lie with cervix completely dilated and membranes intact
c. Double footling breech
d. Impacted shoulder presentation
e. Compound presentation

69. A 19-year-old has been bleeding intermittently for 26 weeks. She presents with a
change to dark bleeding and minimal cramping for 4 days. You clinic ultrasound notes
an intrauterine fetal demise. She is distraught. She would prefer to let “nature take its
course.” She asks if there any risk to her. What do you explain as the only significant
risk?
A 28-year-old G3P0AB2 has a quantitative hCG of 2,850. She has spotting and
abdominal pain. An ultrasound shows fluid in the cul de sac and no intrauterine
pregnancy.
70. What is the most common site of an ectopic pregnancy?
a. External fallopian tube
b. Ovarian surface
c. Mesosalpinx
d. Ampulla of the fallopian tube
e. Interstitial portion of the fallopian tube

71. An interstitial (cornual) pregnancy is discovered on ultrasound, and it is verified by


laparoscopy. Which of the following best describes the condition?
a. It rarely exceeds 4 weeks’ gestation
b. It is generally more dangerous than an ampullary ectopic pregnancy
c. It requires hysterectomy
d. It is quite common
e. It is extrauterine

72. Which of the following condition is suitable for medical management of ectopic
pregnancy with methotrexate?

REVISED BY TEAM “BANG JAGO” NOV 2020 UNHAS Page 176


a. Unstable hemodynamics
b. Gestational sac 4 cm
c. Negative embryonic cardiac motion
d. Thrombocytopenia
e. Creatinine level 2.8 mg/dl

73. After your explanation, she refuses to breast-feed because she wants to place the child
for adoption. What is simplest and safest method of lactation suppression?
a. Breast binding, ice packs, and analgesics
b. Bromocriptine
c. Domperidone
d. Depo-Provera
e. Oral contraceptive pills
A 42-year-old G4P3 woman presents to your emergency departement with a 6-month
history of irregular bleeding and a new onset of coughing up blood. Her history reveals
three term vaginal deliveries, her last being approximately 6 months ago. That delivery
was uncomplicated. On physical examination, vital signs are stable, her uterus is
approximately 10 to 12 weeks size, and there is a moderate amount of blood in the
vaginal vault. CXR shows a new single nodule in the left lower lobe suspicious for a
metastatic lesion from unknown location.
74. Which of the following laboratory tests will most likely assist in her diagnosis?
a. CA 125
b. Serum β-hCG
c. CBC
d. Prothrombin time
e. Fibrinogen

75. The pelvic ultrasound reveals bilateral multicystic ovarian masses along with an
enlarged uterus. What is the most likely diagnosis and most appropriate management
of this findings?
a. Metastatic lesions/surgical intervention
b. Primary epithelial ovarian carcinoma/rgical intervention
c. Theca lutein cysts/percutaneous drainage
d. Metastatic lesions/chemotherapy
e. Theca lutein cysts/conservative surveillance

76. You refer the patient to a gynecologic oncologist for evaluation and management of
choriocarcinoma. What is the most likely intervention to be recommended?

REVISED BY TEAM “BANG JAGO” NOV 2020 UNHAS Page 177


a. Total abdominal hysterectomy
b. Serum β-hCG surveillance
c. Chemotherapy
d. Whole pelvis radiation
e. Pulmonary wedge resection
A 24-year-old G1P1 female comes to the office for colposcopy due to LSIL pap 1 month
prior. Her pap was normal 3 years ago. She began sexual activity at age 16 and has had
five partners. She has had Chlamydia and vulvar warts in the past, both of which have
been treated and resolved. She is smoker of approximately 15 cigarettes per day since
she was 15. She uses Depo-Provera for contraception. Her mother died last year of
cervical cancer at age 44.
77. What are her risk factors for CIN?
a. Age 16 at onset of sexual activity
b. Five sex partners
c. Her history of STDs
d. Smoking 15 cigarettes a day
e. All of the above

78. If dehydration condition, which correct information:


a. Dehydration and increased osmolalitas serum maternal (>30 mOsm/kg) can cause
oligohydramnion
b. Dehydration and increased osmolalitas serum maternal (>30 mOsm/kg) can cause
polyhydramnion
c. Decreased level of osmolalitas maternal (>20 mOsm/kg) can cause increasing volume
of amniotic fluid
d. A+C are correct
e. B+C are correct

79. During pregnancy, recommendation intake water increase due to:


a. Support fetus circulation
b. Amniotic fluid production
c. Increased blood volume
d. A, B, C are correct
e. B, C are correct

80. Based on Flack et al. Study (2009):


a. Excess water consumption will not impact to amniotic fluid volume, and less of water
consumption not impact to amniotic fluid
b. Excess water consumption will impact to amniotic fluid

REVISED BY TEAM “BANG JAGO” NOV 2020 UNHAS Page 178


c. Excess water consumption will not impact to amniotic fluid volume, and less of water
consumption will impact on decreased of amniotic fluid until 8%
d. Normal water consumption will not impact to amniotic fluid volume
e. 75% pregnant mother with oligohydramnion with 2 L water will improve amniotic fluid
index

81. During pregnancy, role of water as nutrient:


a. As main medium to translate macro nutrient into energy
b. Gives energy
c. As sweetener
d. As antioxidant
e. Part of carbohydrate

NOVEMBER 2018

REVISED BY TEAM “BANG JAGO” NOV 2020 UNHAS Page 179


A 36-year-old G2PO at 26 weeks gestational age presents to the office with palpitations,
anxlety, lack of sleep, and a 5 kg weight loss over the past 2 weeks. She started to develop
substernal chest pain this morning. She has no significant medical history. On examination, she
has afebrile with a heart rate of 152 and a BP of 158/82. She appears anxious and her eyes are
prominently open. Her thyroid is enlarged but nontender. An EKG reveals sinus tachycardla
without ST segment changes.
1. Which of the following is the appropriate Immediate first line therapy?
a. Propanolol
b. Lorazepam
c. Hydrocortisone
d. Radloactive lodine
E Propylthlouracil (PTU)
2. Which clinical syimptom is not characteristic of mild thyrotoxicosis
a. Palpltation
b. tacycardia.
c. Thyromegaly
d. Cold Intolerance
e. Failure to gain weight
A 24year-old women at 32 weeks gestation complains of shortness of breath during her
pregnancy, especially with physical exertion. She has no prior medical history. Her respiratory
rate is 16x/m; her lungs are dear to auscultation; and your office oxygen saturation monitor
reveals her oxygen saturation to be 98% on room air.
3. you reassure her that this sensation is normal and explain which of the following?

a. Pulmonary resistance Increases during pregnancy.


b. Airway conductance Is decreased during pregnancy.
c. Small amniotic fluid emboli are shed throughout pregnancy.
d. Maximal breathing capacity Is not altered by pregnancy.
e. Because of enlarging uterus pushing up on the diaphragm, her vital capacity is decreased
by 20%.

REVISED BY TEAM “BANG JAGO” NOV 2020 UNHAS Page 180


4. A 33-year-old G2P1 presents at 35 weeks' gestation with complaints of nausea and vomiting.
Laboratory evaluation reveals markedly decreased glucose level, 'elevated transaminase levels, renal
dysfunction, and coagulopathy. What is the possibility diagnosis in this patient ?

a. Acute hepatitis
b. Cirrhosis hepatis
c. Intrahepatic cholestasis
d. Acute fatty liver of pregnancy
e. Non alcoholic fatty liver disease
A 32-year-old lady, G2P1A0 presented to delivery ward at 3 weeks gestation with worsening
abdominal pain for few hours. She had also had some vaginal bleeding within the past hour.
Her uterus was tender and firm to palpation. She was found to have low-amplitude, high-
frequency uterine contractions, and the fetal heart rate tracing showed recurrent late
decelerations and reduced variability. Her blood pressure was 160/100 mmHg and she has had
a +2 proteinuria. She did her antenatal care in your hospital and ultrasound examination was
performed 3 times with no remarkable abnormalities.
5. The most likely diagnosis is :

a. Vasa previa
b. Preterm labor
c. Placenta previa
d. Placental abruption
e. Preterm Premature Rupture of Membrane (PPROM)
6. From obstetrical examination you found her cervix was unfavorable. Your next plan is to deliver the
baby by :

a. Vaginal delivery
b. Elective C-section
c. Emergency C-section
d. Operative vaginal delivery
e. Observation until the cervix was favorable
7. You are counseling a couple in your clinic who desire VBAC. Her baby is in a vertex presentation,
appropriate size for 37 weeks, and her previous low transverse procedure was for breech
presentation. You have to give inform consent about VBAC. In providing informed consent, in which
of the following ways do you explain the risk of uterine rupture?

REVISED BY TEAM “BANG JAGO” NOV 2020 UNHAS Page 181


a. Less than 1%
b. Between 2% and 5 %
c. Between 15-20%
d. Depend on the length of her labor
e. Depend on the location and proximity of the scar site to the placental implantation

A 17-years-old G2P1 woman with no prenatal care at 29 weeks' gestation presents with
painful
contractions and pressure. Her cervix is 1 cm, 40% effaced, and breech at -station 2. There is
no evidence'of ruptured membranes. Her contractions are every 4 minutes. FHR are 150
bpm with acceleratlons. Maternal vital signs are temperature 36.8°C, pulse 96x/m, BP
110/72 mmHg.
8. What should you do?

a. C-section
b. Performed fetal fibronectin
c. Begin tocolytic agents and corticosteroids
d. Observe the cervical changes and labor progress
e. Perform amniocentesis to rule out chorioamnionitis
9. What fetal complication is associated with the non steroidal anti inflammatory agent indomethacin
as tocolytics agent?
a. Hydramnions
b. Achondroplasia
c. Pulmonary valve atresia
d. Bronchopulmonary dysplasia
e. Premature closure of the ductus arteriosus
10. The Nonstress test (NST) has which of the following characteristics?
a. Low positive predictive value
b. Low specificity (with reactive NST)
c. Low false positive rate ( with non reactive NST)
d. FHR reactivity depends on normal cardiac development
e. Acceleration without fetal movement should not be accepted
11. A 24 year old patient ( parity 2) has just delivered vaginally an infant weighing 4300 gr after a
spontaneous uncomplicated labor. Her prior obstetric history was a low uterine segment transverse
cesarean section section for breech. She has had no problems during the pregnancy and labor. The
placenta delivers spontaneously. There is immediate vaginal bleeding of greater than 500cc.

REVISED BY TEAM “BANG JAGO” NOV 2020 UNHAS Page 182


although all of the following can be the cause for post partum hemorrhage, which is the most
frequent cause of immediate hemorrhage as seen in this patient?
a. Uterine atony
b. Coagulopathies
c. Uterine rupture
d. Retained placental fragments
e. Vaginal and/or cervical lacerations

Mr. X, 24 yo come to the ER with complains of headaches since the last day of examinations
obtained expecting her first child, gestational age 32 weeks with blurred vision and denied
heart burn. On physical examination found BP 190/120 mmHg, Pulse 90 x/m, breathing 16
x/m. leopold found the lower left back head, FHR 140 bpm, Irregular contraction. Pelvic
score of 1 was found, pelvis size wide. Laboratory investigation: Hb 11.5 g%, Platelets
9000/mm3, LDH 510 iu/L, Proteinuria +2, ALT 10 u/L, AST 15 u/L
12. What is the best diagnosis for mrs. X
a. HELLP Syndrome
b. Severe Preeclampsia
c. Chronic hypertension
d. Gestational hypertension
e. Superimposed Preeclampsia
13. What is the most clinically effective anti hypertensive agent for mrs. X
a. ISDN
b. Atenolol
c. Nifedipine
d. Furosemide
e. Metildopa
14. Which antihypertension drugs can cause fetal growth restriction?
a. Nifedipine
b. Atenolol
c. Hydralazine
d. Captopril
e. Methyldopa
15. A woman is being treated with magnesium sulphate, there is concern about magnesium
Toxicity. What is the first sign of magnesium toxicity?
a. Bradycardia
b. Rediuced consciousness
c. Respiratory depression
d. Decreased urine output
e. Loss of deep tendon reflexes
16. With two home pregnancy tests and ultrasound revealed 6-7 wga pregnancy. As her pregnancy
continues, you would expect her cardiac output to increase by which of the following
mechanisms:
a. An increased heart rate alone
b. A decrease in systemic vascular resistance

REVISED BY TEAM “BANG JAGO” NOV 2020 UNHAS Page 183


c. First an increase in stroke volume then an increase in heart rate
d. Cardiac output would not change significantly until the third trimester
e. An increase in systemic vascular resistance facilitated by elevated progesterone levels
17. Which of the following is true regarding the physiologic changes she might expect during her
pregnancy?
a. An overall decrease in the number of WBC and platelets
b. Gastric emptying and large bowel motility are increased in pregnancy
c. An increase in the tidal volume along with an increase in total lung capacity (TLC)
d. BUN and creatine will decrease as a result of an increase in glomerular filtration rate (GFR)
e. Nausea and vomiting that should be treated aggressively with antiemetics and intravenous
hydration
18. A woman with a previous stillbirth and post partum DVT is found to have lupus anticoagulant
and medium titre Immunoglobulin M (IgM) anticardiolipin anyibodies (ACL) on two occasions in
a subsequent pregnancy:
a. Warfarin should be discontinued
b. She has an increase risk of miscarriage
c. She requires antibiotic prophylaxis to cover delivery
d. Low dose aspirin should be discontinued at 34 weeks
e. She does nor require postpartum heparin is she has vaginal delivery

A 28 yo G1P1 woman is being discharged from the hospital in postoperative day 4 after having
received a primary low transverse cesarean section for breech presentation, with an estimated
blood loss of 700 mL. her pregnancy was otherwise uncomplicated and her hospital course was
also uncomplicated. Ten days after cesarean section, the patient came complaining of
abdominal pain and fever. Fundal height 2 fingers below navel.
19. What is the most likely probable diagnosis of the patient?
a. Metritis
b. Mastitis
c. Typhoid fever
d. Urinary tract infection
e. Breast enlargement
20. Lower urinary tract symptoms with pyuria but a steriole urine culture are likely due to which
pathogen?
a. Candida
b. E. coli
c. Proteus mirabilis
d. Klensiella pneumoniae
e. Chlamydia trachomatis

A 29 yo G2P1 woman came to outpatient clinic with obesity, a histort of GDM in the prior
pregnancy, and a strong family history for type 2 Diabetes mellitus (T2DM) presents at 7 weeks
gestation. In her previous pregnancy, she required insulin therapy. She delivered at 39 weeks
and her baby boy weighed 4300g

REVISED BY TEAM “BANG JAGO” NOV 2020 UNHAS Page 184


21. In addition to the routine prenatal laboratory tests, what other testing will you obtain at this
point?
a. HbA1C
b. No other testing needed
c. An ultrasound to estimate gestational age
d. Perform a glucose challenge test at first visit
e. Perform a glucose challenge test at 24 weeks gestation
22. Laboratory test result return, and her fasting blood glucose is 145 mg/dL. An ultrasound reveals
the pregnancy to be 7 weeks and 2 days, consistent with LMP, HbA1C is 7.5%. her diagnosis is:
a. GDM
b. T1DM
c. T2DM
d. No diabetes in pregnancy
e. Impaired glucose tolerance
23. A 41 yo women present at 36 weeks of gestation in active labour. The ultrasound scan at 32
weeks showed low lying placenta. Immediately after rupture of the membrane, she started
bleeding vaginally with associated cardiotocography (CTG) of non reassuring. What is the most
likely diagnosis?
a. Vasa previa’
b. Placenta previa
c. Placenta accrete
d. Abruptio placenta
e. Low lying placenta

A woman has a booking scan in16 weeks gestation, which reveals a monochorionic diamniotic
twin pregnancy. She asks you about the risks regarding her pregnancy.
24. Regarding complications of twin pregnancy:
a. Cesarean section is the preffered route of delivery
b. With significant growth discordance, particularly when the first twin is the smaller
c. In twin to twin transfusion syndrome, the haemoglobin levels both twins are often nor
discordant
d. In atwin pregnancy with one fetal loss in the third trimester, in 90% of cases the remaining
twin will be delivered within 72 h
e. Twin reversed srterial perfusion sequence is associated with high mortality in the recipient
twin due to prematurity and intrauterine cardiac failure
25. Regarding twin to twin transfusion syndrome
a. The donor develop hydrops
b. The recipient develops polyhydramnios
c. Quinterro classification is up to quinterro IV
d. The perinatal mortality in twins reaches to 85%
e. Complicates up to 35% of dichorionic multiple pregnancies
26. The most frequent twin pregnancy is:
a. Conjoined twins
b. Dizygotic twins

REVISED BY TEAM “BANG JAGO” NOV 2020 UNHAS Page 185


c. Dichorionic diamniotic
d. Monochorionic diamniotic
e. Monochorionic mono amniotic
27. Which of the following statements regarding chorionocity is true?
a. A dichorionic pregnancy is always dizygotic
b. Monochorionic membranes habe four layers
c. Monochorionic twins are always monozygotic
d. Determnination of chorionicity is easiest in the second trimester
e. Complications in twin pregnancy is more frequent in dichorionic pregnancy

A 38 yo P6 lady is being operated for abdominal delivery. Unfortunately uterine contraction is


not good. The patient suffers for massive bleeding. The operator quickly decides to perform
uterine removal in order to stop the bleeding
28. Which artery that should be blocked if the operator would like to stop the blood flows to the
uterine artery?
a. Pudenda artery
b. Abdominal aorta
c. Hypogastric artery
d. Common iliac artery
e. Uterine and ovarian arteries
29. The uterine artery:
a. Gives a branch to ovary
b. Runs at the back of the ureter
c. May anstomose with femoral artery
d. Gives myometrium vascularization only
e. Is a branch of the anterior division of the internal iliac artery
30. The ovarian arteries:
a. Are crossed by the uterus
b. Arise just above the renal artery
c. Reach the ovary through round ligament
d. Reach the ovary through infundibulo-pelvic ligament
e. Anastomose with the descendent branch of uterine artery

a 22 years old female, G1 at 26 weeks gestation, presents to the office for her routine
obstetrical visit. Currently, she is complaining shortness of breath. She has no other complaints.
On physical examination. BGA results: PH 7.45 (7.35-7.45), pO2 103 mmHg (75-100), pCO2 28
mmHg (35-45), HCO3 17 mEg/L (22-26), BE 2 mmol/L, O2 sat 99% (>95%)
31. What is your explanation abput the cause of symptom of this patient?
a. Cardiac output increases 20%
b. Maternal blood volume increases 50%
c. Decrease in stroke volume and blood viscocity
d. The uterus and the diaphragm becomes elevated
e. The heart is displaced upward, and somewhat to the right with rotation on its long axis

REVISED BY TEAM “BANG JAGO” NOV 2020 UNHAS Page 186


32. What is the result of blood gas analysis (BGA) stated above?
a. Normal BGA
b. Metabolic Acidosis
c. Metabolic Alcalosis
d. Respiratory acidosis
e. Respiratory alkalosis
33. What is the most appropriate next management in this patient?
a. Ask patient to perform echocardiography examination
b. Ask patient to perform chest radiography examination
c. Admit the patient to the hospital and give non rebreathing mask
d. Counsel the patient that this is physiologic changes during pregnancy
e. Terminate the pregnancy because the symptom will become worsen in 32 weeks pregnancy
34. What is ECG finding consider normal during pregnancy?
a. Specific ST changes
b. Increased heart rate (15%)
c. Inverted T wafes in lead II
d. 15 degree right axis deviation
e. Irreversible ST waves changes

A 32 yo female, G1 at 8 weeks gestation, presents to the office for her routine obstetrical visit.
She asks you about the nutrition demand during pregnancy. Her BMI is 24 kg/m2. No
remarkable past medical history is noted
35. According to who Asian Criteria, her BMI is classified as:
a. Normal
b. Underweight
c. Overweight
d. Obese type 1
e. Obese type 2
36. She ask you what is the optimal total weight gain during her pregnancy:
a. <5 kg
b. 5-9 kg
c. 7-11.5 kg
d. 11.5-16 kg
e. 12.5 – 18 kg
37. What is the most likely risk of the patient?
a. Anemia
b. Congenital anomaly
c. Gestational diabetes
d. Spontaneous abortion
e. Post partum hemorrhage
38. A 24 yo woman with a positive pregnancy test presents with a good history of tissue expulsion
vaginally. A transvaginal ultrasound scan shows an empty uterus with an endometriak thickness
of 11 mm. regarding her diagnosis, you consider that:
a. She has had a complete miscarriage and needs no further treatment

REVISED BY TEAM “BANG JAGO” NOV 2020 UNHAS Page 187


b. She has had a pregnancy of unknown location and needs further investigations
c. She should be offered a hysteroscopy
d. She should be offered medical management of miscarriage
e. A laparascopy should be performed to exclude an ectopic pregnancy
39. Which of the following us true about the screening and diagnosis of spina bifida?
a. A maternal serum alpha fetal protein (MSAFP) of 2.5 multiples of the median (MoM) is
diagnostic of an NTD
b. First trimester ultrasound screening for NTDs primarily involves identification of a normal
posterior fossa during aneuploidy screening to rule out the presence of a Arnold chiari II
malformation
c. MRI has proven to be a better diagnostic tool for NTDs than ultrasound
d. The banana sign is indicative of ventriculomegaly, which is present in most cases of open
NTDs
e. Diagnosis of lesion level requires three dimensional ultrasound or MRI modalities
40. On prenatal ultrasound, which of the following feature characterize gastrochisis?
a. The abdominal wall defect is superior to cord insertion
b. Ectopia cordis is present
c. The abdominal wall defect is lateral to cord insertion
d. The abdominal wall defect is lower than cord insertion
e. The bladder cannot be visualized
41. A 39 years old female G2P1A0, 15 weeks pregnant presents to your clinic for having routine
ANC. On physical examination, you found her fundal height equals umbilical point. You
performed ultrasound and saw a multilocular hypoechoic mass sized 10 cm (in diameter) in her
left adnexa. No free fluid in her abdomen and pelvis. What is your consideration in this case?
a. The incidence of adnexal masses in pregnancy is 1 %
b. The incidence of ovarian cancers in pregnancy is between 1:1000
c. The most common type of benign ovarian cyst in pregnancy is a mature teratoma
d. The most common histopathological subtype for malignant ovarian tumor in pregnancy
is epithelial ovarian tumor
e. The resolution rate of adnexal masses in the second trimester of pregnancy is 60-70%

42. The patient does not believe that she has ovarian cyst during pregnancv. She is really
concern about the possibility of malignancy. Regarding this situation, what Would you
inform her?
a. The most common mode of presentation of an adnexal mass is pain
b. The sensitivity of detection of ovarian cysts on clinical examination alone is less than
5%.
c. The size of ovarian cyst that should prompt investigation for malignancy is 10 cm
d. The validated sensitivity and specificity of IOTA rules on ultrasound evaluation of an
ovarian cyst is sensitivity: 78%, specificity: 87%
e. The sensitivity and specificity of MRI in the diagnosis of a malignancy is 100 and 94
respectively

REVISED BY TEAM “BANG JAGO” NOV 2020 UNHAS Page 188


43. You have checked her CA125 serum level and the result was 350 µ/ml. You performed
conservative surgical staging by laparotomy because her frozen section revealed
malignancy cyst. After 1 week, pathology result serous papillary carcinoma of the left
ovary. She is planned for chemotherapy. What will you Inform to her regarding
chemotherapy for ovarian cancer during pregnancy?
a. in a patient with ovarian cancer In pregnancy receiving chemotherapy the delivery
should be planned at completion of chemotherapy
b. use of chemotherapy pregnancy generally considered safe after 20 weeks of
gestation
c. CNS and neural tube complications occur during the week 8-12 weeks in pregnancy
d. This percentage of patients receiving chemotherapy in pregnancy who develop
major congenital malformations is 30-40%
e. cardiovascular defects are common congenital malformations in platin based
chemotherapy regimens

28 yo woman, G1 36 weeks of gestational age, went to your clinic to do routine antenatal


are. During ultrasound, the doctor told her that she will be expecting baby boy with estimated
fetal weight 2500 g. however, amniotic fluid considered to be less than normal. Then you asked
..
he patient to drink minimal of 2L of Water a day and get herself another ultrasound within 3
days to evaluate the amniotic fluid.

44. Amniotic fluid volume is a balance between production and resorption. What is
the primary
a. Fetal breathing
b. Fetal swa llowing
c. Absorption across fetal skin
d. Ab sorption by fetal kidneys
e. Filtration by fetal kidneys
45. ln a normal fetus at term, what is the daily volume of fetal urine that contributes to the
amount of amniotic fluid present?
a. 200 ml
b. 250 ml
c. S00 ml
d. 750 ml
e. 1000 ml
46. 37-yea rs-old ca me lo your office at 32 weeks of gestation according to her last
menstrual period. She has no ultrasound examination before and did not do her
routine antenatal ca re. the vital sign is withi n normal limit. She has body mass index 19
kg/m2. During physical examination, the uterine fundal height is 22 cm. From
ultrasound examination, the Foetus has biometric values that correlate with 30 fetus.
Which of the following is the next best step in managing this patient?
a. Antenatal care routinely for the next 2 weeks
b. Evaluate maternal status and comorbidities
c. Consider deliver the baby

REVISED BY TEAM “BANG JAGO” NOV 2020 UNHAS Page 189


d. Repeat sonography for fetal growth in 2. Weeks
e. Doppler velocimetry evaluation every 3 days
47. According to algorithm for management of fetal-growth restriction, you evaluate the
Doppler Velocimetry the find reversed end-diastolic flow and oligohydramnios. What is
the appropriate management at this time?
a. Regular fetal testing
b. Weekly evaluation of amniotic fluid
c. Consider corticosteroids for lung maturation
d. Deliver the baby '
e. Reevaluate middle cerebral arteries and ductus venosus

A 26-years-old woman, GlP0A0 was admitted to ER because she lost her consciousness around 1
hour ago. According to her husband, she Is 36 weeks pregnant. She performed antenatal care at
scheduled time, and never missed one. Her husband said, she never had any hypertension or
any other disease before. Three days prior hospitalization, she had severe nausea and vomiting.
Physical examination reveals, BP 120/80 mmHg, Pulse rate 87 x/m, RR 18 x/min. Temperature
36.5oC. there is an icteric sclera. Other physical examination was remarkable. obstetrical
examination reveals no fetal heartbeat was detected laboratory examination reveals CBC
10.2/29.9/8900/263.000; Ur/Cr l8/0.; AST/ALT. 458/878 Urinalysis was within normal limit.

51. What is the best next management in this case?


a. Abdominal ultrasound
b. lnduction of labor
c. Emergency Caesarean section
d. Whole Blood transfusion
e. Injections of 40% Dextrose

52. What is the underlying pathophysiology of intrahepatic cholestasis of pregnancy?


a. Acute hepatocellular destruction
b. lncomplete clearance of bile adds
c. Microvasculnr thrombus accumulation
d. Eosinophil infiltration of the liver
e. Hepatocellular injury

A 33-year-old woman, GlP0A0, came to hospital with major complaint, watery leakage. She was
on her 33 weeks of gestational age. Data from medical record showed that she came previously
a week ago, complaining vaginal discharge. Vaginal swab has been done.

53. In case above, what kind of examination should you performed for establishing
diagnosis.
a. Vaginal examination
b. inspeculo
c. Blood test
d. Ultrasound

REVISED BY TEAM “BANG JAGO” NOV 2020 UNHAS Page 190


e. Simple Urine Test

54. You found on Leopold 1, hard, round with ballottement (+). Contraction was Infrequent
and weak. A What was your next plan?
a. Went for labour induction
b. Immediate C section
c . Tocolytic and corticosteroid provision
d. Performed ultrasound
e. Performed external version
55. Ultrasound examination showed that trans-cerebellar diameter was proper to
gestational age, abdominal circumference was lower than: 2,5 persentil and amniotic
fluid pocket was 1.2 cm. What Is the most likely diagnosis?
a. Growth restriction with oligohydramnios
b. Normal Growth with olygohydramnion
c. Growth restriction with normal amniotic fluid
d. Normal growth with normal amniotic fluid
e. Need another examination for establishing diagnosis.

56. Lack of baby movement had been felt for two days, fetal heart rate was 146 bpm. What
was your next step?
a. Termination of pregnancy
b. Giving oxygenation and left lateral position
c. Ensuring Fetal well-being b Manning criteria
d. Fetal tung maturation
e. Giving Intravenous fluid rehidration.

57. Cardiotocography, showed low variability with checkmark pattern and no desceleration.
What was your lnterpretation and the best management through?
a. category one, continued for fetal lung maturation
b. category two, intrauterine resuscitation for 24 hours and reevaluationa after.
c. Category two, went for doppler velocymetri
d. Category three, went for doppler velocymetry ultrasound exam
e. Category three, delivered the baby

58. A patient wishes to consider pregnancy after treatment for her breast cancer. What is
the most important predictor of a good prognosis?
a. Young age
b. Herceptin positivity
c. Estrogen receptor positivity
d. BRCA gene positivity
e. Family history of treatable breast cancer

REVISED BY TEAM “BANG JAGO” NOV 2020 UNHAS Page 191


59. A primigravida at 36 weeks gestation is measuring large for dates. Ultrasound shows AC
> 97 th centile. GIT performed shows poorly controlled gestational diabet_es. What is the
immediate management plan?
a. Give steroids
b. Start induction
c. Start hypoglycaemic drugs
d. Wait and watch
e. Start sliding scale and delivered the baby

60. During cesarean section, the uterine contraction suddenly became poor. No significant
bleeding was found. The vital sign was within normal limit. B Lynch procedure is then
applied. The main concep- t of B Lynch is
.
a. If bimanual compression of the exteriorized uterus decreases the uterine bleed Ing.
b. If bimanual compression of the exteriorized uterus does not decrease the uterine
bleeding.
c. If bimanual compression of the exteriorized uterus increases the uterine bleeding.
d. If bimanual compression of the exteriorized uterus decreases the uterine contraction.
e. If bimanual compression of the exteriorized uterus Increases the uterine contraction.

61. 27 yo patient and her husband present to you with primary infertility. The patient ports
regular periods every 28 to 30 days. The patient has no significant medical history and
does not take any medications other than prenatal vitamins. Her husband is also in good
health, 30 yea r of age, and has two children from a previous marriage. When you asked
the patient how long they have been trying to achieve a pregnancy, they tell you 6
months. Your instructions to the couple are the following:
a. They will likely need IVF to achieve a pregnancy
b. They will likely need lUI cycles
c. Continue trying appropriately timed intercourse for 6 more months and return for
follow up if no pregnancy is achieved
d. Consider .egg donor
e. C_onsider adoption

62. A 29 year old lady with 30 year old husband come with 2 years of primary infertility.
Semen analysis, HSG and an endocrine evaluation including FSH, E2, TSH prolactin levels,
and ovarian reserve testing had been done. All of the tests come back normal. Your next
recommendation.
a. Have 6 more months of timed intercourse and return for follow up visit if no
pregnancy Is achieved
b. Clomiphene citrate with IUI
c. Gonadotropin lnjection
d. Human gonadotrophin
e. IVF

REVISED BY TEAM “BANG JAGO” NOV 2020 UNHAS Page 192


63. Characteristically, menopause begins with cycle irregularity that extends to 1 year after
permanent cessation of menses. The more correct, scientific terminology for this time is
menopausal transition, and it tipically takes place over a span of how many years?
a. 1 to 2 years
b. 2 to 3 years
c. 4 to 7 years
d. 5 to 10 years
e. 11 to 12 years
64. A 49 year old woman had a radical hysterectomy and lymoh node sampling for stage 1B
squamous cell cancer of the cervix. A suprapubic catheter was placed at the time of surgery. She
is now 8 weeks postoperative and has not been able to void. She is also leaking urine with
activity, coughing, and sneezing. What is the most likely reason for voiding difficultya?
a. Spasm of the pelvc floor muscles
b. Outflow obstruction
c. Postoperative swelling around the bladder
d. Innervation to the lower urinary tract was transected
e. Overdistention of the bladder

A 46 years old lady, parity 2 presents to you with a complaint of LLQ pain, intermittent nausea,
abdominal preassure, and bloating. Her history is notable for mild obesity, right breast cancer ,
and hypertension. Her family history is notable for premenopausal breast cancer in her mother
and maternal aunt. She had a pelvic ultrasound showing a left ovarian mass containing internal
setations and papillary projection. She has moderate ascites and her CA 125 was 719
65. Which of the following is associated with an increased risk of ovarian cancer?
a. History of breast cancer
b. Breastfeeding
c. Multiparity
d. Tubal ligation
e. Obesity
66. She underwent TAH, BSO, collection of pelvic washings, omentectomy, cytoreduction or
debulking and bilateral and para aortic lymph node sampling. The mass had spread beyond the
ovary to the omentum, peritoneum, and bowel. She was found to have ascites and pelvic
wahings were positive. What stage of ovarian cancer does she have?
a. Stage I
b. Stage II
c. Stage III
d. Stage IV
e. Stage V

a couple presents because they have been trying to conceive for 18 months. During the
interview you learn that the man has fathered a child in a previous relationship and is in good
health. The woman is 28 and reports that she has had painful menses for the past 5 to 6 years

REVISED BY TEAM “BANG JAGO” NOV 2020 UNHAS Page 193


67. You begin to suspect that she ,may have endometriosis. All of information below would increase
that suspicios expect:
a. She reports that a maternal cousin has a history of endometriosis
b. She has experienced dyspareunia with deep penetration for several years
c. Her ethnicity is Caucasian
d. She report the development of abnormal bleeding in the last year
e. Her menarche began at age 9
68. A 35 yo women with 2 children came to specialist clinics with the complaint of vaginal discharge
with her pap smear result showed : CIN 3+ HPV infection, she ask for the appropriate treatment
What is the next diagnostic procedure should be offered to the patient?
a. HPV DNA test
b. Liquid based cytology papsmear
c. Ultrasound
d. VIA
e. Colposcopy

 Although serum progesterone levels vary in the normal luteal phase in a pulsatile
manner, a serum progesterone level above 3 to 5 ng/mL suggests some ovulatory
function, but it cannot indicate the adequacy of normal ovulation. Progesterone
levels of 10 ng/mL or higher are found during at least 1 day of the luteal phase of
normal ovulatory cycles in which conception occurred 

In most women, and particularly in women older than 35 years, serum follicle-
stimulating hormone (FSH) and estradiol (E 2 ) levels should be obtained on cycle
day 2 or 3. Elevated FSH values (>10 mIU/mL), suggest decreased ovarian reserve,
which reflects the pool of viable oocytes remaining in the ovary. Levels over 20
mIU/mL afford a particularly poor prognosis. However, although FSH levels tend to
fluctuate from cycle to cycle, once the FSH level has been elevated in a given cycle,
the overall prognosis is reduced. E 2 levels, if elevated on days 2 and 3 (>70 pg/mL),
do not allow for a valid interpretation of FSH values and may independently suggest
a decreased prognosis regarding ovarian reserve ( Evers, 1998 ).

Levels are highest in young women and lower with reproductive aging; various
nomograms by age have been established ( Seifer, 2011 ) ( Fig. 42.4 ). Serum
AMH/MIS decreases with aging, and when levels reach 0.05 ng/mL (essentially
undetectable levels), menopause occurs within 4 to 5 years ( Sowers, 2008 ). Levels are
higher in women with polycystic ovary syndrome (PCOS) ( Iliodromiti, 2013 ). In terms
of ovarian reserve, higher levels (>2 ng/mL) suggest a larger cohort of small available
follicles and low levels (<0.5 ng/mL) suggest a decreased ovarian reserve, with the
levels of AMH reflecting the sensitivity of the ovary to gonadotropic stimulation, and
thus the choice of treatment when ovarian stimulation is desired ( La Marca, 2014 ).

REVISED BY TEAM “BANG JAGO” NOV 2020 UNHAS Page 194


TSH values in the normal range (<4.4 μU/mL), but higher than 2.5 mIU/mL are
often considered to be abnormal in women presenting with infertility. This is because
normal values in the first trimester of pregnancy should be <2.5 μU/mL.

A 36 yo patient, P0, presents to your clinic for fertility workup. She had been married for 2
years with regular intercourse. Her menstrual cycle is normal. Her general status was normal.
Vaginal examination revealed normal findings
69. Which of the following examination that is not included in basic workup in patient above?
a. Hysterosalpingodraphy
b. Ultrasonography
c. Semen analysis
d. Mid luteal progesterone examination
e. Serum AMH
70. The following month she came back with the result of hysterosalpingography

pictures

What will be your next plan?


a. Repeat HSG next month
b. Schedule diagnostic laparascopy
c. Gives clomiphene citrate and plan for natural conception
d. Gives clomiphene citrate and plan for intrauterine insemination
e. Plan for IVF
71. A 68 yo woman presents with vulvar pruritus since the previous year that has been increasing
over the last few months. She has tried antifungal medications, which seem to help, but the
symptoms keep recurring. She went through menopause at age 49 and has not been sexually
active for 10 years . she does not use any douching products and is not taking any antibiotics. On
physical examination, you note thin white epithelium of the labia minora with multiple red oval
shaped erosions, varying in size from 0.5 to 1.5 cm. how would you proceed?
a. Culture the vagina and treat with high dose antifungal
b. Wide local excision of the lessions
c. Cryotherapy to eradicate the lessions
d. Punch biopsy of the vulvar lesions
e. Treat with moderate high potency topical steroids
72. A 27 yo woman presents to her gynecologist for evaluation of new vulvar lesions. She first
several “bumps” on her outer labia 3 weeks ago. She is otherwise without any associated
symptoms. On pelvic examination there are six flesh colored, nontender, 1 to 3 mm verrucous
papules. Which of the following is the most likely cause of the patient’s vulvar lesions?
a. HPV serotype 16
b. Herpes simplex virus (HSV) type 1
c. Pox virus
d. HPV serotype 6
e. Haemophillus ducreyi
73. A 45-year-old presents for evaluation because her primary care physician has diagnosed her
with pelvic organ prolapse while performing annual care. She denies any pelvic pressure, bulge,

REVISED BY TEAM “BANG JAGO” NOV 2020 UNHAS Page 195


or difficulty with urination. Her only medical comorbidity is obesity. For asymptomatic grade 1
pelvic organ prolapse, what do you recommend?
a. Conservative management with pelvic floor muscle exercises and weight loss
b. Colpodleisis obliterative procedure
c. Gellhorn pessary
d. Round ligament suspension
e. Hysterectomy
74. A 62-year-old G2 P2 presents to the urogynecology clinic with complaints of urinary
incontinence. she has urinary urgency and can't make it to the bathroom before leaking a large
amount of urine. She gets up twenty three times per night to urinate. A urinalysis and urine
culture done 1 week ago at her PCP's office are both negative. What is the most likely diagnosis
and appropriate treatment option for this type of urinary incontinence?
a. Stress incontinence, mid-urethral sling
b. Urgency incontinence, oxybutynin(anticholinergic medication)
c. Overflow incontinence, oxybutynin(anticholinergic medication)
d. Urinary fistula, surgical repair
e. Functional incontinence, bladder suspension
75. The majority of vulvar, vaginal and cervical cancers appear to have a common cause and usually
caused by:
a. High risk type of HSV infection
b. High Risk Type of HPV infection
c. Increased exposure to endogenous estrogen
d. Increased exposure to exogenous estrogen
e. Chronic bacterial and parasitic infection
76. Childhood neoplastic ovarian masses most commonly originate from:
a. Gonadal epithelium
b. Gonadal stroma
c. Sex cords
d. Germ cells
e. Metastatic disease
77. Your patient is a 13 yo adolescent girl who presents with cyclic pelvic pain. She has never had a
menstrual cycle. She denies any history on intercourse. She is afebrile and her vital signs are
stable. On physical examination, she has qge appropriate breast and pubic hair development
and normal external genitalia. However, you are unable to locate a vaginal introitus. Instead
there is a tense bulge where the introitus would be expected. You obtain a transabdominal
ultrasound, which reveals a hematocolps and hematometra. What Is the most likely diagnosis?
a. Transverse vaginal septum
b. Longitudinal vaginal septum
c. Imperforate hymen
d. Vaginal atresia
e. Bicornuate uterus
78. An 89-year-old female patient with multiple, serious medical comorbidities presents to discuss
options for treatment of her high-grade prolapse. The prolapse Is externalized and becoming
ulcerated from friction against her undergarments. She cannot tolerate a pessary. Her main

REVISED BY TEAM “BANG JAGO” NOV 2020 UNHAS Page 196


priority is to "fix or get rid of this thing "but her primary care provider has cautioned against a
lengthy or open abdominal procedure. She is not Interested in future Intercourse. What can you
offer this patient?
a. Nothing can be done
b. Open abdominal sacral colpopexy
c. Robot-assisted laparoscopic sacral colpopexy
d. Hysterectomy with anterior and posterior colporrhaphy, vault suspension.
e. Colpocleisis
79. A patient retumns for a postoperative checkup 2 weeks after a total abdominal hyserectomy for
fibroids. She is distressed because she is having continous leakage of urine from the vagina. Her
leakage is essentially continous and worsens with coughing, laughing, or movement. Given her
history and physical,you perform both a methylene blue dye test, which is negative and an
indigo carnine test, which is positive. The most likely diagnosis is:
a. Rectovaginal flstula
b. Uretro vaginal fistula
c. Vesico vaginal fistula
d. Uretero vagina fistula
e. Impossible to distinguish
80. A 38 years old multi gravid woman complains of the painless loss of urine, beginning
immediately with coughing, laughing, lifting, or straining. Immediate cessation of the activity
stops the urine loss after only a few drops. This history is most suggestive of
a. Fistula
b. Stress incontinence
c. Urge incontinence
d. Urethral diverticulum
e. UTI
81. Criteria for diagnosis of Gestational Trophoblastic Neoplasia or Postmolar Gestational
Trophoblastic Disease:
a. Plateau of serum BhCG level(±10%)for four measurements during a period of 3 weeks or
ionger-days 1,7,14,21.
b. Rise of serum BhCG<10% during 3 weekly consecutive measurements or longer, during a
period of 2 weeks or more-days 1,7,14
c. Rise of serum BhCG<10% during 2 weekly consecutive measurements or longer, during a
period of 4 weeks or more-days 1,7,14,21
d. The serum BhCG levels remains detectable for 3 months or less
e. The serum BhCG levels remains detectable for 12 months or less
82. A 53-year-old woman presents for counseling and management of her low-grade anterior wall
prolapse. She is only symptomatic on days when she has engaged in heavy lifting. She is
morbidly obese and would like to begin a formal weight loss program. She agrees to a trial with
a pessary. You choose a supportive pessary and counsel her regarding pessary insertion,
removal, cleaning, etc Which of the following are factors associated with a good response for
pessary in uterine prolapse?
a. Vaginal length 9 cm
b. Obeslty

REVISED BY TEAM “BANG JAGO” NOV 2020 UNHAS Page 197


c. History of traumatic vaginal deliveries
d. Menopausal state
e. Large genital hiatus

REVISED BY TEAM “BANG JAGO” NOV 2020 UNHAS Page 198


MARET 2019

9. The Nonstress Test (NST) has which of the following characteristics?


a. Low positive predictive value
b. Low specificity (with reactive NST)
c. Low false-positive rate (with nonreactive NST)
d. FHR reactively depends on normal cardiac development
e. Acceleration without fetal movement should not be accepted

10. Corticosteroids administered to women at risk for preterm birth have been
demonstrated to decrease rates of neonatal respiratory distress if the birth is delayed for
at least what amount of times after the initiation of therapy?
a. 12 hours
b. 24 hours
c. 36 hours
d. 48 hours
e. 72 hours
11. A 24 yo patient, P2, has just delivered vaginally weighing 4300 gr after a
spontaneous uncomplicated labor. Her prior obstetric history was a low uterine segment
transverse caesarean section for breech. She has had no problems during the pregnancy
and labor. The placenta delivers spontaneously there is immediate vaginal bleeding of
greater than 500 cc. although all of the following can be the cause for PPH. Which is the
most frequent cause of immediate hemorrhage as seen in this patient?
a. Uterine atony
b. Coagulopathjies
c. Uterine rupture
d. Retained placental fragments
e. Vaginal and/ or cervical lacerations

REVISED BY TEAM “BANG JAGO” NOV 2020 UNHAS Page 199


Mrs X, 24 yo came to the ER with complaints of headaches since the last day of
examinations obtained expecting her first child. Gestasional age is 32 weks With
blurred vision and dented heartburn. On physical examination found BP 190/120
mmHg, pulse 90 breathing 16 X/M. Leopold found the lower left back head, FHR 140
bpm, irregular contraction. Pelvic score of I was found, pelvis size wide. taborðtory
investigation; hemoglobin IIS platelets 49000/MM3, LDH 754 iu/L, Proteinurea +2, ALT
60 u/L, ast 75 U/L.

14. What is the best diagnosis for Mrs. X

a. Preeclampsia
b. HELLP syndrome
c. Chronic hypertension

d. Gestational hypertension

e. Superimposed preeklampsia

15. What is the most clinicIlly effective antihypertensive agent for Mrs. X
a. ISDN
b. Atenolol
c. Nifedipin
d. Furosemide
e. metyldopa
16. Which antihypertension drugs con cause feta' growth restriction7

a. nifedipin
b. atenolol
c. Hydratazinc

d. Captopni

e. Methyldopa

REVISED BY TEAM “BANG JAGO” NOV 2020 UNHAS Page 200


17. A woman is being treated W/th magnesium sulphate- There is concern about
magnesium toxicity. What is the first Sign of magneqium toncity?
a. Bradycardia
b. Reduced consciousness
c. Respiratory depression
d. Decreased urine output
e. Loss of deep tendon reflexes

18. With two home pregnancy tests and ultrasound revealed 6-7 wea pregnancy. As her
pregnancy continues, you would expect her cardiac output to increase by which of the
following mechanisms:
a. An increased heart rate alone
b. A decrease in systemic vascular resistance
c. First an increase in stroke volume, then an increase in heart rate
d. Cardiac output would not change significantly until the third trimester
e. An increase in systemic vascular resistance facilitated by elevated progesterone level
19. Which Of the following is true regarding the physiologic Changes she might expect
during her pregnancy?
a. An overall decrease in the number of WBC and platelets
b. Gastric emptying and largo bowel motility are in pregnancy
c. An increase in the tidal volume with an increase in total lung c•ðpaclty (TLC)
d. BUN and creatinine will decrease as Of an increase in glomerular filtration rote (GFR)
e. Nausea and vomiting that should treated aggressive with antiemetics and intravenous
hydration

20. A woman with previous stillbirth and postpartum DVT is found to have lupus
anticoagulant and medium-titre Imunoglobulin M (IgM) anticardiolipin antibodies (aCL)
on two occasion. In a subsequent in pregnancy
a. Warfarin should be discontinued
b. She has an increase risk of miscarried

REVISED BY TEAM “BANG JAGO” NOV 2020 UNHAS Page 201


c. She requires antibiotic prophylaxis to cover delivery
d. Low dose aspirin should be discontinued at 34 weeks
e. She doesn,t required postpartum heparin if she has vaginal delivery

A 28 years old G1P1 women being discharged from the hospital on postoperative day 4
after having a primary low transverse caesarean section for breech presentation, with an
estimated blood lost 700ml, her pregnancy was otherwise no complication and her
hospital course was also no complication
Ten day after the caesarean section, the patient complaining of abdominal pain and fever,
Fundal height 2 finger below navel

21. what is the most likely propable diagnosis of the patient


a. Metritis
b. Mastitis
c. Thyfoid Fever
d. Urinary tract infection
e. Breast engorgement

22. A patient calls your clinic complaining of continued heavy vaginal bleeding, she
has an “uncomplicated” vaginal birth 2 weeks ago of her second child, what is the most
likely diagnosis from the following differentiated?
a. Uterine Atony
b. Uterine Rupture
c. Coagulopathies
d. Vaginal laceration
e. Retained Placenta fragment

A 29 years old G2P1woman came to outpatient clinic with obesity, a history of GDM in
the prior pregnancy and a strong family history for type 2 diabetes mellitus (T2DM)

REVISED BY TEAM “BANG JAGO” NOV 2020 UNHAS Page 202


present at 7 weeks, in her previous pregnancy, she required insulin therapy, she delivered
at 39 weeks and her baby boy weight 4300
23. in addition to the prenatal laboratory test , what other testing will you obtain at
the point
a. HbA1c
b. No other test needed
c. An ultrasound to estimate gestasional age
d. Perform a glucose challenge test at first visit
e. Perform a glucose challenge test at 24 weeks gestasion

24. laboratory test result return, and her fasting blood glucose 145 mg/dl, an
ultrasound reveals the pregnancy to be 7 weeks 2 days consisted with LMP. Hemoglobin
A1c is 7,5%, Her diagnosis is:
a. GDM
b. T1DM
c. T2DM
d. No diabetes in pregnancy
e. Impaired glucose tolerance

25. by which mechanism of transfer does glucose cross the placenta


a. Carrier Mediated active transport
b. Channels
c. Facilitated diffusion
d. Simple diffusion
e. Solvent drag

26. A 41 years old woman present at 36 weeks of gestation in active labour, the
ultrasound scan at 32 weeks showed low lying placent. Immediately after rupture of
membran, she start bleeding vaginally with associated cardiotocography of non

REVISED BY TEAM “BANG JAGO” NOV 2020 UNHAS Page 203


reassuring, what is the most likely diagnosis?
a. Vasa previa
b. Placenta previa
c. Placenta accreta
d. Abruptio Placenta
e. Low lying placenta

A woman has a booking scan in 16 weeks gestation, which reveal monochorionic


diamniotic twin pregnancy , she ask you about the risk regarding pregnancy
27. regarding the complication of twin pregnancy
a. Caesarean section is the preferred routine of delivery
b. With the significant growth discordance, particularly when the twin is smaller
c. In twin to twin transfusion syndrome , the haemoglobin level for both twinsare often
not discordant
d. In a twin pregnancy one fetal loss in the third trimester, in 90 percent of cases the
remaining twin will be delivered within 72 H
e. Twin reserved arterial perfusion sequence is associated with high mortality in the
recipient twin due prematurity and intra-uterine cardiac failure

28. the timing of separation of the embryo in monochorionic diamniotic is


a. Up to 4 days
b. 4-7 days
c. 7-14 days
d. ¿ 14 days
e. ¿ 28 days

29. regarding twin to twin transfusion syndrome

REVISED BY TEAM “BANG JAGO” NOV 2020 UNHAS Page 204


a. The donor develops hydrops
b. The recipient develops polyhidramnios
c. Quinterro classification up to quinterro IV
d. The perinatal mortality in twins reach to 85%
e. Complicates up to 35% of dichorionic multiples pregnancies

30. the most frequent twin pregnancy is


a. Conjoined twins
b. Dizygotic twins
c. dichorionic diamniotic
d. monochorionic diamniotic
e. monochorionic monoamniotic

31. which of the following statement regarding chorionocity is true


a. A dichorionic pregnancy is always dizygotic
b. Monochorionic membran have four layers
c. Monochorionic pregnancy is always monozygotic
d. Determination of chorionocity is easiest in the second trimester
e. Complication in twin pregnancy is more frequent in dichorionic pregnancy

A 38 years old P6 lady is being operated for abdominal delivery. Unfortunately uterine
contraction is not good. The patient suffer for massive bleeding, the operator quickly
decides to perform uterine removal in order to stop bleeding

32. Which artery that should be blocked if the operator would like to stop the blood flow
to the uterine artery ?
a. Pudenda artery
b. Abdominal aorta

REVISED BY TEAM “BANG JAGO” NOV 2020 UNHAS Page 205


c. Hypogastric artery
d. Common iliac artery
e. Uterine and ovarian arteries

33. the uterine artery:


a. Gives the branch to ovary
b. Runs at the back of the ureter
c. May anastomose with femoral artery
d. Gives myometrium vascularization only
e. Is a branch of anterior division of the internal iliac artery

34. the ovarian arteries


a. Are crossed by the ureters
b. Arise just above the renal artery
c. Reach the ovary through round ligament
d. Reach the ovary through infundibulo-pelvic ligament
e. Anastomose with the descendent branch of the uterine artery

A 22 years old female G1 at 26 weeks gestation, present to the office for her routine
obstetrical visit, currently, she is complaining shortness of breath, she has no other
complaints, on physical examination, blood pressure 100/70 mmhg, PR 90 Bpm, RR
22x/m. no abnormality was found in chest examination. BGA result PH 7,45 ( 7,35 - 7,45),
pO2 103 mmhg (75 - 100), PcO2 28 mmHg ( 35-45 ) HCO3 -17 mEq/L (22-26), BE 2 mmol (-
2 - 2), O2 sat 99% (¿ 95 %

35. what is your explanation about the cause of symptom of this patient
a. Cardiac output increased 20%
b. Maternal blood volume increased 50%

REVISED BY TEAM “BANG JAGO” NOV 2020 UNHAS Page 206


c. Decrease in stroke volume and blood viscocity
d. The uterus and the diaphragma became elevated
e. The heart is displace upward, and somewhat to the right with rotation on its long axis

36. What is the result of the Blood Gas Analysis ( BGA stated above?
a. Normal BGA
b. Metabolic Acidosis
c. Metabolic Alkalosis
d. Respiratory Ascidosis
e. Respiratory Alkalosis

37. what is the most appropriate next management in this patient


a. Ask patient to perform echocardiography examination
b. Ask patient to perform chest radiography examination
c. Admit the patient to the hospital and give non-rebreathing mask
d. Counsel to the patient that this is physiologic changes during pregnancy
e. Terminated the pregnancy because the symptom will became worsen in 32 weeks
pregnancy

38. what is ECG Finding consider normal during pregnancy


a. Spesific ST changes
b. Increased heart rate (15%)
c. Inverted T waves in lead II
d. 15 degreeright axis deviation
e. Irreversible ST waves changes

REVISED BY TEAM “BANG JAGO” NOV 2020 UNHAS Page 207


A 32 years old female, G1 at 8 weeks gestation, present to the office for her routine
obstetrical visit. She ask you about the nutrition demand during pregnancy, Her BMI is
24kg/m2. no remarkable past medical history is noted

39. According to WHO Asian criteria, her BMI Classified as:


a. Normal
b. Underweight
c. Overweight
d. Obese type 1
e. Obese type 2
40. she ask you what is the optimal total weigt gain during her pregnancy
a. < 5 kg
b. 5-9 kg
c. 7-11,5 kg
d. 11,5-16 kg
e. 12,5-18 kg

41. what is the most likely rosk of this patient?


a. Anemia
b. Congenital anomaly
c. Gestational diabetes
d. Spontaneous abortion
e. Post partum hemorrhage

42. A 29 years old woman with a positive pregnancy test presents with a good history of
tissue expulsion vaginally. A transvaginal ultrasound scan shows an empty uterus with an
endometrial thickness of 11 mm. regarding her diagnosis, you consider that:
a. She has had a complete miscarriage and needs no further investigations
b. She has had a pregnancy of unknown location and needs further investigations
c. She should be offered a hysteroscopy
d. She should be offered medical management of miscarriage
e. A laparascopy should be perform to exclude an ectopic pregnancy

43. A 39 years old female G2P1A0, 15 weeks pregnant present to your clinic for having
routine ANC. On physical examination, you found her foundal height equals umbilical

REVISED BY TEAM “BANG JAGO” NOV 2020 UNHAS Page 208


point. You performed ultrasound and saw a multilocular hypoechoic mass size 10 cm (in
diameter) in her left adnexa. No free fluid in her abdomen and pelvis. What is your
consederation inyhis case?
a. The incidence of adnexal masses in pregnancy is 1%
b. The incidence of ovarian cancers in pregnancy is between 1; 1000
c. The most common type of benign ovarian cyst in pregnancy is a mature teratoma
d. The most common histopathological subtype for malignant ovarian tumor in
pregnancy is epithelial ovarian tumor
e. The resolution rate of adnexal masses in the second trimester of pregnancy is 6-70%

44. you have checked her CA125 serum level and the result was 350 u/m. you performed
conservative surgical staging by laparatopmy because her frozen section revealed
malignancy cyst. After 1 week, pathology result serous papillary carcinoma of the left
ovary. She isplanned for chemotherapy. What will you inform to her regarding
chemotherapy for ovarian cancer during pregnancy?
a. In a patient with ovarian cancer in pregnancy receiving chemotherapy the delivery
should be planned at completion of chemotherapy
b. Use of chemotherapy in pregnancy generally considered safe after 20 weeks of
gestation
c. CNS and neural tube complication occur during the week 8-12 weeks in pregnancy
d. This percentage of patients receiving chemotherapy in pregnancy who develop major
congenitl malformations is 30-40%
e. Cardiovascular defects are common congenital malformations in platin based
chemotherapy regiments

A 28 yo woman, G1 36 weeks of gestasional age, went to your clinic to do routine


antenatal care. During ultrasound, the doctor told her that she will be expecting baby boy
with estimated fetal weight 2500g, however, amniotic fluid considered to be less than
normal. The you asked the patient to drink minimal of 2L of water a day and get herself
another ultrasound within 3 days to evaluate the amniotic fluid.
45. Amniotic fluid volume is a balance between production and resorption. What is the
primary mechanism of fluid resorption?
a. Fetal breathing
b. Fetal swallowing
c. Absorption across fetal skin
d. Absorption across fetal kidneys
e. Filtration by fetal kidneys

REVISED BY TEAM “BANG JAGO” NOV 2020 UNHAS Page 209


46. In a normal fetus at term, what is the daily volume of fetal urine that contributes to
the amount of amniotic fluid present?
a. 200 mL
b. 250 mL
c. 500 mL
d. 750 mL
e. 1000 mL

Mrs. B 37-years-old came to your office at 32 weeks of gestation according to her last
menstrual period. She has no ultrasound examination before and did not do her routine
antenatal care. The vital sign is within normal limit. She has body mass index 19kg/m2.
During physical examination, the uterine fundal height is 27 cm. From ultrasound
examination, the fetus has biometric values the correlate with 30 weeks fetus.
47. Which of the following is the next best step in managing this patient?
a. Antenatal care routinely for the next 2 weeks
b. Evaluate maternal status and comorbidities
c. Consider deliver the baby
d. Repeat sonography for fetal growth in 2 weeks
e. Doppler velocimetry evaluation every 3 days
48. According to algoritm for management of fetal-growth restriction, you evaluate the
Doppler Velocimetry then find the reversed end-diastolic flow and oligohydramnios. What
is the appropriate management at this time?
a. Regular fetal testing
b. Weekly evaluation of amniotic fluid
c. Consider corticosteroids for lung maturation
d. Deliver the baby
e. Reevaluate middle cerebral arteries and ductus venosus
A 33-year-old woman, G1P0A0, came to hospital with major complaint, watery leakage.
She was on her 33 weeks of gestational age. Data from medical record showed that she
came previously a week ago, complaining vaginal discharge. Vaginal swab has been done.
49. In case above, what kind of examination should you performed for estabilishing
diagnosis?
a. Vaginal examination
b. Inspeculo
c. Blood test
d. Ultrasound

REVISED BY TEAM “BANG JAGO” NOV 2020 UNHAS Page 210


e. Simple urine test
50. You found on Leopold 1, hard, round with ballottement (+). Contraction was
infrequent and weak. What was your next plan?
a. Went for labour induction
b. Immediate C-section
c. Tocolytic and corticosteroid provision
d. Performed ultrasound
e. Performed external version
51. Ultrasound examination showed that trans-cerebellar diameter was proper to
gestational age, abdominal circumference was lower than 2.5 centile and amniotic fluid
deepest pocket was 1.2 cm. What is the most likely diagnosis?
a. Growth restriction with oligohydramnios
b. Normal Growth with olygohydramnion
c. Growth restriction with normal amniotic fluid
d. Normal growth with normal amniotic fluid
e. Need another examination for establishing diagnosis
52. Cardiotocography, showed low variability with checkmark pattern and no
acceleration. What was your interpretation and the best management through?
a. Category one, continued for fetal lung maturation
b. Category two, intrauterine resuscitation for 24 hours and reevaluation after
c. Category two, went for doppler velocymetri
d. Category three, went for doppler velocymetri ultrasound exam
e. Category three, delivered the baby
53. A primigravida came to your clinic with premature rupture of membrane at 39 weeks
gestation. Cervix was unfavorable. Ultrasound shows AC > 97 th centile, with estimated
fetal weight 4300 g. Random blood glucose performed shows 385 g/dL. What is the
immediate management plan?
a. Give steroids
b. Start induction
c. Wait and watch
d. Start hypoglychaemic drugs
e. Start sliding scale and deliver the baby
54. During cesarean section, the uterine contraction suddenly became poor. No
significant bleeding was found. The vital sign was within normal limit. B lynch procedure is
then applied. The main concept of B Lynch is
a. If bimanual compression of the exteriorized uterus decreases the uterine bleeding.

REVISED BY TEAM “BANG JAGO” NOV 2020 UNHAS Page 211


b. If bimanual compression of the exteriorized uterus does not decrease the uterine
bleeding.
c. If bimanual compression of the exteriorized uterus increases the uterine bleeding.
d. If bimanual compression of the exteriorized uterus decreases the uterine contraction.
e. If bimanual compression of the exteriorized uterus increases the uterine contraction
55. A 26 years old woman, parity 2 gave birth to a baby a month ago and is breast feeding
the baby. She requests a short term contraception. What is the most appropriate option
with the highest successful rate to prevent pregnancy?
a. Condom
b. Combination oral contraceptive pill
c. Intra uterine devices
d. Progestogen only pills
e. Lactation Amenorhea Method (LAM)
56. Which of the following is not included in non contraceptive benefits of COC ?
a. Lower incidence of ovarian cancer
b. Lower incidence of endometrial cancer
c. Lower incidence of cervical cancer
d. Lower incidence of dysmenorrhea
e. Reducing skin acne with using cryproterone acetate
57. Which of the following is included in class 4 of WHO medical eligibility of IUD
a. Benign ovarian tumor
b. Gestational trophoblastic disease with lowering of HCG
c. Fibroid distorting uterine cavity
d. History of PID
e. HIV infected
A 28-year-old G3P1A1 presents with the history of fatigue, mild palpation and dyspnea
upon exertion. She was unable to tolerate her prenatal vitamins during pregnancy,
because of nausea. Examination reveals pallor and spooning of her nails. Vital signs are
normal. There is no lymphadenopathy or hepatosplenomegaly.
58. If her Hb level is 8.2 g/dL, MCV 70fL, MCH 25 picograms/red cell, MCHC 22 g/dL, RDW
16% peripheral blood smear result was microcytic and hypochromic red cells. The most
likely diagnosis is :
a. Iron deficiency anemia
b. Megaloblastic anemia
c. Haemolytic anemia
d. Aplastic anemia

REVISED BY TEAM “BANG JAGO” NOV 2020 UNHAS Page 212


e. Acute blood loss anemia
59. to provide accurate diagnostic, you next diagnostic tools in:
a. Platelet count
b. Hb electrophoeisis
c. Reticulocyte count
d. Coombs test
e. Serum ferritin
60. if the symptoms were worsening to cardiac compromise ie. Symptomatic at rest
withdyspnea, chest pain, or presyncope, your advice is?
a. High fibre diet
b. Red cell transfusion
c. Adjunct ascorbic acid
d. Oral iron supplementation
e. Parenteral iron replacement
A 28 yo woman and her partner present to their physicians as they have not been able to
achieve pregnancy during the past 2 years. Her partners, age 35 and she are both healthy
and take no prescription medications. They are sexually active two to three times per
week and do not use any contraception. She had regular menstrual period
61. appropriate investigation for this couple:
a. AMH examination
b. Complete gynecology examination
c. HSG and semen analysis
d. Progesterone level
e. Follicular antral basal investigation
62. this patient has begun therapy with clomiphene citrate. Before she starts therapy.
What information should you provide her regarding the medication?
a. typically, the timing of ovulation is increased by a week
b. approximately 40% of patients will respond to clomiphene citrate with increased
endometrial thickness
c. the risk of multiple gestation is 25%
d. clomiphene citrate improves the fecundity rate principally through its effect on the
endometrial lining
e. clomiphene citrate can cause nausea, hot flushes weight gain, and mood swings
a 34 yo woman came to out patient clinic due to secondary ammenorhea for 4 months.
She has 17 yo child and no history if contraception. She said that her period is not regular
since 1 year ago. Her BMI was 30. Her father has diabetes

REVISED BY TEAM “BANG JAGO” NOV 2020 UNHAS Page 213


63. if you do a pelvic ultrasound , what result that you expect to find?
a. Normal uterus and both ovary
b. Normal sized uterus with thickened endometrium, both ovary are normal
c. Normal uterus, Polycistic appearance in both ovary
d. Normal uterus ovarian cyst with fine internal ratio
e. Enlarged with intramural clear border mass
64. what laboratory test that you suggest?
a. LH levels
b. Progesterone levels
c. FSH and E2 levels
d. Glucose tolerance test
e. CA 125 levels
65. if the ultrasound result a normal sized uterus with endometrial thickness of 8 mm and
both ovary have typical polycystic appearance. What would be the recommended
management in this patient?
a. Endometrial biopsy
b. Induction of 3 monthly withdrawal bleeds with progestin
c. Methformin twice daily
d. Ovulation induction with gonadotropin
e. Reassure and discharge
A 22 yo patient and her husband present to you with primary infertility. The patient
reports regular periods every 28 to 30 days. The patient has no significant medical history
and does not take any medication other prenatal vitamins. Her husband is also in good
health, is 30 year of age. And has two children from a previous marriage. When you asked
the patient how long they have been trying to achieve a pregnancy, they tell you 6
months
66. your instructions to the couple are the following:
a. They will likely need IVF to achieve a pregnancy
b. They will likely need IUI cycles
c. Continue trying approximately timed intercourse for 6 more months and return for
follow up if no pregnancy is achieved
d. Consider egg donor
e. Consider adoption
67. A 29 year old lady with 30 year old husband come with 2 years of primary
infertility. Semen analysis, HSG and an endocrine evaluation including FSH, E2, TSH

REVISED BY TEAM “BANG JAGO” NOV 2020 UNHAS Page 214


prolactin levels, and ovarian reserve testing had been done. All of the tests come back
normal. Your next recommendation.
f. Have 6 more months of timed intercourse and return for follow up visit if no
pregnancy Is achieved
g. Clomiphene citrate with IUI
h. Gonadotropin lnjection
i. Human gonadotrophin
j. IVF

68. Characteristically, menopause begins with cycle irregularity that extend to 1 year
after permanent cessation of menses. The more correct, scientific terminology for this
time is menopausal transition, and it typically takes place over a span of how many years?
a. 1 to 2 years
b. 2 to 3 years
c. 4 to 7 years
d. 5 to 10 years
e. 11 to 12 years
69. A 49 year old woman had a radical hysterectomy and lymoh node sampling for
stage 1B squamous cell cancer of the cervix. A suprapubic catheter was placed at the time
of surgery. She is now 8 weeks postoperative and has not been able to void. She is also
leaking urine with activity, coughing, and sneezing. What is the most likely reason for
voiding difficultya?
f. Spasm of the pelvc floor muscles
g. Outflow obstruction
h. Postoperative swelling around the bladder
i. Innervation to the lower urinary tract was transected
j. Overdistention of the bladder
A 46 years old lady, parity 2 presents to you with a complaint of LLQ pain, intermittent
nausea, abdominal preassure, and bloating. Her history is notable for mild obesity, right
breast cancer , and hypertension. Her family history is notable for premenopausal breast
cancer in her mother and maternal aunt. She had a pelvic ultrasound showing a left
ovarian mass containing internal setations and papillary projection. She has moderate
ascites and her CA 125 was 719
70. Which of the following is associated with an increased risk of ovarian cancer?
f. History of breast cancer
g. Breastfeeding
h. Multiparity
i. Tubal ligation
j. Obesity
71. She underwent TAH, BSO, collection of pelvic washings, omentectomy, cytoreduction
or debulking and bilateral and para aortic lymph node sampling. The mass had spread

REVISED BY TEAM “BANG JAGO” NOV 2020 UNHAS Page 215


beyond the ovary to the omentum, peritoneum, and bowel. She was found to have
ascites and pelvic wahings were positive. What stage of ovarian cancer does she have?
f. Stage I
g. Stage II
h. Stage III
i. Stage IV
j. Stage V

72. A 35 years old women with 2 children came to specialist clinics with the
complaint of vaginal discharge with her pap smear result showed : CIN 3 + HPV Infection,
she ask for the appropriate treatment.
What is the next diagnostic procedure should be offered to the patient?
a. HPV DNA Test
b. Liquid Based Cytology Papsmear
c. Ultrasound
d. VIA
e. Colposcopy

A 45 years old women came to hospital with major complaint chronic leucorhea. She had
been treated by various antibiotics given by general practicioner and midwives. Recently
sge got bleeding each after intercourse. General condition is unwell with anemic
appearance. BP 70/palpable, pulse weak 120x/minutes

73. What is the best step for this patient?


a. Performed bimanual examination
b. Performed ultrasound examination
c. Giving oxygen and putting the IV line
d. Performed emergenct curettage
e. Giving tranexamic acid

74. By first impression, what would the diagnosis would like to be?
a. Uterine sarcoma
b. Cervical polyps
c. Adenomyosis
d. Uterine myoma
e. Cervical cancer

75. What is the etio-pathogenesis of the diagnosis above?


a. Malignant transformation of myocyte
b. Hyperestrogenic intra cavitary condition
c. HPV type 6 infection
d. Herpes simplex virus infection
e. HPV type 16 infection

REVISED BY TEAM “BANG JAGO” NOV 2020 UNHAS Page 216


46 years old G2P2 obese woman is referred from her primary care physician because of
increasingly heavy and painful menses over the last 118 months; She has tried an oral
contraceptive with some improvement of her bleeding but no improvement in her pain.
She did sports no other history of pelvic pain or abnormal bleeding in the past. She has
never had an abnormal pap smear and states she has never had any infections, “down
there’ On examination, you note normal external genitalia, vagina, and cervix. However,
her uterus is slightly enlarged, mildly tender, and softer than you expected, she has no
adnexal mass or tenderness.
76. You explain to your patient that you think she may have adenomyosis and that it
is most likely causing her symptoms. She is reluctant for having surgery. Which of the
following management is appropriate?
a. Start combined oral contraception
b. Depo provera injection every month
c. LNG-IUD
d. GnRH analog injection for 3 months
e. Oral NSAID
A 30 years old patient came with complaint of infertility. Her husband is a 33 years old
who has had a semen analysis, which was reported as normal. On further history, the
patient reports that her periods have been quiet irregular over the last year and that she
missed her period in the last 4 months. She also reports insomnia, vaginal dryness, and
decreased libido.
77. What is the most likely diagnosis for this patient based on her history
a. Polycystic ovarian syndrome
b. Primary ovarian insufficiency
c. Endometriosis
d. Kallman Syndrome
e. Spontaneous pregnancy
A 34 years old woman with primary infertility 3 years, oligomenorrhea and a body mass
index (BMI) of 26. Day 23 progesterone level result was 5 ng/ml. Transvaginal ultrasound
shows multiple small follicle size 5-8 mm in both ovary. HSG show bilateral patent tubes.
Her partner’s semen analysis show a volume of 3 ml, pH of 7 and a sperm count of 20
million/ml
78. The patient later on given clomiphene citrate 1x50 mg for 5 days, but during
follow up there were no dominant follicle. Which of the following is appropriate?
a. We should increase the dose to 100 mg/day for 5 days
b. The patient corresponds to CC resistance and should undergo laparoscopy ovarian
drilling

REVISED BY TEAM “BANG JAGO” NOV 2020 UNHAS Page 217


c. The patient corresponds to CC failure and should switch to gonadotropin injection
d. Give aromatase inhibitor
e. Combine letrozole with CC

A 20 years old lady come to outpatient clinic never had menstruation. She had no history
surgery or medication. On physical examination reveal BMI 20kg/m2, tanner stage M3P3.
From ultrasound uterus cannot be visualized. Both ovaries was vusialized sized 28x25mm
and 26x20mm with dominant follicle size 18mm on the left ovary.

79. Which of the following is the most appropriate diagnosis?


a. PCOS
b. Kallman syndrome
c. Premature ovarian insufficiency
d. MRKH syndrome
e. Hyperprolactinemia

80. Which characteristics that differentiate the above diagnosis with androgen
insensitivity syndrome?
a. Presence of axillary and pubic hair
b. Elevated level of prolactin
c. Elevated level of FSH
d. Low level of AMH
e. Elevated level of testosterone

81. If the diagnosis is confirmed, what is her option to achive pregnancy?


a. Conventional IVF
b. IVF-ICSI
c. Minimal stimulation IVF
d. Oocyte donaton
e. Gestational surrogacy

82. A 7 year old girl present to her pediatrician with her parents who concerned
anout her early sexual development. She is developing breast , axillary hair, and pubic hair
and they are noticing body odor. A through clinical workup reveal the child has an
irregular echogenic, thickly septated ovarian mass on her left ovary. What type of the
tunor is responsible for child’s clinical presentation?
a. Dysgerminoma
b. Embryonal Carcinoma
c. Sertoli-Leydig Cell Tumor
d. Endodermal Sinus Tumor
e. Granulosa-Theca Cell Tumor

REVISED BY TEAM “BANG JAGO” NOV 2020 UNHAS Page 218


A 49 year old woman experience irregular vaginal bleeding for 3 months duration. You
performed endometrial biopsy, which obtain copious tissue with velvety, lobulated
texture. The pathologist report show proliferation of glandular and stromal elements with
dilated endometrial glands, consistent with simple hyperplasia. Cytologic atypia is absent

83. Which of the following is the best way to advise the patient?
a. She should be treated to estrogen and progestin hormone therapy
b. The tissue will progress to cancer in approximately 10% cases
c. The tissue may be weakly premalignant and progress to cancer in approximately 1% of
cases
d. She requires a hysterectomy
e. No further therapy is needed

84. Which of the following factors is productive against endometrial hyperplasia ?


a. Obesity
b. Tamoxifen
c. Oral Contraceptive Pills (OCPs)
d. Early menarche or late menopause
e. Unopposed exogenous estrogen therapy

A-36-years old patient P0, present to your clinic for fertility workup. She had been
married for 2 years with regular intercourse. Her menstrual cycle is normal. Her general
status was normal vagina examination revealed normal findings.

85. Which of the following examination that is not included in basic workup in the
patient above?
a. HSG
b. Ultrasonography
c. Semen analysis
d. Mid luteal progesterone examination
e. Serum AMH

86. The following month she come back with the result of hysterosalphingography

What will be your next plan ?

REVISED BY TEAM “BANG JAGO” NOV 2020 UNHAS Page 219


a. Repeat HSG next month
b. Schedule diagnostic laparoscopy
c. Gives Clomiphene citrate and plan for natural conception
d. Gives clomiphene citrate and plan for intrauterine insemination
e. Plan for IVF
87. A 68 years old woman present with vulvar pruritus since the previous year that
has been increasing over the last few months. She had tried antifungal medication, which
seems to help, but the symptoms keep recurring. She went through menopause at age 49
and has not been sexually active for 10 years. She does not use any douching products
and is not taking any antibiotics. On physical examination, you note thin white epithelium
of the labia minora with multiple red oval shaped erosions, varying in size from 0.5 to 1.5
cm. How would you proceed?
a. Culture the vagina and treat with high dose antifungal
b. Wide local excision of the lesion
c. Cryotherapy to eradicate the lesion
d. Punch biopsy of the vulvar lesion
e. Treat with moderate high potency topical streoids
88. A 27 years old woman presents to her gynecologist for evaluation of new vulvar
lesion. She first notice several “bumps” on her outer labia 3 weeks ago. She is otherwise
without any associated symptoms. On pelvic examination there are six fresh-colored,
non-tender, 1 to 3 mm verrucous papules. Which of the following is the most likely cause
of the patient’s vulvar lesion?
a. Human papillomavirus (HPV) serotype 16
b. Herpes simplex virus (HSV) type 1
c. Pox virus
d. Human papillomavirus (HPV) serotype 6
e. Haemophilus ducreyi

94. An 18 years old nulligravid woman presents to the student health clinic with a 4 weeks
history if yellow vaginal discharge. She also reports vulvar itching and irritation. She is
sexually active and monogamous with her boyfriend. They use condoms inconsistently.
Om physical examination, she is found to be nontoxic and afebrile. On genitourinary
examination, vulvar and vaginal erythema is noted along with a yellow, frosty,
malodorous discharge with a PH of 6.5. The cervix appear to have erythematous
punctuation. There is no cervical, uterine, or adnexal tenderness. The addition of 10%
KOH to the vaginal discharge does not produce an amine odor. Wet prep microscopic
examination of the vaginal swabs is performed. What would you expect to see under
microscopy?
a. Branching hyphae

REVISED BY TEAM “BANG JAGO” NOV 2020 UNHAS Page 220


b. Multinucleated giant cells
c. Scant WBC
d. Flagellated, motile organism
e. Epithelial cells covered with bacterial
95. An 89 years old female patient with multiple, serious medical comorbidities presents
to discuss options for treatment for her high grade prolapse. The prolapse is externalized
and becoming ulcerated from friction against her undergarments. She cannot tolerate a
pessary. Her main priority is to “fix or get rid of this thing”, but her primary care provider
has cauntioned against a lengthy or open abdominal procedure. She is not interested in
future intercourse. What can you offer this patient?
a. Nothing can be done
b. Open abdominal sacral colpopexy
c. Robot-assisted laparoscopic sacral colpopexy
d. Hysterectomy with anterior and posterior colporrhaphy, vault suspension
e. Colpocleisis
96. A patient return for as postoperative checkup 2 weeks after a total abdominal
hysterectomy for fibroids. She is distressed because she is having a continuous leakage of
urine from the vagina. Her leakage is essentially continuous and worsen with coughing,
laughing or movement. Giver her history and physical, you perform both a methylene
blue dye test which is negatibe and an indigo carnine test which is positive. The most
likely diagnosis
a. Rectovaginal fistula
b. Uretro vaginal fistula
c. Vesico vaginal vistula
d. Uretero vagina fistula
e. Impossible to distinguish
98. A 38 years old multi gravida woman complaints of the painless loss of urine, beginning
immediately with coughing, laughing, lifting, or straining. Immidiate cessation of the
activity stops urine loss after only a few drops. This history is most suggestive of
a. Fistula
b. Stress incontinence
c. Urge incontinence
d. Urethral diverticulum
e. UTI

99. The principles for follow up hydatidiform molar pregnancy :


a. Prevent pregnancy during the follow up period at a minimum for 6 months
b. Measure serum chorionic gonadotropin levels every 4 weeks
c. Measure serum chorionic gonadotropin levels every 4 months
d. Measure serum chorionic gonadotropin levels every 3 weeks

REVISED BY TEAM “BANG JAGO” NOV 2020 UNHAS Page 221


e. Observation in selected, highly individualized patient may be an option, to perform
hysterectomy

100. A 25 years old lady come with abnormal pap smear result. She underwent
colposcopy examination and the result is a acetowhite lesion with punctation and atypical
vessel. Biopsy result confirms CIN 1 with HPV DNA positive test. What do you suggest for
patient?
a. LEEP procedure
b. Reevaluation of HPV DNA
c. Cold knife conization
d. Repeat cytology in 12 months
e. Repeat cytology in 6 months

JULI 2019

1. A 17-years-old G2P1 woman with no prenatal care at 29 weeks gestation presents


with painful contractions and pressure. Her cervix is 1 cm, 40% effaced, and breech
at station 2. There is no evidence of ruptured membranes. Her contractions are
every 4 minutes. FHR are 250 bpm with accelerations. Maternal vital signs are
temperature 36,8 degree C, pulse 96x/m, BP 110/72mmHg. What fetal
complication is associated with the nonsteroidal anti-inflamatory agent
Indomethacin as tocolytics agent?
a. Hydramnios
b. Achondroplasia
c. Pulmonary valve atresia
d. Bronchopulmonary dysplasia
e. Premature closure of ductus arteriosus

REVISED BY TEAM “BANG JAGO” NOV 2020 UNHAS Page 222


2. A 17-years-old G2P1 woman with no prenatal care at 29 weeks gestation presents
with painful contractions and pressure. Her cervix is 1 cm, 40% effaced, and breech
at station 2. There is no evidence of ruptured membranes. Her contractions are
every 4 minutes. FHR are 250 bpm with accelerations. Maternal vital signs are
temperature 36,8 degree C, pulse 96x/m, BP 110/72mmHg. The nonstress test
(NST) has which of the following characteristic?
a. Low positive predictive value
b. Low specificity (with reactive NST)
c. Low false positive rate (with nonreactive NST)
d. FHR reactivity depends on normal cardiac development
e. Acceleration without fetal movement should not be accepted
3. Corticosteroids administered to women at risk for preterm birth have been
demonstrated to decrease rates of neonatal respiratory distress if the birth is
delayed for at least what amount of time after the initiation of therapy?
a. 12 hours
b. 24 hours
c. 36 hours
d. 48 hours
e. 72 hours
4. A 24 years old patient (Parity 2) has just delivered vaginally an infant weighing 4300
g after a spontaneous uncomplicated labor. Her prior obstetric history was a low
uterine segment transverse caesarean section for breech. She heas had no
problems during the pregnancy and labor. The placenta delivers spontaneously.
There is immediate vaginal bleeding of greater than 500 cc. Although all of the
following can be the cause for postpartum haemorrhage
Which is the most frequent cause of immediate hemorrheage as seen in this patient?
a. Uterine atony
b. Coagulopathies
c. Uterine rupture

REVISED BY TEAM “BANG JAGO” NOV 2020 UNHAS Page 223


d. Retained placental fragments
e. Vaginal and/or cervical lacerations
5. Mrs. X, 24 yo came to the ER with complaints of headaches since the last day of
examinations obtained expecting her first child, gestational age 32 weeks with
blurred vision and denied heartburn. On physical examination found BP 190/120
mmHg, pulse 90 x/m, breathing 16 x/m, Leopold found the lower left back head,
FHR 140 bpm, irregular contraction, Pelvic score of 1 was found, pelvis size wide,
Laboratory investigation; hemoglobin 11.5 g%, plateles 49000/mm3, LDH 754 iu/L,
Proteinuria +2, ALT 60 u/L, AST 75 u/L. What is the best diagnosis?
a. Preeclamsia
b. HELLP Syndrome
c. Chronic hypertension
d. Gestational hypertension
e. Superimposed preeclampsia
6. Mrs. X, 24 yo came to the ER with complaints of headaches since the last day of
examinations obtained expecting her first child, gestational age 32 weeks with
blurred vision and denied heartburn. On physical examination found BP 190/120
mmHg, pulse 90 x/m, breathing 16 x/m, Leopold found the lower left back head,
FHR 140 bpm, irregular contraction, Pelvic score of 1 was found, pelvis size wide,
Laboratory investigation; hemoglobin 11.5 g%, plateles 49000/mm3, LDH 754 iu/L,
Proteinuria +2, ALT 60 u/L, AST 75 u/L. What is the most clinically effective
antihypertensive agent?
a. ISDN
b. Atenolol
c. Nifedipine
d. Furosemide
e. Methyldopa
7. Which antihypertension drugs can cause fetal growth restriction?
a. Nifedipine

REVISED BY TEAM “BANG JAGO” NOV 2020 UNHAS Page 224


b. Atenolol
c. Hydralazine
d. Captopril
e. Methyldopa
8. A woman is being treated with magnesium sulphate. There is no concern about
magnesium toxicity. What is the first sign of magnesium toxicity?
a. Bradycardia
b. Reduced consciousness
c. Respiratory depression
d. Decreased urine output
e. Loss of deep tendon reflexes
9. With two home pregnancy test and ultrasound revealed 6-7 wga pregnancy. As her
pregnancy continuous, you would expect her cardiac output to increase by which
of the following mechanisms:
a. An increased heart rate alone
b. A decrease in systemic vascular resistance
c. First an increase in stroke volume then an increase in heart rate
d. Cardiac output would not change significantly until the third trimester
e. An increase in systemic vascular resistance facilitated by elevated progesterone
levels
10. Which of the following is true regarding the physiologic changes she might expect
during her pregnancy?
a. An overall decrease in the number of WBC and platelets
b. Gastric emptying and large bowel motility are increased in pregnancy
c. An increase in the tidal volume along with an increase in total lung capacity (TLC)
d. BUN and creatinine will decrease as a result of an increase in glomerular filtration
rate (GFR)
e. Nausea and vomiting that should be treated aggressively with antiemetics and
intravenous hydration

REVISED BY TEAM “BANG JAGO” NOV 2020 UNHAS Page 225


11. A woman with previous stillbirth and postpartum DVT is found to have lupus
anticoagulant and medium-titre Imunoglobulin M (IgM) anticardiolipin antibodies
(aCL) on two occasion. In a subsequent in pregnancy
f. Warfarin should be discontinued
g. She has an increase risk of miscarried
h. She requires antibiotic prophylaxis to cover delivery
i. Low dose aspirin should be discontinued at 34 weeks
j. She doesn,t required postpartum heparin if she has vaginal delivery

12. A 28 years old G1P1 women being discharged from the hospital on postoperative
day 4 after having a primary low transverse caesarean section for breech
presentation, with an estimated blood lost 700ml, her pregnancy was otherwise no
complication and her hospital course was also no complication. Ten day after the
caesarean section, the patient complaining of abdominal pain and fever, Fundal
height 2 finger below navel. What is the most likely propable diagnosis of the
patient?
f. Metritis
g. Mastitis
h. Thyfoid Fever
i. Urinary tract infection
j. Breast engorgement
13. A patient calls your clinic complaining of continued heavy vaginal bleeding, she has
an “uncomplicated” vaginal birth 2 weeks ago of her second child, what is the most
likely diagnosis from the following differentiated?
f. Uterine Atony
g. Uterine Rupture
h. Coagulopathies
i. Vaginal laceration
j. Retained Placenta fragment

REVISED BY TEAM “BANG JAGO” NOV 2020 UNHAS Page 226


A
14. A 22 years old female G1 at 26 weeks gestation, present to the office for her
routine obstetrical visit, currently, she is complaining shortness of breath, she has
no other complaints, on physical examination, blood pressure 100/70 mmhg, PR 90
Bpm, RR 22x/m. no abnormality was found in chest examination. BGA result PH
7,45 ( 7,35 - 7,45), pO2 103 mmhg (75 - 100), PcO2 28 mmHg ( 35-45 ) HCO3 -17
mEq/L (22-26), BE 2 mmol (-2 - 2), O2 sat 99% (¿ 95 % ¿ .What is the result of the
Blood Gas Analysis (BGA) stated above?
f. Normal BGA
g. Metabolic Acidosis
h. Metabolic Alkalosis
i. Respiratory Ascidosis
j. Respiratory Alkalosis
15. A 22 years old female G1 at 26 weeks gestation, present to the office for her
routine obstetrical visit, currently, she is complaining shortness of breath, she has
no other complaints, on physical examination, blood pressure 100/70 mmhg, PR 90
Bpm, RR 22x/m. no abnormality was found in chest examination. BGA result PH
7,45 ( 7,35 - 7,45), pO2 103 mmhg (75 - 100), PcO2 28 mmHg ( 35-45 ) HCO3 -17
mEq/L (22-26), BE 2 mmol (-2 - 2), O2 sat 99% (¿ 95 % ¿ .What is the most
appropriate next management in this patient
f. Ask patient to perform echocardiography examination
g. Ask patient to perform chest radiography examination
h. Admit the patient to the hospital and give non-rebreathing mask
i. Counsel to the patient that this is physiologic changes during pregnancy
j. Terminated the pregnancy because the symptom will became worsen in 32 weeks
pregnancy
16. What is ECG Finding consider normal during pregnancy
f. Spesific ST changes
g. Increased heart rate (15%)

REVISED BY TEAM “BANG JAGO” NOV 2020 UNHAS Page 227


h. Inverted T waves in lead II, LIII dan AVF
i. 15 degreeright axis deviation, Left axis deviasi
j. Irreversible ST waves changes
17. A 32 years old female, G1 at 8 weeks gestation, present to the office for her routine
obstetrical visit. She ask you about the nutrition demand during pregnancy, Her
BMI is 24kg/m2. no remarkable past medical history is noted. According to WHO
Asian criteria, her BMI Classified as:
f. Normal
g. Underweight
h. Overweight
i. Obese type 1
j. Obese type 2
18. A 32 years old female, G1 at 8 weeks gestation, present to the office for her routine
obstetrical visit. She ask you about the nutrition demand during pregnancy, Her
BMI is 24kg/m2. no remarkable past medical history is noted. She ask you what is
the optimal total weight gain during her pregnancy
a. < 5 kg
b. 5-9 kg
c. 7 -11,5 kg
d. 11,5- 16 kg
e. 12,5-18 kg
19. A 32 years old female, G1 at 8 weeks gestation, present to the office for her routine
obstetrical visit. She ask you about the nutrition demand during pregnancy, Her
BMI is 24kg/m2. no remarkable past medical history is noted. What is the most
likely risk of the patient
a. Anemia
b. Congenital anomaly
c. Gestational diabetes
d. Spoontaneus abortion

REVISED BY TEAM “BANG JAGO” NOV 2020 UNHAS Page 228


e. Post partum hemorrhage
20. A 29 year old woman with positive pregnancy test present with a good history of
tissue expulsion vaginaly. A transvaginal ultrasound can shows unempty uterus
with anendometrial thickhnes of 11mm. Regarding her diagnosis,you consider that:
a. She has had a complete miscarriage and needs furthere treatment
b. She has had a pregnancy of unknown locations and needs furthere investigations
c. She should be offered hysteroscopy
d. She should be offered medical management of miscarriage
e. A Laparascopy should be perform to exclude an ectopic pregnancy
21. A 39 years old female G2P1A0, 15 week pregnancy present to your clinic for having
routine ANC. On Physical examination, you found her fundal height equals
umbilicals point.you perform ultrasound and saw a multilocular hipoechoic mass
size 10cm(in diameter) in her lab adnexa.no free fluid in her abdomen and pelvis.
What is your concideration in this case?
a. The incidents of adnexal masses in pregnancy is 1%
b. The incidents of ovarian cancers in pregnancy is between 1:1000
c. The most common type of beningn of ovarian cyst in pregnancy is a mature
teratoma
d. The most common histopatoligical subtype for malignant ovarian tumor in
pregnancy is ephytelial ovarian tumor
e. The resolution rate of adnexal masses in the second trimester of pregnancy is 60-
70%

22. A 28 year woman G1 36 weeks of gestation age,when to your clinic to do routine


ANC. During ultrasound, the doctor told her that she will be expecting baby boy
with estimatd fetal weigh 2500g, however, amniotic fluid considered to be less
than normal. Then you ask the patient to drink minimal of 2L of water a day and
get herself another ultrasould within 3 days to evaluate amniotic fluid. Amiotic

REVISED BY TEAM “BANG JAGO” NOV 2020 UNHAS Page 229


fluid is a balace between production and resortion.What is the primary mechanism
of the fluid resortion?
a. Fetal breathing
b. Fetal swallowing
c. Absorbtion a cross fetal skin
d. Absortion by fetal kidney
e. Filtration by fetal kidney

23. Mrs B, 37 years old come to your office at 32 week of gestation according to her
last menstrual period. She has no ultrasound examination before and did not do
her routine ANC the vital sign is within normal limit. She has BMI 19kg/m2. During
physical examination the uterine fundal heigt is 27 cm. From ultrasound
examimnation,the fetus has biometri values that correlated with 30 weeks fetus.
Which is the following is the next best step in manging this patient
a. ANC routine for the next 2 weeks
b. Evaluate maternal status and commorbities
c. Conciders deliver the baby
d. Repat sonograpy For fetal growth in 2 weeks
e. Doppler velocimetry evaluation every 3 days
24. According to algoritm for management of fetal growth restriction,you evaluated
the Doppler velocimentry than find reseve and diastolic flow
oligohydroamnios.What is the appropriate management at this time
a. Regular fetal testing
b. Weekly evaluation of amniotic fluid
c. Concider corticosteroid for lung maturation
d. Delivey the baby
e. Reevaluated middle cerebral arteries and ductus venosus

REVISED BY TEAM “BANG JAGO” NOV 2020 UNHAS Page 230


25. If the symptoms were worsening to cardiac compromise, ie. symptomatic at rest
with dyspnea, chest pain, or presyncope, your advice is
a. High fiber diet
b. Red cell transfusion
c. Adjunct ascorbic acid
d. Oral iron supplementation
e. Parenteral iron replacement

REVISED BY TEAM “BANG JAGO” NOV 2020 UNHAS Page 231


26. A 36-year-old para 3 woman attends day assessment unit a er a growth scan for
large for gestational age at 28 weeks' gestation. She had three previous normal
vaginal deliveries and was low risk at booking. She declined 1st trimester screening.
but had normal dating and anomaly scans. Growth scan showed polyhydramnios.
small for gestation foetus with a double bubble sign, and she was referred to the
foetal medicine unit by the sonographers. Which one of the aneuploidies is the
foetus more likely to have?
a. Down syndrome
b. Edwards syndrome
c. Klinefelter syndrome
d. Patau syndrome
e. Turner syndrome
27. Which of the following imaging finding increased suspicion of placenta accreta?
a. A small intraplacental lakes (dark intraplacental band)
b. A distance less than 5 mm between uterine serosa and retroplacental vessel
c. Heterogenous signal intensity within the placenta on MRI examination
d. Presence of no intraplacental bands on T2-weighted imaging
e. Uterine bulging in ultrasound

REVISED BY TEAM “BANG JAGO” NOV 2020 UNHAS Page 232


NOVEMBER 2019

1. You found on Leopold 1, hard, round with ballotement (+). Contraction was infrequent
and weak. A what was your next plan?
A. went for labour induction
B. Immediate C-Section
C. Tocolyctic and corticosteroid provision
D. Performed ultrasound
E. Performed external version

2. A maternal fetal medicine specialist is consulted and performs an indepth sonogram. The
sonogram indicates that the fetuses are both male, and the placenta apears to be diamniotic
and monochorionic. Twin B is noted to have oligohydramnios and to be much smaller than
twin A. In this clinical picture, all of the follwoing are concerns for twin A except
A. Congestive heart failure
B. Anemia
C. Hypervolemia
D. Polycitemia
E. Hydramion

3. You are called in to evaluate the heart of a 19-year-old primigravida at term.


Listening carefully to the heart, you determine that there is a split Sl, normal S2, S3
easily audible With a 2/6 systolic ejection murmur greater during inspiration, and a soft
diastolic murmur. You immediately recognize that
a. The presence of the S3 is abnormal
b. The systolic ejection murmur is unusual in a pregnant woman at term
c. Diastolic murmurs are rare In pregnant women
d. The combination of a prominent S3 and soft diastolic murmur is a significant abnormality

REVISED BY TEAM “BANG JAGO” NOV 2020 UNHAS Page 233


e. All findings recorded are normal changes in pregnancy

A 25-year-old woman in her first pregnancy is noted to have prolonged first and
second stages of labour. She was induced at 38 weeks' pregnancy. The baby was
delivered by forceps. After delivery the placenta she is noted to have heavy vaginal
bleeding. Abdominal examination demonstrates a relaxed uterus.
4. What is the most likely cause of
bleeding. A Uterine atony
B. Uterine rupture
C. Retained placenta
D. Genital tract laceration
E. DIC

5. What should we do if the fundus not firm after placental delivery.


A. Methylergonovine (Methergine)
B. Carboprost (Hemabate, PGF2-alpha)
C. Fundal Massage
D. Misoprostol (PGEI)
E. Dinoprostone-prostaglandin E2

A 30-year-old multiparous woman has rapid delivery soon after arriving in emergency
room. After delivery the placenta she is noted to have heavy vaginal bleeding. Help has
been summoned. Abdominal examination demonstrates the fundus was soft.
6. What is the most appropriate next step?
A. Intravenous access for fluid resuscitation
B. Uterine packing
C. Balloon tamponade
D. Suture the laceration
E. Misoprostol administration

7. After use of a 20 units of oxytocin in 1000 mL of crystalloid solution to increase the tone
of her uterus stop the bleeding; however, you continue to notice a massive bleeding from
the vagina. What is the most appropriate next step in the evaluation of this patient's
bleeding?
A. Perform a bedside ultrasound for retained products of conception
B. Perform a ultrasound to look for blood in the abdomen significant for uterine rupture
C. Perform a manual exploration of the uterine fundus and exploration for retained clots
or products
D. Examine the perineum and vaginal for laceration during delivery
E. Consult interventional radiology for uterine artery embolization

REVISED BY TEAM “BANG JAGO” NOV 2020 UNHAS Page 234


8. Labor induction and augmentation are NOT associated with which Of the following risk?
A. postpartum hemorrhage from uterine atony is more common in women undergoing
induction or augmentation
B. Amniotic fluid embolism in a laboring patient receiving oxytocin can be occurred
C. The increased risk for cesarean delivery undergoing induction is related with cervical
favorability (Bishop Score)
D. The uterine rupture risk is increased threefold for women in spontaneous labor with
uterine Scar
E. Women Whose labor is managed with amniotomy have lower incidence of
chorioamnionitis compared with those in spontaneous labor

A 32-years-old woman comes to your clinics due to shortness of breath, that worsen
since 2 days ago. On history taking, she told you that she had ever diagnosed of having
significant mitral stenosis. She is 33 weeks pregnant. The fetus is size-date appropriate.
She has had a recent echocardiography showing ejection fraction of 54% with
moderate-severe pulmonary hypertension.
9. What is the best management for this patient currently?
A. Perform emergency C section
B. Lung maturation and C section
C. Conservative management until term pregnancy
D. Second stage acceleration
E. Induction of labor

10. What is the most common cause Of heart failure during pregnancy and the puerperium?
A. Chronic hypertension with severe preeclampsia
B. Viral myocarditis
C. Obesity
D. Valvular heart disease
E. Pulmonary Artery Hypertension

11. For patients with congenital heart disease, what is the most common adverse
cardiovascular event encountered in pregnancy?
A. Heart failure
B. Arrhythmia
C. Thromboembolic event
D. Cerebrovascular hemorrhage
E. Heart axis changes

REVISED BY TEAM “BANG JAGO” NOV 2020 UNHAS Page 235


12. Corticosteroids administered to women at risk for preterm birth have been
demonstrated to decrease rates Of neonatal respiratory distress if the birth is delayed
for at least what amount of time after the initiation of therapy?
A. 12 hours
B. 24 hours
C. 36 hours
D. 48 hours
E. 72 hours

13. A 24-year-old patient, P2, has just delivered vaginally an infant weighing 3000 g after
a spontaneous uncomplicated VBAC. Her prior obstetric history was a low uterine
segment transverse cesarean section for breech. She has had no problems during the
pregnancy and labor. The placenta delivers spontaneously. There is immediate vaginal
bleeding of greater than 500 cc. Although all of the following can be the cause for
postpartum hemorrhage, which is the most frequent cause of immediate hemorrhage as
seen in this patient?
A. Uterine atony
B. Coagulopathies
C. Uterine rupture
D. Retained placental fragments
E. Vaginal and/or cervical lacerations

14. A 89-year-old female patient with multiple, serious medical comorbidities presents
to discuss options for treatment of her high-grade prolapse. The prolapse is externalized
and becoming ulcerated from friction against her undergarments. She cannot tolerate a
pessary. Her main priority is to "fix or get rid of this thing: but her primary care provider
has cautioned against a lengthy or open abdominal procedure. She is not interested in
future intercourse. what can you offer this patient?
A. Nothing can be done
B. Open abdominal sacral colpopexy
C. Robot-assisted laparoscopic sacral colpopexy
D. Hysterectomy with anterior and posterior colpomhaphy, vault suspension.
E. Colpocieisis

82 years old woman P6 came to outpatient clinic with chief complaint of bulging mass
rotrudes from vagina since 3 months ago. The mass usually occurs during activity and
also 'hen she defecate , and disappear when lying down. There were no difficulty in
voiding and efecation. No urinary leakage during coughing and sneezing. She is not
sexually active.

REVISED BY TEAM “BANG JAGO” NOV 2020 UNHAS Page 236


15. If the patient choose to use pessary instead of surgery, how to choose the right size
of the pessary?
A. The smallest size that do not fall off and doesn't cause pain and obstruction of urination
and defecation
B. The biggest size that do not fall off. but doesn't cause pain and obstruction of urination
and defecation
C. The intermediate size that do not fall off, but doesn't cause pain and obstruction of
urination and defecation
D. By measuring the genital hiatus of the patient
E. By measuring the total vaginal length

Mrs. N, 37 years old with chief complain of infertility for 6 years with history of severe
dysmenorrhea. From hysterosalpingography, both tubes were non-patent. Pelvic ultrasound
found bilateral cystic mass with internal echo sized 50 and 60 mm in diameter. Her husband
sperm examination was within normal limit.
16. What is the next appropriate management?
a. Offer IUI
b. Perform laparoscopy cystectomy and adhesiolysis
c. Give GnRH analog for 3 months continue with IUI
d. Give Dinogest 1x2 mg for 6 months
e. Offer her IVF

17. Her AMH level was 0,9 ng/ml. What is the reason for performing surgery in subfertile
patient with bilateral endometrioma and diminished ovarian reserve?
a. Removal endometrioma
b. Ablation of all endometriosis lesion
c. To prevent infection in endometrioma
d. To improve access for follicle aspiraton
e. Removal of deep infiltrating endometriosis

18. Which of the following is true regarding low ovarian reserve in endometriosis?
a. Ovulation rate in ovary with endometrioma is higher compared to ovary without
endometrioma
b. There is higher density of follicle in ovary with endometrioma
c. Ovary with endometrioma has higher response rate to gonadotropin
d. Loss of ovarian stromal appearance and fibrosis are present in ovarian cortex with

REVISED BY TEAM “BANG JAGO” NOV 2020 UNHAS Page 237


endometrioma
e. Low ovarian reserve in endometriosis only happen after surgery

A 18-year old adolescent female complains of not having started her menses. Her breast
development is Tanner stage IV, pubic hair development was stage I. from vaginal examination
found a blind vaginal pouch and no uterus and cervix.
19. Which of the following describes the most likely diagnosis?
a. Partial androgen insensitivity syndrome
b. Complete androgen insensitivity syndrome
c. Kallman syndrome
d. Turner syndrome
e. Polycystic ovarian syndrome

20. From ultrasound examination found no uterus and there was difficulty in identifying in the
gonads. What is the next plan?
a. Prolactin measurement
b. Kariotyping
c. FSH and LH examination
d. FSH, LH, dan E2 examination
e. TSH, fT4 examination

21. Which of the following management will be appropriate for this condition?
a. Give progestin 14 days on off
b. Give estrogen-progestin sequential
c. Give combined oral contraception
d. Vaginal reconstructive surgery
e. Laparoscopy gonad removal

An-18 year-old young woman presents to you with complaint of amenorrhea. She notes that
she has never had a menstrual period, but that she had mild cyclic abdominal bloating. She is
sexually active, but she complains of painful sexual intercourse. Her past medical and surgical
history is unremarkable. On physical examination, you note normal appearing axillary and
pubic hair. Her breast development is normal. Pelvic examination reveals normal appearing
external genitalia, and a shortened vaginal ending in a blind pouch.

22. Which of the following test would be your first step in determining the diagnosis?
a. Karyotype
b. Pelvic ultrasound
c. Serum FSH

REVISED BY TEAM “BANG JAGO” NOV 2020 UNHAS Page 238


d. Serum FSH, E2
e. Diagnostic laparoscopy

23. From further examination it was found that uterus cannot be visualized both both ovaries
were normal. What is the most likely diagnosis?
a. Imperforate hymen
b. Transverse vaginal septum
c. Mullerian agenesis
d. Androgen insensitivity dynrome
e. Gonadal dysgenesis

24. Which additional organ system should you be evaluating in patient with this disorder?
a. Pancreas and duodenum
b. Cerebral circulation
c. Olfactory system
d. Renal and urinary collecting system
e. Distal gastrointestinal tract
A 34 year old female, para 1, presented to our clinic with secondary amenorrhea and severe,
progressive hirsutism. On clinical examination she was noted to have severe hirsutism and
male-pattern scalp balding. Her BMI was 30 kg/m2. Laboratory results showed an elevated
total testosterone (T) level of 140 ng/dl (reference value in our laboratory is 0-80 ng/dl) and
androstenendione of 272 ng/dl (reference value of 30-250 ng/dl). CT of the abdomen and
pelvis showed normal adrenal glands. Pelvic ultrasound of the pelvis demonstrated mildly
prominent ovaries, containing numerous small follicle around periphery.
25. What is your most probable diagnosis?
a. Multicystic ovary
b. Congenital adrenal hyperplasia
c. Polycystic ovary syndrome
d. Hyperprolactinemia
e. Microadenoma pituitary

26. Your diagnosis according to ASRM/ESHRE definition, based on two of the following criteria:
a. Polycystic ovaries on ultrasound, oligo or amenorrhea, or evidence of hyperandrogenism
b. Polycystic ovaries on ultrasound, amenorrhea, obesity
c. Polycystic ovaries on ultrasound, amenorrhea, hirsutism
d. Presence of hyperandrogenism, ovarian dysfunction and exclusion of related disorders
e. Polycystic ovaries on ultrasound, hirsutism, obesity

A 46 years old woman experiences irregular vaginal bleeding of 3 months duration. You

REVISED BY TEAM “BANG JAGO” NOV 2020 UNHAS Page 239


perform an endometrial biopsy, which obtains copious tissue with a velvety, lobulated
textrure. The pathologist report shows proliferation of glandular and stroma elements with
dilated endometrial glands, consistent with simple hyperplasia. Cytologic atypia is absent.
27. Which of the following is the best way to advise the patient?
a. She should be treated to estrogen and progestin hormone therapy
b. The tissue will progress to cancer in approximately 10% of cases
c. The tissue may be weakly premalignant and progresses to cancer in approximately 1% of
cases
d. She requires a hysterectomy
e. No further therapy is needed

28. She agreed for a medical treatment, which of the following is the most appropriate?
A. Norethisterone acetate 1x5 mg for 14 days on-off
B. MPA 1x2,5 mg for 14 days on-off
C. Nomegestrek 1x2,5 mg for 14 days on-off
D. LNG IUS
E. Combined oral contraception

Unas Mei 2020 UNHAS (B) Gabungan


REVISED BY TEAM “BANG JAGO” NOV 2020 UNHAS Page 240
A 82 years old woman P6 came to outpatient clinic with chief complaint of bulging mass
protrudes from vagina since 3 months ago. The mass usually occurs during activity and also
when she defecate , and disappear when lying down. There were no difficulty in voiding and
defecation. No urinary leakage during coughing and sneezing. She is not sexually active.

a. If on the Pelvic Organ Prolapse Quantification examination result showing below, what is
the diagnosis of this patient?

Aa Ba C
+3 +4 +5
GH Pb TVL
5 2 8
Ap Bp D
0 0 +3

a. Uterine prolapse grade 2, cystocele grade 2, rectocele grade 1


b. Uterine prolapse grade 4, cystocele grade 3, rectocele grade 2
c. Uterine prolapse grade 3, cystocele grade 3, rectocele grade 2
d. Uterine prolapse grade 3, cystocele grade 2, rectocele grade 2
e. Uterine prolapse grade 4, cystocele grade 4, rectocele grade 3

If the patient choose to use pessary instead of surgery, how to choose the right size of the
pessary?
The smallest size that do not fall off and doesn’t cause pain and obstruction of urination and
defecation
The biggest size that do not fall off, but doesn’t cause pain and obstruction of urination
and defecation

REVISED BY TEAM “BANG JAGO” NOV 2020 UNHAS Page 241


The intermediate size that do not fall off, but doesn’t cause pain and obstruction of
urination and defecation
By measuring the genital hiatus of the patient
By measuring the total vaginal length

REVISED BY TEAM “BANG JAGO” NOV 2020 UNHAS Page 242


33. Women with one or more previous caesarean section scars and an anterior placenta are at
of placenta accreta. Which test has been shown in recent research to provide the highest
sensitivity and specificity for antenatal diagnosis of placenta accreta?
a. Colour Doppler
b. 3D Power Doppler
c. Contrast CT
d. Gadolinium Contrast MRI
e. Grey Scale Ultrasound

34. A 22 year old unbooked primigravida presents to the Emergency Department at 26 weeks of
gestation with a history of spontaneous painless bleeding af about 500 ml. What is the best
investigation to secure a diagnosis?
a. MRI scan
b. Transabdominal scan
c. CTG
d. Transvaginal scan
e. CT scan

35. A primigravida at 35 weeks of gestation presents with pain in the right hypochondrium and
right side of her back. There is no history of nausea or vomiting, hypertension, urinary
symptoms and bowel problems. Vital signs: pulse-106, temperature 38.1, BP 128/75.

REVISED BY TEAM “BANG JAGO” NOV 2020 UNHAS Page 243


Abdominal examination is unremarkable. Chest is clear. Fetal monitoring is normal. Urine
shows 2+ leucocytes and 1+ blood. White cell count 16 x 10 9/L. What is the most likely
diagnosis?
1. Appendicitis
2. Cholecystitis
3. Pyelonephritis
4. Abruption
5. Right basal pneumonitis

36. A 35-year old P2 is 36 weeks pregnant. Clinically there is a suspicion of left calf DVT. CTG is
normal. What is the next step in the immediate management?
A. Plan delivery
B. Therapeutic dose of tinzaparin
C. Prophylactic dose of tinzaparin
D. FBC, coagulation screen, LFTs
E. Thrombophilia screen

37. A 25-year-old G1P0 presents to the emergency room with vaginal bleeding. Her last normal
menstrual period was 6 weeks earlier. She reports that she is sexually active with male
partners and does not use any hormonal or barrier methods for contraception. On arrival,
her temperature is 37°C, blood pressure is 115/80, pulse is 75 beats per minute, respiratory
rate is 16 breaths per minute, and she has 100% oxygen saturation on room air. A pelvic
examination reveals a small amount of dark blood in the vagina. The external cervical os
appears 1 to 2 cm dilated. Her uterus is mildly enlarged, anteverted, and nontender. A urine
pregnancy test is positive. A pelvic ultrasound is obtained and shows an intrauterine
gestational sac with a yolk sac. No fetal pole or cardiac motion is seen. Bilateral adnexa are
normal. What is her diagnosis?
A. Incomplete abortion
B. Threatened abortion

REVISED BY TEAM “BANG JAGO” NOV 2020 UNHAS Page 244


C. Ectopic pregnancy
D. Missed abortion
E. Inevitable abortion

38. During a routine return OB visit, an 18-year-old G1P0 patient at 23 weeks gestational age
undergoes a urinalysis. The dipstick done by the nurse indicates the presence of trace
glucosuria. All other parameters of the urine test are normal. Which of the following is the
most likely etiology of the increased sugar detected in the urine?
a. The patient has diabetes
b. The patient has a urine infection
c. The patient’s urinalysis is consistent with normal pregnancy
d. The patient’s urine sample is contaminated
e. The patient has kidney disease

39. A maternal fetal medicine specialist is consulted and performs an indepth sonogram. The
sonogram indicates that the fetuses are both male, and the placenta appears to be
diamniotic and monochorionic. Twin B is noted to have oligohydramnios and to be much
smaller than twin A. In this clinical picture, all of the following are concerns for twin A
except
A. Congestive heart failure
B. Anemia
C. Hypervolemia
D. Polycythemia
E. Hydramnion

40. You are called in to evaluate the heart of a 19-year-old primigravida at term. Listening
carefully to the heart, you determine that there is a split S1, normal S2, S3 easily audible
with a 2/6 systolic ejection murmur greater during inspiration, and a soft diastolic murmur.
You immediately recognize that
a. The presence of the S3 is abnormal

REVISED BY TEAM “BANG JAGO” NOV 2020 UNHAS Page 245


b. The systolic ejection murmur is unusual in a pregnant woman at term
c. Diastolic murmurs are rare in pregnant women
d. The combination of a prominent S3 and soft diastolic murmur is a significant
abnormality
e. All findings recorded are normal changes in pregnancy

A 25-year-old woman in her first pregnancy is noted to have prolonged first and second stages
of labour. She was induced at 38 weeks’ pregnancy. The baby was delivered by forceps. After
delivery the placenta she is noted to have heavy vaginal bleeding. Abdominal examination
demonstrates a relaxed uterus.

41. What should we do if the fundus not firm after placental delivery.
a. Methylergonovine (Methergine)
b. Carboprost (Hemabate, PGF2-alpha)
c. Fundal Massage
d. Misoprostol (PGE1)
e. Dinoprostone-prostaglandin E2

A 30-year-old multiparous woman has rapid delivery soon after arriving in emergency room.
After delivery the placenta she is noted to have heavy vaginal bleeding. Help has been
summoned. Abdominal examination demonstrates the fundus was soft.

42. What is the most appropriate next step?


a. Intravenous access for fluid resuscitation
b. Uterine packing
c. Balloon tamponade
d. Suture the laceration
e. Misoprostol administration

REVISED BY TEAM “BANG JAGO” NOV 2020 UNHAS Page 246


43. For patients with congenital heart disease, what is the most common adverse
cardiovascular event encountered in pregnancy?
a. Heart failure
b. Arrhythmia
c. Thromboembolic event
d. Cerebrovascular hemorrhage
e. Heart axis changes

44. A 24-year-old patient, P2, has just delivered vaginally an infant weighing 3000 g after a
spontaneous uncomplicated VBAC. Her prior obstetric history was a low uterine segment
transverse cesarean section for breech. She has had no problems during the pregnancy and
labor. The placenta delivers spontaneously. There is immediate vaginal bleeding of greater
than 500 cc. Although all of the following can be the cause for postpartum hemorrhage,
which is the most frequent cause of immediate hemorrhage as seen in this patient?
a. Uterine atony
b. Coagulopathies
c. Uterine rupture
d. Retained placental fragments
e. Vaginal and/or cervical lacerations

A 22-year-old G1A0 at 10 weeks presents for her scheduled obstetric (OB) appointment.
Laboratory examination showed hemoglobin 11.2 g/dL, hematocrit 34%, MCV 86 fl MCH 32 pg.

a. Which of the following shows iron deficiency anemia:


a. Low ferritin, normal serum iron
b. Low ferritin, serum iron and transferrin
c. Normal hemoglobin level, low ferritin and serum iron

REVISED BY TEAM “BANG JAGO” NOV 2020 UNHAS Page 247


d. Microcytic hypochromic, low serum iron and transferrin saturation
e. Microcytic hyperchromic, low serum iron and transferrin saturation

b. What is total iron need during pregnancy?


A. 500 mg
B. 750 mg
C. 1000 mg
D. 1500 mg
E. 2000 mg

A 22-year-old primiparous woman presents for her first prenatal evaluation. On physical
examination you hear a grade 3/6 pansystolic murmur.

c. Which is the most common CHD in pregnancy that would cause that type of murmur?
a. Aortic stenosis
b. Pulmonary stenosis
c. Atrial septal defect (ASD)
d. Patent ductus arteriosus (PDA)
e. Ventricular septal defect (VSD)

d. Which of the following mother’s cardiac abnormalities is the greatest risk for fetal
structural congenital cardiac disease?
a. Aortic coarctation
b. Tetralogy of Fallot
c. Atrial septal defect
d. Pulmonary stenosis
e. Ventricular septal defect

REVISED BY TEAM “BANG JAGO” NOV 2020 UNHAS Page 248


66. The following statement is true regarding the physiological adaptations to pregnancy?
A. The increase in maternal heart rate contributes to an increase in cardiac output during
pregnancy.( 10x meningkat, CO di usia kehamilan 22-24 minggu sebesar 30-5-%)
B. Mean arterial blood pressure falls because of a rising in systemic vascular resistance.
C. Anatomical and physiological changes in the lungs allow a pregnant woman to withstand
hypoxia better than a non-pregnant woman.
D. The lower bicarbonate levels in pregnant women reflect a state of metabolic acidosis.
E. TSH (thyrotropin) levels fall in the first trimester but returns fastly to normal by term.

67. A 29 year old woman with a positive pregnancy test presents with a good history of tissue
expulsion vaginally passing tissue per vagina. A transvaginal ultrasound scan shows an
empty uterus with an endometrial thickness of 11 mm. Regarding her diagnosis, you
consider that :
A. She has had a complete miscarriage and needs no further treatment
B. She has had a pregnancy of unknown location and needs further investigations
C. She should be offered a hysteroscopy
D. She should be offered medical management of miscarriage
E. A laparoscopy should be performed to exclude an ectopic pregnancy

68. Which of the following is true about the screening and diagnosis of spina bifida?
A. A maternal serum alpha-fetal protein (MSAFP) of 2.5 multiples of the median (MoM) is
diagnostic of an NTD.(x)
B. First-trimester ultrasound screening for NTDs primarily involves identification of a
normal posterior fossa during aneuploidy screening to rule out the presence of a
Arnold-Chiari II malformation.
C. Magnetic resonance imaging (MRI) has proven to be a better diagnostic tool for NTDs
than ultrasound.(X)
D. The banana sign is indicative of ventriculomegaly, which is present in most cases of
open NTDs. ( gatau)
E. Diagnosis of lesion level requires three-dimensional ultrasound or MRI modalities. (x)

69. On prenatal ultrasound, which of the following feature characterise gastroschisis?


A. The abdominal wall defect is superior to cord insertion.
B. Ectopia cordis is present.
C. The abdominal wall defect is lateral to cord insertion.
D. The abdominal wall defect is lower than cord insertion.
E. The bladder cannot be visualised.

70. A 39 years old female G2P1A0, 15 weeks pregnant presents to your clinic for having routine
ANC. On physical examination, you found her fundal height equals umbilical point. You
performed ultrasound and saw a multilocular hypoechoic mass sized 10 cm (in diameter) in

REVISED BY TEAM “BANG JAGO” NOV 2020 UNHAS Page 249


her left adnexa. No free fluid in her abdomen and pelvis. What is your consideration in this
case?
A. The incidence of adnexal masses in pregnancy is 1%
B. The incidence of ovarian cancers in pregnancy is between 1:1000
C. The most common type of benign ovarian cyst in pregnancy is a mature teratoma
D. The most common histopathological subtype for malignant ovarian tumor in
pregnancy is epithelial ovarian tumor
E. The resolution rate of adnexal masses in the second trimester of pregnancy is 60-70%

71. The patient does not believe that she has ovarian cyst during pregnancy. She is really
concern about the possibility of malignancy. Regarding this situation, what would you
inform her?
A. The most common mode of presentation of an adnexal mass is pain
B. The sensitivity of detection of ovarian cysts on clinical examination alone is less than 5%.
C. The size of ovarian cyst that should prompt investigation for malignancy is 10 cm
D. The validated sensitivity and specificity of IOTA rules on ultrasound evaluation of an
ovarian cyst is sensitivity: 78%, specificity: 87%
E. The sensitivity and specificity of MRI in the diagnosis of a malignancy is 100 and 94%
respectively

72. You have checked her CA125 serum level and the result was 350 µ/ml. You performed
conservative surgical staging by laparotomy because her frozen section revealed malignancy
cyst. After 1 week, pathology result serous papillary carcinoma of the left ovary. She is
planned for chemotherapy. What will you inform to her regarding chemotherapy for
ovarian cancer during pregnancy?
A. In a patient with ovarian cancer in pregnancy receiving chemotherapy the delivery
should be planned at completion of chemotherapy
B. Use of chemotherapy in pregnancy generally considered safe after 20 weeks of
gestation
C. CNS and neural tube complications occur during the week 8-12 weeks in pregnancy
D. This percentage of patients receiving chemotherapy in pregnancy who develop major
congenital malformations is 30-40%
E. Cardiovascular defects are common congenital malformations in platin based
chemotherapy regimens

A 28 yo woman, G1 36 weeks of gestational age, went to your clinic to do routine antenatal


care. During ultrasound, the doctor told her that she will be expecting baby boy with estimated
fetal weight 2500 g, however, amniotic fluid considered to be less than normal. Then you asked
the patient to drink minimal of 2L of water a day and get herself another ultrasound within 3
days to evaluate the amniotic fluid.

73. Oligohydramnios is defined as which of the following?


A.Amniotic fluid index < 5 cm

REVISED BY TEAM “BANG JAGO” NOV 2020 UNHAS Page 250


B.Single deepest pocket < 2 cm
C.Amniotic fluid index < 90th percentile
D. All of the above
E. None of the above

74. Amniotic fluid volume is a balance between production and resorption. What is the primary
mechanism of fluid resorption?
A. Fetal breathing
B. Fetal swallowing
C. Absorption across fetal skin
D. Absorption by fetal kidneys
E. Filtration by fetal kidneys

75. In a normal fetus at term, what is the daily volume of fetal urine that contributes to the
amount of amniotic fluid present?
A. 200 mL
B. 250 mL
C. 500 mL
D. 750 mL
E. 1000 mL

Mrs A, 26-years-old, G1P0A0, according to her last LMP is 34 weeks pregnant, came for her first
antenatal care. She said that she had 20 kg of weight gain during her pregnancy with swelling
ankles for the past 4 weeks. She never took any iron or vitamin supplementation. From the
physical findings, BP 145/95 mmHg, HR 86x/min, RR 20x/min, BMI 35 kg/m 2. Ultrasound
examination confirmed twins in breech presentation. Results from urinalysis were as follows:
color cloudy yellow, spesific gravity 1.013, albumin 2+, RBC 0 - 1, WBC 2-5, negative bacterial
count.

76. What is the most likely diagnosis?


A. Acute fatty necrosis of the liver
B. Chronic hypertension
C. Preeclampsia
D. Renal disease
E. Pyelonephritis

Mrs. B, 37-years-old came to your office at 32 weeks of gestation according to her last
menstrual period. She has no ultrasound examination before and did not do her routine
antenatal care. The vital sign is within normal limit. She has body mass index 19 kg/m2. During
physical examination, the uterine fundal height is 22 cm. From ultrasound examination, the
fetus has biometric values that correlate with 30 weeks fetus.

REVISED BY TEAM “BANG JAGO” NOV 2020 UNHAS Page 251


77. Which of the following is the next best step in managing this patient?
A. Antenatal care routinely for the next 2 weeks
B. Evaluate maternal status and comorbidities
C. Consider deliver the baby
D. Repeat sonography for fetal growth in 2 weeks
E. Doppler velocimetry evaluation every 3 days

78. According to algorithm for management of fetal-growth restriction, you evaluate the
Doppler Velocimetry then find reversed end-diastolic flow and oligohydramnios. What is the
appropriate management at this time?
A. Regular fetal testing
B. Weekly evaluation of amniotic fluid
C. Consider corticosteroids for lung maturation
D. Deliver the baby
E. Reevaluate middle cerebral arteries and ductus venosus

79. Fetal growth restriction is associated with all of the following, EXCEPT ( dari william )
A. Antiphospholipid Antibody Syndrome
B. Inherited Thrombophilias
C. Infertility
D. Immunosuppressive drugs
E. Social deprivation

Mrs E, 32 yo referred from midwife with antepartum hemorrhage. She is G3P2 term pregnancy.
On examination her blood pressure is 160/100 mmHg, HR 100 bpm. She looks anemic, not
icteric. Obstetrical examinations reveal contraction 4-5x/10 minutes, FHR 170 bpm, head
presentation 3/5. After thorough examination it is concluded that there is a placental
abruption with retroplacental hematoma size 6x5 cm. This patient is planned to do cesarean
section.

80. If during operation the uterus is couvelaire but with good contraction, how would you
manage that condition?
A. Perform prophylactic b-lynch suture
B. Ascending uterine artery ligation
C. Hypogastric artery ligation
D. Sub total hysterectomy
E. Uterotonic and observation

A 33-year-old woman, G1P0A0, came to hospital with major complaint watery leakage. She was
on her 33 weeks of gestational age. Data from medical record showed that she came previously
a week ago, complaining vaginal discharge. Vaginal swab has done.

REVISED BY TEAM “BANG JAGO” NOV 2020 UNHAS Page 252


81. In case above, what kind of examination should you performed for establishing diagnosis.
A. Vaginal examination
B. Inspeculo
C. Blood test
D. Ultrasound
E. Simple urine test

82. You found on Leopold 1, hard, round with ballottement (+). Contraction was infrequent and
weak. A What was your next plan?
A. Went for labour induction
B. Immediate C-section
C. Tocolytic and corticosteroid provision
D. Performed ultrasound
E. Performed external version

83. Lack of baby movement had been felt for two days, fetal heart rate was 146 bpm. What was
your next step?
A. Termination of pregnancy
B. Giving oxygenation and left lateral position
C. Ensuring Fetal well-being by Manning criteria
D. Fetal lung maturation
E. Giving intravenous fluid rehidration.
84. Cardiotocography, showed low variability with checkmark pattern and no desceleration.
What was your interpretation and the best management through?
A. Category one, continued for fetal lung maturation
B. Category two, intrauterine resuscitation for 24 hours and reevaluation after
C. Category two, went for doppler velocymetri
D. Category three, went for doppler velocymetry ultrasound exam.
E. Category three, delivered the baby

85. A 24-year-old G2P1 woman at 39 weeks and 3 days is seen in clinic. She has been
experiencing more frequent contractions and thinks she might be in labor. Her last
pregnancy ended with a cesarean delivery after a stage 1 arrest. There was no evidence of
cephalopelvic disproportion. Earlier in the course of her current pregnancy she had desired
a scheduled repeat cesarean, but now that she might be in labor she would like to try and
delivery vaginally.
What would be a contra indication to a trial of labor after cesarean (TOLAC)?
A. Prior classical hysterotomy
B. Prior Kerr hysterotomy
C. Small for gestational age fetus
D. Oligohydramnios
E. GBS + mother

REVISED BY TEAM “BANG JAGO” NOV 2020 UNHAS Page 253


86. A 24-year-old women at 32 weeks’ gestation complains of shortness of breath during her
pregnancy, especially with physical exertion. She has no prior medical history. Her
respiratory rate is 16x/m; her lungs are clear to auscultation; and your office oxygen
saturation monitor reveals her oxygen saturation to be 98% on room air. You reassure her
that this sensation is normal and explain which of the following?
A. Pulmonary resistance increases during pregnancy.
B. Airway conductance is decreased during pregnancy.
C. Small amniotic fluid emboli are shed throughout pregnancy.
D. Maximal breathing capacity is not altered by pregnancy. ( LANGE 99)
E. Because of enlarging uterus pushing up on the diaphragm, her vital capacity is
decreased by 20%.

87. You are counseling a couple in your clinic who desire VBAC. Her baby is in a vertex
presentation, appropriate size for 37 weeks, and her previous low transverse procedure
was for breech presentation. You have to give inform consent about VBAC. In providing
informed consent, in which of the following ways do you explain the risk of uterine rupture?
A. Less than 1% ( RCOG 1x less 1%)
B. Between 2% and 5 %
C. Between 15-20%
D. Depend on the length of her labor
E. Depend on the location and proximity of the scar site to the placental implantation

88. Corticosteroids administered to women at risk for preterm birth have been demonstrated
to decrease rates of neonatal respiratory distress if the birth is delayed for at least what
amount of time after the initiation of therapy?
A. 12 hours
B. 24 hours
C. 36 hours
D. 48 hours
E. 72 hours

89. A 89-year-old female patient with multiple, serious medical comorbidities presents to
discuss options for treatment of her high-grade prolapse. The prolapse is externalized and
becoming ulcerated from friction against her undergarments. She cannot tolerate a pessary.
Her main priority is to “fix or get rid of this thing,” but her primary care provider has
cautioned against a lengthy or open abdominal procedure. She is not interested in future
intercourse. What can you offer this patient?
A. Nothing can be done
B. Open abdominal sacral colpopexy
C. Robot-assisted laparoscopic sacral colpopexy
D. Hysterectomy with anterior and posterior colporrhaphy, vault suspension.
E. Colpocleisis

REVISED BY TEAM “BANG JAGO” NOV 2020 UNHAS Page 254


A 46 years old woman experiences irregular vaginal bleeding of 3 months duration. You
perform an endometrial biopsy, which obtains copious tissue with a velvety, lobulated texture.
The pathologist report shows proliferation of glandular and stromal elements with dilated
endometrial glands, consistent with simple hyperplasia. Cytologic atypia is absent.

92. Which of the following is the best way to advise the patient?
A. She should be treated to estrogen and progestin hormone therapy.
B. The tissue will progress to cancer in approximately 10% of cases.
C. The tissue may be weakly premalignant and progresses to cancer in approximately
1% of cases.
D. She requires a hysterectomy.
E. No further therapy is needed.

93. A 7-year-old girl presents to her pediatrician with her parents who are concerned about
her early sexual development. She is developing breasts, axillary hair, and pubic hair, and
they are noticing body odor. A thorough clinical workup reveals the child has an irregular,
echogenic, thickly septated ovarian mass on her left ovary. What type of tumor is
responsible for this child's clinical presentation?
A. Dysgerminoma
B. Embryonal carcinoma
C. Sertoli-Leydig cell tumor
D. Endodermal sinus tumor
E. Granulosa-theca cell tumor

94. What is the cause of necrotic and degenerative process in fibroids?


A. Mitotic activity
B. Limited blood supply within tumors ( pembuluh darah nya di perifer)
C. Chromosomal defects
D. Hyper perfusion
E. Cytogenetic mutations

95. Childhood neoplastic ovarian masses most commonly originate from:


A. Gonadal epithelium
B. Gonadal stroma
C. Sex cords
D. Germ cells
E. Metastatic disease

96. An 18-year-old nulligravid woman presents to the student health clinic with a 4-week
history of yellow vaginal discharge. She also reports vulvar itching and irritation. She is
sexually active and monogamous with her boyfriend. They use condoms inconsistently.
On physical examination, she is found to be nontoxic and afebrile. On genitourinary

REVISED BY TEAM “BANG JAGO” NOV 2020 UNHAS Page 255


examination, vulvar and vaginal erythema is noted along with a yellow, frothy,
malodorous discharge with a pH of 6.5. The cervix appears to have erythematous
punctuations. There is no cervical, uterine, or adnexal tenderness. The addition of 10%
KOH to the vaginal discharge does not produce an amine odor. Wet prep microscopic
examination of the vaginal swabs is performed. What would you expect to see under
microscopy? ( blueprint)
A. Branching hyphae
B. Multinucleated giant cells
C. Scant WBC
D. Flagellated, motile organisms
E. Epithelial cells covered with bacteria

97. A 25 year old lady come with abnormal pap smear result. She underwent colposcopy
examination and the result is a acetowhite lesion with punctation and atypical vessels.
Biopsy result confirms CIN I with HPV DNA test positve. What do you suggest for
patient ?
A. LEEP procedure
B. Reevaluation of HPV DNA
C. Cold knife conization
D. Repeat cytology in 12 months
E. Repeat cytology in 6 months

98. A 16 year old girl presents with primary amenorrhea. Her breast development is Tanner
stage 2. Pubic and axillary hair show stage I development. The girl appears otherwise
well. Ultrasound show the presence of a normal uterus and tubes. Her height is 140 cm.
From these clinical syndromes, what diagnose suitable for that girl?
A. Patau syndrome
B. Edward syndrome
C. Turner syndrome
D. Down syndrome
E. Klinefelter syndrome

REVISED BY TEAM “BANG JAGO” NOV 2020 UNHAS Page 256


99. What is the implantation of a placenta in which there is a defect in the fibrinoid layer at
the implantation site, allowing the placental villi to invade and penetrate into but not
through the myometrium called?
A. Placenta increta
B. Placenta percreta
C. Placenta previa
D. Placenta infarct
E. Placenta accreta
100. A 25 years old women Para 1,6 weeks after delivery. She wants to use contraception. She
is asking about the oral contraceptive pills.
The mechanism of action of oral contraceptive pills is:
A. Inhibiting ovulation by suppression of serum FSH
B. Inducing endometrial atrophy
C. Inducine endometritis
D. Increasing cervical mucous hostility
E. Inhibiting prolactin

101. A 32 year old woman has a pelvic ultrasound that bilateral 5cm “kissing” ovarian cysts in
the pouch of Douglas. Both of which contain diffuse, low level echous giving a solid
ground-glass appearances. She reports severe dysmenorrhea and dyspareunia.
Which of the following condition that can be found associated with this finding?
A. Adenomyosis in the posterior uterus
B. Increased level of serum AMH
C. Increased level of He4
D. Fifty percent risk of malignant transformation
E. Normal level of Ca 125

102. A 26 year old woman complain of reccurent bouts of bacterial vaginosis (BV) despite
successful initial treatment. She does not douche or smoke and has been in a
monogamous relationship or 6 years. Recurrence of BV after initial treatment is common
(up to 30 percent), which can be frustrating of the patient. Which of the following
consistently decreases recurrence rates and should be recommended to this patient?
A. No intervention consistently decreases recurrence
B. Treatment of long continuous antibiotics
C. Use of acidiying vaginal gels
D. Probiotics and reintroduction of lactobacilli
E. Treatment of male partners

103. Which of the following is true regarding injury related to trocar insertion in laparoscopy?
A. Optical trocar insertion has the lowest rate of injury
B. Hasson technique has the lowest injury
C. Closed entry using voross needle has the highest rate of injury

REVISED BY TEAM “BANG JAGO” NOV 2020 UNHAS Page 257


D. Open umbilical entry has the lowest rate of injury
E. All technique have similar rate of injury
104. A 35 year old P3 with a positive high risk HPV on DNA testing, and a Pap smear showing
high-grade squamous intraepithelial lesion of the cervix (CIN III) has an inandequate
colposcopy.
If the histopathology result shows squamous cell cancer that has invaded only 1 mm
beyond the basement membrane. There are no confluent tongues of tumor, and there is
no evidence of lymphatic or vascular invasion. The margins of the biopsy specimen are
free of disease.
How would you classify or stage the patient’s disease?
A. Atypical squamous cells of undetermined significance
B. Carcinoma in situ
C. Invasive cancer, stage Ia
D. Microinvasive cancer
E. Carcinoma of low malignant potential

REVISED BY TEAM “BANG JAGO” NOV 2020 UNHAS Page 258


AGUSTUS 2020
1. A 27-year-old woman presents to your office with a positive home pregnancy test and a
3-day history of vaginal bleeding. She is concerned that she may be having a miscarriage.
On examination, the uterine fundus is at the level of the umbilicus. By her last period,
she should be around 8 weeks gestation. On pelvic examination, there is a moderate
amount of blood and vesicle-like tissue in the vaginal vault, and the cervix is closed. The
lab then calls you to say that her serum β-hCG result is greater than 1,000,000 mIU/mL.
The patient undergoes an uncomplicated suction D&C. The pathology report is available
the next day and is consistent with a complete molar gestation. What is the best next
step in the care of this patient’s condition.
a. Repeat pelvic imaging
b. Radiation therapy
c. Chemotherapy
d. Surveillance of serum β-hCG
e. No further follow-up is required

REVISED BY TEAM “BANG JAGO” NOV 2020 UNHAS Page 259


REVISED BY TEAM “BANG JAGO” NOV 2020 UNHAS Page 260
2. A 25-year-old woman underwent a uterine curettage after a miscarriage and has had no
menses since. Pregnancy test was negative. Intrauterine adhesions were suspected.
Which of the following is associated with asherman syndrome?
f. Associated with low cortisol levels
g. Associated with low estradiol levels
h. It is associated with class 1 anovulation
i. Presence of monthly LH surge
j. Associated with a positive estrogen-progestin test

3. What type of cyst that frequently associated with ovarian torsion?


a. Ovarian fibroma
b. Follicular cyst
c. Dermoid cyst  Ada komponen padat
d. Endometrioma
e. Mucinous cyst

4. A 26-year-old woman complains of recurrent bouts of bacterial vaginosis (BV) despite


successful initial treatment. She does not douche or smoke and has been in a
monogamous relationship or 6 years. Recurrence of BV after initial treatment is common
(up to 30 percent), which can be frustrating of the patient. Which of the following
consistently decreases recurrence rates and should be recommended to this patient?
f. Treatment of male partners

REVISED BY TEAM “BANG JAGO” NOV 2020 UNHAS Page 261


g. Use of acidiying vaginal gels
h. Probiotics and reintroduction of lactobacilli
i. No intervention consistently decreases recurrence
j. Treatment of long continous antibiotic

5. A 82 years old woman P6 came to outpatient clinic with chief complaint of bulging mass
protrudes from vagina since 3 months ago. The mass usually occurs during activity and
also when she defecate , and disappear when lying down. There were no difficulty in
voiding and defecation. No urinary leakage during coughing and sneezing. She is not
sexually active. If on the Pelvic Organ Prolapse Quantification examination result showing
below, what is the diagnosis of this patient?

Aa Ba C

+3 +4 +5
GH Pb TVL

5 2 8
Ap Bp D

0 0 +3

a. Uterine prolapse grade 2, cystocele grade 2, rectocele grade 1


b. Uterine prolapse grade 4, cystocele grade 3, rectocele grade 2
c. Uterine prolapse grade 3, cystocele grade 3, rectocele grade 2
d. Uterine prolapse grade 3, cystocele grade 2, rectocele grade 2
e. Uterine prolapse grade 4, cystocele grade 4, rectocele grade 3

REVISED BY TEAM “BANG JAGO” NOV 2020 UNHAS Page 262


6. A 26-year-old G2P1 (no live child) is seen for her first prenatal visit at 18 weeks’ gestation
by menstrual history. Her first child was born at 28 weeks spontaneously, the baby was
died after hospitalized for 1 month in NICU. She is worried this pregnancy also will be
ended with spontaneous preterm birth.What is the most accurate examination that can
be done at 18 weeks to predict the risk of preterm birth?
a. Cell-free fetal DNA  First trimester loss
b. IGFBP-1 examination  PPROM
c. PAMG-1 examination  PPROM
d. Fibronectin examination  prediksi preterm labor, kalau sudah ada tanda inpartu
e. Measure cervical length

7. A 23 year-old G1 32 weeks is being admitted to the hospital because of preterm


contraction. The patient complaint regular contraction. Antenatal care was done
regularly in PHC. No remarkable abnormality was found during ANC. BMI before
pregnancy was 30 kg/m2, weight gain during pregnancy is 14 kg. Abdominal examination
showed FUT 36 cm, regular contractions, fetal heart beats 154 bpm. Speculum
examination showed closed ostium uteri externa. An ultrasound shows the estimated
fetal weight 2400 g, AFI 30 cm, no fetal morphology abnormalities, placenta implanted in
anterior corpus, cervical length 1.8 cm, funneling positive. Laboratory results were Hb
10,7 g/dL, Ht 33%, Leucocyte 13.500, Thrombocyte 315.000 MCV 82 MCH 30. What is the
most likely etiology of preterm contraction in this case?
a. Maternal obesity
b. Maternal anemia
c. Bacterial vaginosis
d. Uterine overdistention
e. Excessive gestational weight gain

8. A 23 year-old G1 32 weeks is being admitted to the hospital because of preterm


contraction. The patient complaint regular contraction. Antenatal care was done

REVISED BY TEAM “BANG JAGO” NOV 2020 UNHAS Page 263


regularly in PHC. No remarkable abnormality was found during ANC. BMI before
pregnancy was 30 kg/m2, weight gain during pregnancy is 14 kg. Abdominal examination
showed FUT 36 cm, regular contractions, fetal heart beats 154 bpm. Speculum
examination showed closed ostium uteri externa. An ultrasound shows the estimated
fetal weight 2400 g, AFI 30 cm, no fetal morphology abnormalities, placenta implanted in
anterior corpus, cervical length 1.8 cm, funneling positive. Laboratory results were Hb
10,7 g/dL, Ht 33%, Leucocyte 13.500, Thrombocyte 315.000 MCV 82 MCH 30. What is the
most appropriate next step in the management of this patient?
a. Give intravenous iron
b. Schedule for OGTT test
c. Give antibiotic prophylaxis
d. Schedule for cervical cerclage
e. Give MgSO4 for neuroprotection

9. A 18-year-old G1 at 30 4/7 weeks presents for her scheduled obstetric (OB) appointment.
A 28-week ultrasound showed the fetus to be in the 13th percentile for estimated fetal
weight. The patient denies any complaints today. Fetal movement is active. BMI before
pregnancy was 19.6 kg/m2, gestational weight gain is 7 kg. Vital sign are normal.
Abdominal examination today shows a gravid uterus measuring 27 cm. Fetal heart tones
(FHTs) are in the 140s. Laboratory results were Hb 10,1 g/dL, Ht 30%, Leucocyte 10.500,
Thrombocyte 215.000, MCV 78, MCH 28. OGTT result was fasting 92 mg/dL and 2 hours
after 75 glucose 148 mg/dL. What is the appropriate next step in the management of
this patient?
f. Schedule for serum ferritin test
g. Perform fetal growth ultrasound
h. Schedule for a biophysical profile (BPP)
i. Give elemental iron 100-200 mg per oral
j. Admit patient to the hospital for lung maturation

REVISED BY TEAM “BANG JAGO” NOV 2020 UNHAS Page 264


10. A 18-year-old G1 at 30 4/7 weeks presents for her scheduled obstetric (OB) appointment.
A 28-week ultrasound showed the fetus to be in the 13th percentile for estimated fetal
weight. The patient denies any complaints today. Fetal movement is active. BMI before
pregnancy was 19.6 kg/m2, gestational weight gain is 7 kg. Vital sign are normal.
Abdominal examination today shows a gravid uterus measuring 27 cm. Fetal heart tones
(FHTs) are in the 140s. Laboratory results were Hb 10,1 g/dL, Ht 30%, Leucocyte 10.500,
Thrombocyte 215.000, MCV 78, MCH 28. OGTT result was fasting 92 mg/dL and 2 hours
after 75 glucose 148 mg/dL. What is the most likely predisposition factor for this case?
f. Maternal anemia
g. Genetic abnormalities
h. Placental insufficiency
i. Inadequate maternal nutrition (Peningkatan BB N 11-16 kg)
j. Pre-gestational diabetes mellitus

11. A 35-year-old woman, G3P2, presents to labor and delivery (L&D) at 33-week gestation
referred by midwife with BP 180/110 mmHg. BP on arrival is 170/105 mmHg. Urine
protein is 1+ on dipstick. Patient had history of high blood pressure in her previous
pregnancy. ANC was done in midwife. Blood pressure at first trimester was 130-145/90-
95 mmHg, urine protein was negative on dipstick. No antihypertension drug was given.
The patient denies any complaints today. What is the most likely diagnosis of the patient:
a. Preeclampsia
b. Chronic hypertension
c. Gestational hypertension
d. Superimposed preeclampsia
e. Preeclampsia with severe feature

12. Mrs A, 26-years-old, G1P0A0, according to her last LMP is 34 weeks pregnant, came for
her first antenatal care. She said that she had 20 kg of weight gain during her pregnancy
with swelling ankles for the past 4 weeks. She never took any iron or vitamin

REVISED BY TEAM “BANG JAGO” NOV 2020 UNHAS Page 265


supplementation. From the physical findings, BP 145/95 mmHg, HR 86x/min, RR
20x/min, BMI 35 kg/m2. Ultrasound examination confirmed twins in breech presentation.
Results from urinalysis were as follows: color cloudy yellow, spesific gravity 1.013,
albumin 2+, RBC 0 - 1, WBC 2-5, negative bacterial count. What is the most likely
diagnosis?
A. Acute fatty necrosis of the liver
B. Chronic hypertension
C. Preeclampsia
D. Renal disease
E. Pyelonephritis

13. Mrs. B, 37-years-old came to your office at 32 weeks of gestation according to her last
menstrual period. She has no ultrasound examination before and did not do her routine
antenatal care. The vital sign is within normal limit. She has body mass index 19 kg/m2.
During physical examination, the uterine fundal height is 22 cm. From ultrasound
examination, the fetus has biometric values that correlate with 30 weeks fetus. Which of
the following is the next best step in managing this patient?
A. Antenatal care routinely for the next 2 weeks
B. Evaluate maternal status and comorbidities
C. Consider deliver the baby
D. Repeat sonography for fetal growth in 2 weeks
E. Doppler velocimetry evaluation every 3 days

14. Mrs E, 32 yo referred from midwife with antepartum hemorrhage. She is G3P2 term
pregnancy. On examination her blood pressure is 160/100 mmHg, HR 100 bpm. She looks
anemic, not icteric. Obstetrical examinations reveal contraction 4-5x/10 minutes, FHR
170 bpm, head presentation 3/5. After thorough examination it is concluded that there
is a placental abruption with retroplacental hematoma size 6x5 cm. This patient is

REVISED BY TEAM “BANG JAGO” NOV 2020 UNHAS Page 266


planned to do cesarean section. If during operation the uterus is couvelaire but with
good contraction, how would you manage that condition?
F. Perform prophylactic b-lynch suture
G. Ascending uterine artery ligation
H. Hypogastric artery ligation
I. Sub total hysterectomy
J. Uterotonic and observation

15. During a routine return OB visit, an 18-year-old G1P0 patient at 23 weeks gestational age
undergoes a urinalysis. The dipstick done by the nurse indicates the presence of trace
glucosuria. All other parameters of the urine test are normal. Which of the following is
the most likely etiology of the increased sugar detected in the urine?
F. The patient has diabetes

REVISED BY TEAM “BANG JAGO” NOV 2020 UNHAS Page 267


G. The patient has a urine infection
H. The patient’s urinalysis is consistent with normal pregnancy
I. The patient’s urine sample is contaminated
J. The patient has kidney disease

16. A maternal fetal medicine specialist is consulted and performs an indepth sonogram. The
sonogram indicates that the fetuses are both male, and the placenta appears to be
diamniotic and monochorionic. Twin B is noted to have oligohydramnios and to be much
smaller than twin A. In this clinical picture, all of the following are concerns for twin A
except
A. Congestive heart failure
B. Anemia
C. Hypervolemia
D. Polycythemia
E. Hydramnion

17. You are called in to evaluate the heart of a 19-year-old primigravida at term. Listening
carefully to the heart, you determine that there is a split S1, normal S2, S3 easily audible
with a 2/6 systolic ejection murmur greater during inspiration, and a soft diastolic
murmur. You immediately recognize that
A. The presence of the S3 is abnormal
B. The systolic ejection murmur is unusual in a pregnant woman at term
C. Diastolic murmurs are rare in pregnant women
D. The combination of a prominent S3 and soft diastolic murmur is a significant
abnormality
E. All findings recorded are normal changes in pregnancy
\

18. A 30-year-old multiparous woman has rapid delivery soon after arriving in emergency
room. After delivery the placenta she is noted to have heavy vaginal bleeding. Help has

REVISED BY TEAM “BANG JAGO” NOV 2020 UNHAS Page 268


been summoned. Abdominal examination demonstrates the fundus was soft. What is the
most appropriate next step?
A. Intravenous access for fluid resuscitation
B. Uterine packing
C. Balloon tamponade
D. Suture the laceration
E. Misoprostol administration

19. What is the most common cause of heart failure during pregnancy and the puerperium?
A. Chronic hypertension with severe preeclampsia
B. Viral myocarditis
C. Obesity
D. Valvular heart disease
E. Pulmonary Artery Hypertension

20. For patients with congenital heart disease, what is the most common adverse
cardiovascular event encountered in pregnancy?
A. Heart failure
B. Arrhythmia
C. Thromboembolic event
D. Cerebrovascular hemorrhage
E. Heart axis changes

21. A 24-year-old patient, P2, has just delivered vaginally an infant weighing 3000 g after a
spontaneous uncomplicated VBAC. Her prior obstetric history was a low uterine segment
transverse cesarean section for breech. She has had no problems during the pregnancy

REVISED BY TEAM “BANG JAGO” NOV 2020 UNHAS Page 269


and labor. The placenta delivers spontaneously. There is immediate vaginal bleeding of
greater than 500 cc. Although all of the following can be the cause for postpartum
hemorrhage, which is the most frequent cause of immediate hemorrhage as seen in this
patient?
A. Uterine atony
B. Coagulopathies
C. Uterine rupture
D. Retained placental fragments
E. Vaginal and/or cervical lacerations

22. A 22-year-old G1A0 at 10 weeks presents for her scheduled obstetric (OB) appointment.
Laboratory examination showed hemoglobin 11.2 g/dL, hematocrit 34%, MCV 86 fl MCH
32 pg. Which of the following shows iron deficiency anemia:
A. Low ferritin, normal serum iron
B. Low ferritin, serum iron and transferrin
C. Normal hemoglobin level, low ferritin and serum iron
D. Microcytic hypochromic, low serum iron and transferrin saturation
E. Microcytic hyperchromic, low serum iron and transferrin saturation

23. What is total iron need during pregnancy?


A. 500 mg
B. 750 mg
C. 1000 mg
D. 1500 mg
E. 2000 mg

24. How much iron is recommended by WHO for pregnant women in Indonesia? (prevalence
of anemia in Indonesia based on RISKESDAS 48%)
A. 27 mg

REVISED BY TEAM “BANG JAGO” NOV 2020 UNHAS Page 270


B. 30 mg
C. 60 mg
D. 100 mg
E. 200 mg

25. A 22-year-old primiparous woman presents for her first prenatal evaluation. On physical
examination you hear a grade 3/6 pansystolic murmur. Which is the most common CHD
in pregnancy that would cause that type of murmur?
A. Aortic stenosis
B. Pulmonary stenosis
C. Atrial septal defect (ASD)
D. Patent ductus arteriosus (PDA)
E. Ventricular septal defect (VSD)

26. Numerous physiologic changes develop over the course of pregnancy; however the
greatest impact on potentially compromised cardiovascular system is:
A. The red cell mass rises by 40%
B. Increased 30% total plasma volume in 28 weeks
C. Increase in cardiac output occurs by 24 weeks
D. Decreased SVR in the first trimester, and increase after 32 weeks
E. Cardiac output increases to 30-50% above pre-pregnancy levels by the end of the third
trimester

27. A 28-year-old G1 at 26 weeks present for her scheduled obstetric appointment. You
ordered OGTT examination that shows fasting blood glucose 102 mg/dL and 2 hours after
75 g oral glucose 185 mg/dL. Her gestational weight gain during pregnancy is 12 kg. Her

REVISED BY TEAM “BANG JAGO” NOV 2020 UNHAS Page 271


BMI before pregnancy was 26 kg/m2. What is the appropriate next step in the
management of this patient?
A. Schedule nonstress test (NSTs)
B. Schedule fetal growth ultrasound
C. Admit to hospital for fetal monitoring
D. Advise insulin in order to lowering blood glucose
E. Schedule routine antenatal care in 4 weeks

28. A 28-year-old G1 at 26 weeks present for her scheduled obstetric appointment. You
ordered OGTT examination that shows fasting blood glucose 102 mg/dL and 2 hours after
75 g oral glucose 185 mg/dL. Her gestational weight gain during pregnancy is 12 kg. Her
BMI before pregnancy was 26 kg/m2. What is the most likely fetal consequences cause by
patient’s condition?
A. Spina bifida
B. Fetal hypoxia
C. Intrauterine growth restriction
D. Fetal large for gestational age
E. Congenital valvular heart disease

29. A 28-year-old G2P1 is seen for her first prenatal visit at 16 weeks’ gestation by menstrual
history. Her first child was born at 32 weeks spontaneously. She is worried this
pregnancy also will be ended with spontaneous preterm birth. What is the most
accurate examination that can be done at 16 weeks to predict the risk of preterm birth?
A. IGFBP-1 examination
B. Fibronectin examination
C. Measure cervical length
D. Vaginal swab to exclude bacterial vaginosis
E. Urinary test to exclude urinary tract infection

REVISED BY TEAM “BANG JAGO” NOV 2020 UNHAS Page 272


30. What is the next appropriate management for the patient?
A. Progesterone prophylaxis
B. Schedule for cervical cerclage
C. Treat asymptomatic bacterial vaginosis
D. Give tocolytics for inhibiting preterm labor
E. Intervention modifiable risk factors for preterm

31. What is the implantation of a placenta in which there is a defect in the fibrinoid layer at
the implantation site, allowing the placental villi to invade and penetrate into but not
through the myometrium called?
A. Placenta accreta
B. Placenta increta
C. Placenta percreta
D. Placental infarct
E. Placenta previa

32. A 21-year-old G1 P0 patient presents to your office with vaginal bleeding at


approximately 8 weeks’ gestation by her last menstrual period. Her examination is
benign with a 9-week-sized uterus, a closed cervical os, and a small amount of blood
within the vaginal vault. You order a complete pelvic ultrasound that shows an
intrauterine gestational sac containing a fetus measuring approximately 7 weeks’
gestation. Doppler sonography is unable to demonstrate any fetal heartbeat. What is the
most likely diagnosis?
A. Ectopic pregnancy
B. Complete abortion
C. Embryonic demise (Missed ab)
D. Incomplete abortion
E. Threatened abortion

REVISED BY TEAM “BANG JAGO” NOV 2020 UNHAS Page 273


33. A 21-year-old G1 P0 patient presents to your office with vaginal bleeding at
approximately 8 weeks’ gestation by her last menstrual period. Her examination is
benign with a 9-week-sized uterus, a closed cervical os, and a small amount of blood
within the vaginal vault. You order a complete pelvic ultrasound that shows an
intrauterine gestational sac containing a fetus measuring approximately 7 weeks’
gestation. Doppler sonography is unable to demonstrate any fetal heartbeat. You decide
to perform a suction D&C. When giving informed consent, you discuss the risk most
commonly encountered in this operation. Which of the following is the most common
risk associated with suction D&C?
A. Infection
B. Uterine perforation
C. Damage to the bladder
D. Uterovaginal bleeding
E. Need for future surgery

REVISED BY TEAM “BANG JAGO” NOV 2020 UNHAS Page 274

You might also like